Sie sind auf Seite 1von 182

M05

Prfung A

Frage 1
Folgende Aussage(n) ber die bewute Sensibilitt trifft/treffen zu:

1. Der periphere Neurit des ersten Neurons reicht bis zum Spinalganglion.
2. Das erste Neuron schaltet im Spinalganglion um.
3. Das zweite Neuron bildet den Lemniscus medialis.
4. Das dritte Neuron verluft im Pedunculus thalami anterior.

A. 1, 2 und 3 sind richtig


B. 1 und 3 sind richtig
C. 2 und 4 sind richtig
D. nur 4 ist richtig
E. alle sind richtig

Richtige Antwort: B

Frage 2
Was ist kein Teil des Corpus callosum?

A. Splenium
B. Crus
C. Rostrum
D. Genu
E. Truncus

Richtige Antwort: B

Frage 3
Welche der folgenden Kombinationen ist nicht korrekt?

A. Hippocampus - Telencephalon
B. Hypothalamus - Diencephalon
C. Corpus amygdaloideum - Diencephalon
D. Nucleus caudatus - Telencephaon
E. Corpora mammilaria - Diencephalon

Richtige Antwort: C

Frage 4
Der N. thoracicus longus:

1. gelangt ventral des Plexus brachialis zur Achselhhle


2. entsteht aus dem Plexus cervicalis
3. versorgt den M. latissimus dorsi
4. versorgt den M. serratus anterior

A. 1, 2 und 3 sind richtig


B. 1 und 3 sind richtig
C. 2 und 4 sind richtig
D. nur 4 ist richtig
E. alle sind richtig

Richtige Antwort: D

Frage 5
Zu den Kommissuralen Fasern gehren:
A. Radiatio thalami
B. Cingulum
C. Tractus corticospinalis
D. Corpus callosum
E. Fasciculus longitudinalis sup.

Richtige Antwort: D

Frage 6
Der N. dorsalis scapulae versorgt:

A. den M. scalenus medius.


B. den M. sternocleidomastoideus.
C. den M. levator scapulae und die Mm. rhomboidei.
D. den M. trapezius.
E. die Haut an der Dorsalseite der Scapula.

Richtige Antwort: C

Frage 7
Steuerung horizontaler Augenbewegungen - Welche Antworten sind richtig:

1. die Abduzenskerne sind kreuzweise mit den Okulomotoriuskernen verschaltet


2. die lateralen Augenmuskeln werden von den Okulomotiuskernen innveriert
3. Lsion des medialen Lngsbndels fhrt zur Diskonnektion der medialen Augenmuskeln
4. Erhhung der Impulsrate im linken horizontalen Bogengang fhrt zu einer Aktivierung im
ipsilateralen Abduzenskern

A. 1, 2 und 3 sind richtig


B. 1 und 3 sind richtig
C. 2 und 4 sind richtig
D. nur 4 ist richtig
E. alle sind richtig

Richtige Antwort: B

Frage 8
Der N. vagus innerviert nicht:

A. Die Muskulatur des Kehlkopfs


B. Die Schleimhaut der Kehlkopfs
C. Die Muskulatur der Speiserhre
D. Die Eingeweide des Oberbauchs
E. Die Beckeneingeweide

Richtige Antwort: E

Frage 9
Welche Aussage zu folgender Abbildung trifft zu?
A. 1 zeigt die Radiatio acustica
B. 2 zeigt den Thalamus
C. 3 zeigt den Tractus temporooccipitopontinus
D. 3 nimmt ihren Ursprung vom Gyrus praecentralis
E. 1 leitet die gesamte Krpersensibilitt

Richtige Antwort: D

Frage 10
Cerebellre Lsionen bewirken:

1. eine Dysmetrie
2. eine Asynergie
3. eine Ataxie
4. einen Ruhetremor

A. 1, 2 und 3 sind richtig


B. 1 und 3 sind richtig
C. 2 und sind richtig
D. nur 4 ist richtig
E. alle sind richtig

Richtige Antwort: A

Frage 11
Welche Aussage zum Diencephalon ist richtig?

A. Der Subthalamus ist bei der Regulation vegetativer und hormoneller Reaktionen beteiligt
B. Epithalamus, Hypothalamus und Corpora mammilaria gehren zum Diencephalon
C. Der Epithalamus liegt unterhalb des III. Ventrikels
D. Corpus geniculatum med. et lat. haben direkte Verbindung mit dem Hypothalamus
E. Die Hypophyse unterliegt der Steuerung des Thalamus

Richtige Antwort: B

Frage 12
Welche Struktur in der Abbildung ist falsch bezeichnet:
A. 2 zeigt das Splenium corporis callosi
B. 5 zeigt den Ventriculus quartus
C. 1 zeigt die Adhaesio interthalamica
D. 3 zeigt die Hypophyse
E. 4 zeigt die Lamina quadrigemina

Richtige Antwort: D

Frage 13
Folgende Strukturen des Gehirns sind lateral an der Facies convexa sichtbar:

1. Sulcus centralis
2. Gyri temporales transversi (Heschl Querwindung)
3. Gyrus angularis
4. Sulcus calcarinus

A. 1, 2 und 3 sind richtig


B. 1 und 3 sind richtig
C. 2 und 4 sind richtig
D. nur 4 ist richtig
E. alle sind richtig

Richtige Antwort: B

Frage 14
Afferenzen zum Cerebellum:

1. kommen vom Cerebrum.


2. sind Kletterfasern.
3. haben Glutamat als Transmitter.
4. sind Moosfasern.

A. 1, 2 und 3 sind richtig


B. 1 und 3 sind richtig
C. 2 und 4 sind richtig
D. nur 4 ist richtig
E. alle sind richtig

Richtige Antwort: E

Frage 15
Der Lemniscus medialis setzt sich aus folgenden Bahnen zusammen:

1. Tractus nucleothalamicus
2. Tractus spinothalamicus
3. Tractus bulbothalamicus
4. Tractus spinoreticularis

A. 1, 2 und 3 sind richtig


B. 1 und 3 sind richtig
C. 2 und 4 sind richtig
D. nur 4 ist richtig
E. alle sind richtig

Richtige Antwort: A

Frage 16
Zum Frontallappen gehrt/gehren:

1. Gyrus praecentralis
2. Gyri orbitales
3. Gyrus frontalis inferior
4. Gyrus rectus

A. 1, 2 und 3 sind richtig


B. 1 und 3 sind richtig
C. 2 und 4 sind richtig
D. nur 4 ist richtig
E. alle sind richtig

Richtige Antwort: E

Frage 17
Welche Muskeln werden vom N. musculocutaneus versorgt?
1. M. biceps brachii
2. M. brachialis
3. M. coracobrachialis
4. M. deltoideus

A. 1, 2 und 3 sind richtig


B. 1 und 3 sind richtig
C. 2 und 4 sind richtig
D. nur 4 ist richtig
E. alle sind richtig

Richtige Antwort: A

Frage 18
Plexus brachialis gliedert sich in:

1. eine Pars suprascapularis


2. eine Pars supraclavicularis
3. eine Pars infrascapularis
4. eine Pars infraclavicularis
A. 1, 2 und 3 sind richtig
B. 1 und 3 sind richtig
C. 2 und 4 sind richtig
D. nur 4 ist richtig
E. alle sind richtig

Richtige Antwort: C

Frage 19
Welche Struktur wird in der folgenden Abbildung mit Ziffer 2 bezeichnet?

A. Pars centralis (Cella media) des Ventriculus lateralis


B. Capsula externa
C. Capsula extrema
D. Capsula interna
E. Corpus callosum

Richtige Antwort: D

Frage 20
Der N. trochlearis verlt die Schdelhhle durch:

A. das Foramen ovale


B. das Foramen rotundum
C. die Fissura orbitalis superior
D. die Fissura orbitalis inferior
E. das Foramen jugulare

Richtige Antwort: C

Frage 21
Welche der folgenden Aussagen ist richtig?

A. Der Nucleus nervi trochlearis ist ein somatomotorischer Kern und entsendet Fasern fr den M. rectus
lat.
B. Der Nucleus nervi hypoglossi empfngt gescchmacksfasern aus der Zunge.
C. Der Nucleus nervi abducens ist ein visceromotorischer Kern und entsendet Fasern fr den M. obliquus
bulbi superior.
D. der Nucleus nervi hypoglossi entsendet motorische fasern fr die Zungenmuskulatur
E. Der Nucleus centralis PERLIA gibt seine Fasern dem N. trochlearis mit

Richtige Antwort: D

Frage 22
Welche Aussage zur folgenden Abbildung ist nicht richtig?

A. 3 besteht aus einem medialen und einem lateralen Segment


B. 3 zeigt das Putamen
C. 1 ist sowohl bei der Bearbeitung motorischer Programme, als auch bei der Verarbeitung sensorischer
Informationen wichtig
D. Die Abbildung stellt einen Frontalschnitt des Gehirns dar
E. 2 gehrt zur inneren Kapsel
Richtige Antwort: B

Frage 23
Wer wird vom N. gluteus superior nicht versorgt?

1. M. gluteus medius
2. M. tensor fasciae latae
3. M. glutaeus minimus
4. M. gluteus maximus

A. 1, 2 und 3 sind richtig


B. 1 und 3 sind richtig
C. 2 und 4 sind richtig
D. nur 4 ist richtig
E. alle sind richtig

Richtige Antwort: D

Frage 24
Auf welche Struktur weist der Pfeil in der folgenden Abbildung?
A. Glomus caroticum
B. Plexus choroideus des III. Ventrikels
C. Plexus choroideus des Unterhorns des Seitenventrikels
D. Glomus choroideum
E. Vorderhorn des Seitenventrikels

Richtige Antwort: D

Frage 25
Welche der folgenden Strukturen gehrt/gehren zum Gleichgewichtsorgan?

1. Maccula sacculi
2. Cupula
3. Statholithenmembran
4. Membrana tectoria

A. 1, 2 und 3 sind richtig


B. 1 und 3 sind richtig
C. 2 und 4 sind richtig
D. nur 4 ist richtig
E. alle sind richtig

Richtige Antwort: A

Frage 26
Welche Struktur wird in der folgenden Abbildung mit der Ziffer 3 bezeichnet?
A. Nucleus lentiformis
B. Caput nuclei caudati
C. Nucleus subthalamicus
D. Thalamus
E. Corpus geniculatum mediale

Richtige Antwort: D

Frage 27
Welche der folgenden Aussagen ist nicht richtig?

A. Die Rautengrube bidet den Boden des IV. Ventrikels.


B. Die Rautengrube wird durch die Striae medullares ventriculi quarti in einen met- und einen
myelencephalen Teil gegliedert.
C. Der Sulcus medianus teilt die rautengrube in vertikaler Richtung in zwei Hlften.
D. In der Rautengrube findet man u.a. den Colliculus facialis. Er wird durch den Nucleus nervi facialis
aufgeworfen.
E. Die Rautengrube wird nach oben hin durch die Pedunculi carebellaris superiores begrenzt.

Richtige Antwort: D

Frage 28
Der Tractus spinothalamicus anterior:

1. enthlt Fasern aus Haut, Eingeweiden und Muskeln


2. leitet grobe Druck- und Tastempfindungen
3. kreuzt in der Commissura alba anterior medullae spinalis
4. schaltet im Spinalganglion um

A. 1, 2 und 3 sind richtig


B. 1 und 3 sind richtig
C. 2 und 4 sind richtig
D. nur 4 ist richtig
E. alle sind richtig

Richtige Antwort: A

Frage 29
Zur Hhrbahn zhlen folgende Strukturen:

1. Kerne des Lemniscus lateralis


2. Colliculi inferiores der Lamina tecti
3. Corpus trapezoideum
4. Corpus geniculatum mediale

A. 1, 2 und 3 sind richtig


B. 1 und 3 sind richtig
C. 2 und 4 sind richtig
D. nur 4 ist richtig
E. alle sind richtig

Richtige Antwort: E

Frage 30
Welche Aussage zur folgenden Abbildung ist richtig?
A. Der Rckenmarksquerschnitt stammt aus der Sakralregion
B. Die dirsalen bereiche erkennt man unten im Bild, die ventral gelegenen oben
C. 3 stellt das Hinterhorn dar
D. 2 trgt die Ursprungsneurone vegetativer Fasern
E. 1 stellt das Vorderhorn dar

Richtige Antwort: D

Frage 31
Welche Bahn ist mit der dunklen Linie markiert?
A. Tractus spinothalamicus
B. Tractus corticospinalis
C. Lemniscus medialis
D. Fasciculus gracilis
E. Tractus bulbothalamicus

Richtige Antwort: B

Frage 32
Welche Struktur durchbricht der R. profundus n. radialis, um an die Streckseite des Unterarms zu
gelangen?

A. den M. brachioradialis
B. den M. pronator teres
C. den M. extensor carpi radialis longus
D. den M. supinator
E. die Membrana interossea cruris antebrachii

Richtige Antwort: D

Frage 33
Welche der folgenden Aussagen ist richtig?

A. Den metencephalen Anteil des Hirnstamms bilden die Crur cerebri.


B. Im Fu des Hirnstames steigen die afferenten Bahnen auf.
C. Im Fu des Hirnstammes findet man die neencephalen motorischen Bahnen.
D. Die Pyramiden sind der mesencephale Anteil des Pes.
E. Die Pars ventralis pontis bildet den myelencephalen Anteil des Hirnstammfues.

Richtige Antwort: C

Frage 34
Welche Aussage ber die Gefversorgung des ZNS ist falsch?

A. Die A. carotis interna speist den Circulus arteriosus Willisii.


B. Der Circulus arteriosus cerebri umgibt den Hypophysenstiel und das Chiasma opticum.
C. Die Endste der A. basilaris sind die Aa. cerebri med. et post.
D. Die drei groen Aa. cerebri laufen im Subarachnoidalraum.
E. Das sauerstoffreiche Blut kann in der A. communicans post. in beide Richtungen flieen.

Richtige Antwort: C

Frage 35
Der N. tibialis versorgt nicht:

A. die Flexoren im Unterschenkel


B. die Flexoren am Oberschenkel mit Ausnahme des kurzen Bicepskopfes
C. die Mm. interossei pedis.
D. die Mm. peronei (fibulares).
E. die Muskeln der Planta pedis.

Richtige Antwort: D

Frage 36
Welche Struktur ist in folgender Abbildung mit der Ziffer 2 bezeichnet?

A. Thalamus
B. Nucleus caudatus
C. Nucleus lentiformis
D. Nucleus amygdalae
E. Nucleus subthalamicus

Richtige Antwort: C
Frage 37
Auf welche Nerven weisen die beiden Pfeile?

A. N. glossopharyngeus und N. vagus


B. N. vagus und N. accessorius
C. N. trigeminus und n. facialis
D. N. facialis und N. vestibulocochlearis
E. N. vestibulocochlearis und N. abducens

Richtige Antwort: D

Frage 38
Welche Aussage zur folgenden Abbildung ist richtig?
A. 2 ist der Gyrus parahippocampalis
B. 5 ist der gyrus cinguli
C. 3 ist der Pons
D. 4 ist der Lobus parietalis sup.
E. 1 ist der motorische Teil des Lobulus paracentralis

Richtige Antwort: E

Frage 39
Welche Aussage zum Fornix ist nicht richtig?

Er:

A. endet am Corpus mammillare


B. verluft unterhalb des Corpus callosum
C. beginnt mit der Fimbria hippocampi
D. gehrt zum limbischen System
E. begrenzt seitlich den Aqaeductus cerebri

Richtige Antwort: E

Frage 40
Welche Aussagen zu folgender Abbildung trifft zu:
A. 1 zeigt die ffnung des Augenblschens.
B. 2 zeigt eine Extremittenknospe.
C. 1 zeigt ein Foramen interventriculare primitivum.
D. 1 zeigt die ffnung des spteren Aquaeductus cerebri.
E. 3 weist auf das Prosencephalon

Richtige Antwort: C

Frage 41
Welche Aussage ber die ueren Augenuskeln trifft zu?

A. Die ueren Augenmuskeln werden hauptschlich von sympathischen Nerven innerviert.


B. Der M. rectus lat. wird vom N. abducens innerviert.
C. Der 4. Hirnnerv innerviert den M. obliquus inf..
D. Der N. occulomotorius innerviert alle ueren Augenmuskeln, bis auf den M. rectus med. und den M.
obliquus sup..
E. Der motorische Nerv fr die meisten Augenmuskeln ist der N. ophtalmicus.

Richtige Antwort: B

Frage 42
Folgende Aussage ber den Augapfel trifft zu:

A. Der Augapfel hat eine zweischichtige Wand.


B. Im Inneren des Aufapfels befinden sich Trnenflssigkeit, Linse und Glaskrper.
C. Die Tunica vasculosa bulbi wird auch Uvea genannt.
D. Der Sehnerv verlsst den Augapfel am gelben Fleck.
E. Am Discus n. optici befindet sich die Stelle des schrfsten Sehens.

Richtige Antwort: C

Frage 43
Welche Aussage zum Rckenmark ist nicht richtig?

A. Wegen des Ascensus medullae spinalis reicht das Rckenmark beim Erwachsenen bis etwa zum 1.
LWK.
B. Beim Neugeborenen endet das RM auf Hhe des 12. BWK.
C. Die Cauda equina besteht aus Fasern der vorderen und hinteren Wurzeln.
D. Das RM endet mit dem Conus medullaris.
E. Das Filum terminale fixiert das Ende des RM am Ende des Sakralkanals.

Richtige Antwort: B

Frage 44
Welche Aussage trifft zu?

A. Malleus, Incus und Stapes liegen im Innenohr.


B. Zum Innenor gehren Cochlea und Canales semicirculares.
C. Das Innenohr besteht aus luftgefllten Rumen.
D. Der Hr- und Gleichgewichtsnerv verlsst das Innenohr durch den ueren Gehrgang.
E. Die Schnecke ist der Sitz der Gleichgewichtsrezeptoren.

Richtige Antwort: B

Frage 45
Welche Struktur ist mit der Ziffer 5 bezeichnet?

A. Genu corporis callosi


B. Fornix
C. Commissura fornicis
D. Splenium corporis callosi
E. Pulvinar thalami

Richtige Antwort: D

Frage 46
Welche Aussage zum Mandelkern (Corpus amygdaloideum) ist nicht richtig?

Er:

A. ist bei der Verarbeitung olfaktorischer Informationen beteiligt.


B. liegt vor dem Cornu inf. Ventriculus lat..
C. besteht aus mehreren Kerngruppen.
D. hat enge Verbindung zum imbischen System.
E. liegt im Frontallappen

Richtige Antwort: E

Frage 47
Welche der folgenden Aussagen trifft/treffen zu?

Der Plexus choroideus:

1. produziert Liquor cerebrospinalis


2. bildet Zotten
3. ist oberflchlich von Epithel berzogen
4. besitzt viele Blutgefe

A. 1, 2 und 3 sind richtig


B. 1 und 3 sind richtig
C. 2 und 4 sind richtig
D. nur 4 ist richtig
E. alle sind richtig

Richtige Antwort: E

Frage 48
Welche Aussage trifft nicht zu?

Das Trommelfell:

A. bertrgt die Schallwelle auf die Gehrknchelchen.


B. hat normalerweise eine perlgraue Farbe.
C. steht senkrecht zur Achse des knchernen ueren Gehrgangs.
D. trennt das Auenohr vom Mittelohr.
E. liegt etwas weniger als 4cm medial der Ohrmuschel.

Richtige Antwort: C

Frage 49
Was trifft auf den N. ulnaris nicht zu?

A. Er liegt unter dem M flexor carpi ulnaris


B. Er gibt den Ramus dorsalis n. ulnaris ab
C. Er versorgt die Haut an der Beugerseite des Unterarms
D. Er versorgt alle Mm. interossei manus
E. Er versrogt den M. flexor carpi ulnaris und den unteren Teil des M. flexor digitorum profundus
Richtige Antwort: C

Frage 50
Was trifft auf den N. peroneus (fibularis) communis nicht zu?

A. Er versorgt die Flexoren am Unterschenkel


B. Er betritt die Peroneusloge unter dem Caput fibulae
C. Er ist ein Ast des N. ischiadicus
D. Er versorgt am Oberschenkel des Caput breve des M. biceps femoris
E. Er luft an der Innenseite der Sehne des M. biceps femoris in Richtung Fibula

Richtige Antwort: A

Frage 51
Welche Aussage ber den gelben Fleck trifft zu?

A. Am gelben Fleck verlsst der Sehnerv das auge


B. Am gelben Fleck gibt es ausschlielich Stbchen
C. Der gelbe Fleck wird auch Discus n. optici genannt
D. Der gelbe Fleck liegt 8mm medial des blinden Flecks
E. Am gelben Fleck befindet sich die Stelle des schrfsten Sehens

Richtige Antwort: E

Frage 52
Aus dem Augenbecher entsteht der/das:

A. Musculus ciliaris
B. Musculus sphincter pupillae
C. Pigmentepithel
D. Linsenepithel
E. Hornhautendothel

Richtige Antwort: C

Frage 53
Welche Kerngebiete liegen im Prosencephalon?

A. Substantia nigra
B. Corpus amygdaloideum
C. Nucleus ruber
D. Nucleus olivaris sup.
E. Nucleus dorsalis n. vagi

Richtige Antwort: B

Frage 54
Welche der folgenden Aussagen trifft/treffen zu?

Das Cavum tympani:

1. wird durch das Trommelfell vom ueren Gehrgang abgegrenzt


2. wird von einer Schleimhaut ausgekleidet
3. enthlt drei Gehrknchelchen
4. besteht aus einem knorpeligen und einem knchernen Anteil

A. 1, 2 und 3 sind richtig


B. 1 und 3 sind richtig
C. 2 und 4 sind richtig
D. nur 4 ist richtig
E. alle sind richtig

Richtige Antwort: A

Frage 55
Welche der folgenden Aussagen zur Adenohypophyse trifft/treffen zu?

1. Sie wird in ihrer endokrinen Funktion vom Hypothalamus gesteuert.


2. Hier finden sich sinuscide Kapillaren.
3. Sie enthlt chromophobe und chromophile Zellen.
4. Sie entwickelt sich aus dem Ektoderm der Mundbucht.

A. 1, 2 und 3 sind richtig


B. 1 und 3 sind richtig
C. 2 und 4 sind richtig
D. nur 4 ist richtig
E. alle sind richtig

Richtige Antwort: E

Frage 56
Welchen Teil des Ventrikelsystems findet man im Bereich des anatomischen Hirnstamms:

A. Seitenventrikel
B. Aquaeductus cerebri
C. III. Ventrikel
D. Unterhorn
E. Cella media

Richtige Antwort: B

Frage 57
Der N. tibialis versorgt:

1. den M. soleus
2. den M. gastrocnemius
3. den M. semitendinosus
4. den M. sartorius

A. 1, 2 und 3 sind richtig


B. 1 und 3 sind richtig
C. 2 und 4 sind richtig
D. nur 4 ist richtig
E. alle sind richtig

Richtige Antwort: A

Frage 58
Welche Muskeln werden vom N. musulocutaneus versorgt?

1. M. biceps brachii
2. M. brachialis
3. M. coracobrachialis
4. M. deltoideus

A. 1, 2 und 3 sind richtig


B. 1 und 3 sind richtig
C. 2 und 4 sind richtig
D. nur 4 ist richtig
E. alle sind richtig

Richtige Antwort: A

Frage 59
Was trifft auf die Arachnoidea zu?

A. Sie liegt nur im Rckenmarksbereich zwischen Pia und Dura mater


B. Sie ist die harte Hirnhaut
C. Sie endet caudal auf Wirbelhhe L1
D. Sie bildet die Blut-Hirn-Schranke
E. Sie umhllt auch die Radices ventrales und dorsales des RM

Richtige Antwort: E

Frage 60
Was gehrt nicht zum Corpus striatum?

A. Nucleus lentiformis
B. Pallium
C. Cauda nuclei caudati
D. Putamen
E. Globus pallidus

Richtige Antwort: B
Prfung B

Frage 1
Was gehrt nicht zum Mesencephalon?

A. Pars ventralis pontis


B. Lamina tecti
C. Crus cerebri
D. Fossa intercruralis (interpeduncularis)
E. Aqaeuductus cerebri

Richtige Antwort: A

Frage 2
Welche Aussage(n) ber den gelben Fleck trifft/treffen zu?

1. Der gelbe Fleck wird auch Discus nervi optici genannt.


2. Am gelben Fleck gibt es ausschlielich Stbchen.
3. Der gelbe Fleck liegt 8 mm medial des blinden Flecks.
4. Am gelben Fleck befindet sich die Stelle des schrfsten Sehens.

A. 1, 2 und 3 sind richtig


B. 1 und 3 sind richtig
C. 2 und 4 sind richtig
D. nur 4 ist richtig
E. alle sind richtig

Richtige Antwort: D

Frage 3
Die Integration somatosensorischer und visuelle Inputs erfolgt:

1. okzipital
2. in den Limbischen Strukturen
3. in der formatio reticularis
4. parietal

A. 1, 2 und 3 sind richtig


B. 1 und 3 sind richtig
C. 2 und 4 sind richtig
D. nur 4 ist richtig
E. alle sind richtig

Richtige Antwort: D

Frage 4
Der Plexus brachialis wird von Rr. ventrales aus folgenden Segmenten gebildet:

A. C3 - C8
B. C1 - C4
C. C4 - C8
D. C5 - Th1
E. C6 - Th2

Richtige Antwort: D

Frage 5
Wenn man von der Insula aus nach medial in die Hirnsubstanz vordringt gelangt man auf folgende Struktur:
A. Limen insulae
B. Fossa lateralis
C. Mare internum
D. Claustrum
E. Tractus opticus

Richtige Antwort: D

Frage 6
Was gehrt nicht zur Substantia grisea?

A. Capsula interna
B. Cornu ant. (Columna ant.) der Medulla spinalis
C. Substantia grisea centralis
D. Cortex cerebri
E. Cortex cerebelli

Richtige Antwort: A

Frage 7
Am Hirnstamm unterscheidet man von ventral nach dorsal folgende Schichten:

A. Pes - Tegmentum - Tectum


B. Tegmentum - Pes - Tectum
C. Tectum - Pes - Tegmentum
D. Tectum - Tegmentum - Pes
E. Tegmentum - Tectum - Pes

Richtige Antwort: A

Frage 8
Zum Parietallappen gehrt/gehren:

1. Sulcus centralis
2. Gyrus angularis
3. Sulcus calcarinus
4. Gyrus supramarginalis

A. 1, 2 und 3 sind richtig


B. 1 und 3 sind richtig
C. 2 und 4 sind richtig
D. nur 4 ist richtig
E. alle sind richtig

Richtige Antwort: C

Frage 9
Was trifft auf die Trunci des Plexus brachialis nicht zu:

A. Der Truncus sup. entsteht aus den Segmenten C5 und C6


B. Der Truncus inf. entsteht aus den Segmenten C8 und Th1
C. C7 bildet allein den Truncus med.
D. Die Trunci liegen in der vorderen Skalenuslcke
E. Aus den Trunci gehen drei Faszikeln hervor

Richtige Antwort: D

Frage 10
Welche Struktur in der Abbildung ist falsch bezeichnet:

A. 2 zeigt das Splenium corporis callosi


B. 5 zeigt den Ventriculus quartus
C. 4 zeigt die Lamina quadrigemina
D. 1 zeugt die Commissura ant.
E. 3 zeigt das Corpus pineale

Richtige Antwort: D

Frage 11
Die primre Sehrinde:

1. grenzt an den Sulcus calcarinus


2. erhlt Afferenzen aus dem Corpus geniculatum laterale
3. sendet Efferenzen zum Lobus frontalis
4. erstreckt sich hauptschlich ber die Facies convexa des Occipitallappens

A. 1, 2 und 3 sind richtig


B. 1 und 3 sind richtig
C. 2 und 4 sind richtig
D. nur 4 ist richtig
E. alle sind richtig

Richtige Antwort: A

Frage 12
Der Tractus corticonuclearis:

1. entspringt im Gyrus postcentralis.


2. zieht durch die Capsula externa.
3. endet an einer motorischen Vorderhornzelle.
4. verluft durch das Crus cerebri.
A. 1, 2 und 3 sind richtig
B. 1 und 3 sind richtig
C. 2 und 4 sind richtig
D. nur 4 ist richtig
E. alle sind richtig

Richtige Antwort: D

Frage 13
Was gehrt nicht zum inneren Aufbau des Mittelhirns?

A. Nucleus n. occulomotorii
B. Nucleus ruber
C. Lemniscus med. et lat.
D. Nucleus ambiguus
E. Substantia nigra

Richtige Antwort: D

Frage 14
Welche Aussage zur folgenden Abbildung ist richtig?

A. 2 = Gyrus parahippocampalis
B. 1 = Gyrus frontalis sup.
C. 5 = Genu corporis callosi
D. 2 = Gyrus precentralis
E. 3 = Pons

Richtige Antwort: C

Frage 15
Zur Hrbahn zhlen folgende Strukturen:
1. Kerne des Lemniscus lateralis
2. Colliculi inferiores der Lamina tecti
3. Corpus trapezoideum
4. Corpus geniculatum mediale

A. 1, 2 und 3 sind richtig


B. 1 und 3 sind richtig
C. 2 und 4 sind richtig
D. nur 4 ist richtig
E. alle sind richtig

Richtige Antwort: E

Frage 16
Welcher Ventrikelabschnitt besitzt keine eigene Tela choroidea?

A. Cella media
B. Cornu inf.
C. Ventriculus tertius
D. Cornu ant.
E. Ventriculus quartus

Richtige Antwort: D

Frage 17
Der N. ischiadicus verlsst das Becken durch das:

A. Foramen ischiadium minus


B. Foramen suprapiriforme
C. Foramen obduratum
D. Foramen infrapiriforme
E. Foramen intervertebrale zwischen L5 und S1

Richtige Antwort: D

Frage 18
Im Chiasma opticum kreuzt/kreuzen:

A. Opticusnervenfasern aus der temporalen Retinahlfte


B. Opticusganglienzellen
C. Opticusnervenfasern aus der nasalen Retinahlfte
D. die Radiatio optica
E. Stbchen- und Zapfenzellen

Richtige Antwort: C

Frage 19
Welche Struktur ist mit der Ziffer 2 bezeichnet?
A. Tractus opticus
B. Stria olfactoria medialis
C. Tractus olfactorius
D. Nervus opticus
E. Gyrus rectus

Richtige Antwort: C

Frage 20
Welches Ganglion gehrt nicht zum vegetativen Nervensystem?

A. Ganglion ciliare
B. Ganglion submadibulare
C. Ganglion stellatum (cervicothoracicum)
D. Ganglion spinale
E. Ganglion pterygopalatinum

Richtige Antwort: D
Frage 21
Der visuelle Cortex liegt in der Rinde des:

A. Lobus temporalis
B. Lobus frontalis
C. Lobus parietalis
D. Lobus occipitalis
E. Lobus insularis

Richtige Antwort: D

Frage 22
Welche Aussage(n) trifft/treffen zu?

1. Die Auris externa besteht aus Auricula und Meatus acusticus externus.
2. Die Auris media besteht aus Cavitas tympanica, Tuba auditiva und Cellulae mastoideae.
3. Die Auris interna besteht aus Labyrinthus osseus und Labyrinthus membranaceus.
4. Auris media und Auris interna liegen zur Gnze im Schlfenbein

A. 1, 2 und 3 sind richtig


B. 1 und 3 sind richtig
C. 2 und 4 sind richtig
D. nur 4 ist richtig
E. alle sind richtig

Richtige Antwort: E

Frage 23
In welche Hauptstmme teilt sich der N. radialis?

A. In einen Ramus lateralis und medialis


B. In einen Ramus posterior und anterior
C. In einen Ramus superior und inferior
D. In einen Ramus superficialis und profundus
E. In einen Ramus muscularis und cutaneus

Richtige Antwort: D

Frage 24
Fr das Gehrorgan trifft/treffen zu:

1. Das Gehrorgan gliedert sich in Auen-, Mittel- und Innenohr


2. Das Gehrorgan liegt mit wesentlichen Teilen im Os temporale
3. Das Mittelohr steht mit dem Nasenrachenraum ber die Eustachische Rhre in Verbindung
4. Die Tuba auditiva dient der Belftung des Innenohrs

A. 1, 2 und 3 sind richtig


B. 1 und 3 sind richtig
C. 2 und 4 sind richtig
D. nur 4 ist richtig
E. alle sind richtig

Richtige Antwort: A

Frage 25
Welche Recessus befinden sich im III. Ventrikel?

1. Recessus infundibuli
2. Recessus triangularis
3. Recessus pinealis
4. Recessus piriformis

A. 1, 2 und 3 sind richtig


B. 1 und 3 sind richtig
C. 2 und 4 sind richtig
D. nur 4 ist richtig
E. alle sind richtig

Richtige Antwort: A

Frage 26
Der Fasciculus medialis des Plexus brachialis entlsst folgende Nerven:

A. Radix med. des N. medianus, N. ulnaris, N. radialis, N. cutaneus antebrachii med.


B. Radix med. des N. medianus, N. cutaneus brachii med., N cutaneus antebrachii med., N. axillaris
C. Radix med. des N. medianus, N ulnaris, N axillaris, N. cutaneus brachii med., N cutaneus antebrachii
med.
D. N. axillaris, N. radialis, N. ulnaris, Radix med. des N. medianus
E. Radix med. des N. medianus, N. ulnaris, N cutaneus brachii med., N cutaneus antebrachii med.

Richtige Antwort: E

Frage 27
Die Tractus spinocerebellaris anterior und posterior:

A. bilden die Fortsetzung der kurzen HWF


B. kreuzen die Seite
C. bilden die Fortsetzung der mittellangen HWF
D. sind fr die Schmerzleitung us der oberen Extremitt zustndig
E. sind wichtig fr die Willkrmotorik

Richtige Antwort: C

Frage 28
Welche der folgenden Aussagen ist/sind richtig?

1. Der Nucleus motorius nervi trigemini entsendet motorische Fasern fr die mimische
Muskulatur
2. Der Nucleus pontinus nervi trigemini ist der sensible/taktile Kern des nervus V. Hier erfolgt die
Umschaltung von Druck und Berhrung an Hals und Gesicht
3. Der Nucleus tractus mesencephali nervi trigemini empfngt Geschmacksfasern aus der
Zunge
4. Der Nucleus tractus spinalis nervi trigemini ist ein Kern fr Schmerz- und
Temperaturempfindung. er wirft das Tuberculum cinereum auf

A. 1, 2 und 3 sind richtig


B. 1 und 3 sind richtig
C. 2 und 4 sind richtig
D. nur 4 ist richtig
E. alle sind richtig

Richtige Antwort: C

Frage 29
Auf welche Struktur weist der Pfeil in der folgenden Abbildung?
A. Thalamus
B. Genu corporis callosi
C. Corpus nuclei caudati
D. Hippocampus
E. Caput nuclei caudati

Richtige Antwort: E

Frage 30
Welche Struktur wird in der folgenden Abbildung mit der Ziffer 1 bezeichnet?
A. Gyrus parahippocampalis
B. Corpus fornicis
C. Dach des Unterhorns
D. Tapetum
E. Trigonum collaterale

Richtige Antwort: E

Frage 31
Der R. superficialis n. ulnaris versorgt:

A. die ulnaren zwei Mm. lumbricales


B. die Hypothenarmuskeln und die ulnaren uwei Mm. lumbricales
C. die Haut des kleinen Fingers und der ulnaren Seite des Ringfingers
D. die Hypothenarmuskeln
E. die Haut des Handrckens

Richtige Antwort: C

Frage 32
Welche anatomische Struktur gehrt nicht zum Rckenmarksquerschnitt?

A. Funiculus ant.
B. Funiculus post.
C. Cornu post.
D. Canalis centralis
E. Cauda equina

Richtige Antwort: E

Frage 33
Welche der genannten synaptischen Schaltstellen sind Teil der aufsteigenden Hrbahn?

1. Ncl. cochlearis dorsalis


2. Ncl. cochlearis ventralis
3. Ncl. Lemnisci lateralis
4. Colliculus superior

A. 1, 2 und 3 sind richtig


B. 1 und 3 sind richtig
C. 2 und 4 sind richtig
D. nur 4 ist richtig
E. alle sind richtig

Richtige Antwort: A

Frage 34
Welche Aussage(n) zu folgender Abbildung ist/sind richtig?

1. Die Strukturen 6 und 8 gehren zum Kleinhirn


2. Die Strukturen 3 und 5 gehren zum Mesencephalon
3. Die Strukturen 1 und 7 gehren zum Ventrikelsystem
4. Struktur 4 gehrt zum Metencephalon
A. 1, 2 und 3 sind richtig
B. 1 und 3 sind richtig
C. 2 und 4 sind richtig
D. nur 4 ist richtig
E. alle sind richtig

Richtige Antwort: B

Frage 35
Der N. tibialis verluft bis:

A. zur Regio retromalleolaris lat


B. zum Dorsum pedis
C. zur Regio retromalleolaris med
D. zur Extensorenloge
E. zur Peroneusloge

Richtige Antwort: C

Frage 36
Welche Behauptung trifft nicht zu?
Der N. peroneus (fibularis) communis:

A. luft an der Innenseite der Sehne des M. biceps femoris in Richtung Fibul
B. versorgt die Flexoren am Unterschenkel
C. teilt sich in den N. peroneus (fibularis) superficialis und profundus
D. versirgt die Extensoren am Unterschenkel und die Mm. peronei
E. betritt die Peroneusloge unter dem Caput fibulae

Richtige Antwort: B

Frage 37
Der Plexus sacralis liegt:

A. im Canalis sacralis
B. lateral an der Innenseite des Os ileum
C. an der ventralen Seite des Os sacrum
D. lateral an der Innenseite der Sitzbeinhcker
E. an der dorsalen Seite des Os sacrum

Richtige Antwort: C

Frage 38
Welche Aussage zum Rckenmark ist nicht richtig?

A. Die Menge der weien Substanz nimmt von cranial nach caudal zu
B. Das Vorderhorn weist eine somatotopische Gliederung auf
C. Ein Spinalnerv leutet somatomotorische, visceromotorische, somtosensible und viscerosensible
Erregungen
D. In den Plexus mischen sich die Fasern verschiedener Spinalnerven
E. Der erste Spinalnerv tritt zwischen Os occipitale und Atlas aus

Richtige Antwort: A

Frage 39
Folgende Aussage ber den Augapfel trifft zu:

A. Der Augapfel hat eine zweischichtige Wand


B. Im Inneren des Augapfels befindet sich Trnenflssigkeit, Linse und Glaskrper
C. Die Tunica vasculosa bulbi wird auch Retina genannt
D. An der Macula lutea befindet sich die Stelle des schrfsten Sehens
E. Der Sehnerv verlsst den Augapfel am gelben Fleck

Richtige Antwort: D

Frage 40
Welche der folgenden Aussagen ist richtig?

A. Das Archicerebellum ist der Gleichgewichtsanteil des Kleinhirns


B. Das Neocerebellum empfngt Bahnen der Tiefensensibilitt aus dem Rckenmark
C. Das Spinocerebellum umfasst den Lobus post des Corpus cerebelli
D. Das Palocerebellum ist der lteste Anteil des Kleinhirns und wird auch Vestibulocerebellum genannt
E. Das Pontocerebellum besteht aus dem Lobulus flocculonodularis und der Lingula

Richtige Antwort: A

Frage 41
Cerebellre Lsionen knnen bewirken:

1. eine Ataxie
2. eine Dysmetrie
3. eine Asynergie
4. einen Intentionstremor

A. 1, 2 und 3 sind richtig


B. 1 und 3 sind richtig
C. 2 und 4 sind richtig
D. nur 4 ist richtig
E. alle sind richtig

Richtige Antwort: E

Frage 42
Auf welche Struktur weist in der folgenden Abbildung der Pfeil mit der Ziffer 3?

A. Pedunculus cerebellaris sup


B. Pedunculus cerebellaris med
C. Pedunculus cerebellaris inf
D. Crus cerebri
E. Pes pontis

Richtige Antwort: D

Frage 43
Welche Aussage zu folgender Abbildung ist nicht richtig?
A. 3 gehrt zum Seitenventrikel
B. 2 ist ein Bestandteil des Corpus striatum
C. 1 stellt das Putamen dar
D. 2 ist ein Bestandteil der Basalganglien
E. 3 gehrt zum Liquorsystem

Richtige Antwort: C

Frage 44
Welche Aussage zu folgender Abbildung ist richtig?

A. 3 gehrt zum Vorderhorn des Seitenventrikel


B. 2 stellt den Nucleus caudatus dar
C. 1 begrenzt den II. Ventrikel seitlich
D. 2 stellt das Claustrum dar
E. 1 bildet die laterale Wand des Vorderhorns des Seitenventrikels

Richtige Antwort: E

Frage 45
Welche Aussage zu folgender Abbildung trifft zu?
A. 4 zeugt den Nucleus cuneatus
B. 3 zeugt den Nucleus gracilis
C. 4 zeigt ein Corpus mammillare
D. 5 zeigt ein Corpus mammillare
E. 5 zeigt eine Pyramide

Richtige Antwort: D

Frage 46
Welche der folgenden Aussagen ist richtig?
In folgender Abbildung zeigt:
A. 4 die Brcke
B. 6 eine der beiden Hemisphren
C. 8 den Lobus occipitalis des Grohirns
D. 5 die Lamina quadrigemina
E. 7 den III. Ventrikel

Richtige Antwort: D

Frage 47
Welche der folgenden Aussagen ist richt?
In folgender Abbildung zeigt:
A. 4 die Olive
B. 3 die Pyramide
C. 4 ein Corpus mammillare
D. 5 ein Corpus mammillare
E. 1 die Pyramide

Richtige Antwort: D

Frage 48
Seine caudale Begrenzung findet der Hirnstamm auf Hhe der/des:

A. Decussatio nervorum trochlearium.


B. Decussatio lemnisci medialis
C. Corpus trapezoideum.
D. Decussatio pyramidum.
E. Decussatio pendunculorum cerebellarium superiorum

Richtige Antwort: D

Frage 49
Die Hinterstrangbahnen (Fasciculus gracilis und cuneatus):

1. gehen aus den Strangzellen des Rckenmarks hervor


2. zeigen eine segmentale Gliederung der Faserbndel
3. werden hauptschlich von mittellangen Hinterwurzelfasern gebildet
4. schalten erst in den Kernen der medulla oblongata um
A. 1, 2 und 3 sind richtig
B. 1 und 3 sind richtig
C. 2 und 4 sind richtig
D. nur 4 ist richtig
E. alle sind richtig

Richtige Antwort: C

Frage 50
Folgende Aussage(n) zu folgender Abbildung trifft/treffen zu:

1. Ziffer 2 zeigt die Nackenbeuge


2. Ziffer 1 zeigt ein auswachsendes Augenblschen
3. Ziffer 3 weist auf die Brckenbeuge hin
4. Ziffer 1 zeigt ein Hemisphrenblschen

A. 1, 2 und 3 sind richtig


B. 1 und 3 sind richtig
C. 2 und 4 sind richtig
D. nur 4 ist richtig
E. alle sind richtig

Richtige Antwort: D

Frage 51
Die Gehrknchelchen des Mittelohres:

1. bewirken den statischen Druckausgleich in der Paukenhhle


2. btertragen den Schall vom Trommelfell zum runden Fenster
3. sind wichtig fr das Zustandekommen der Knochenleitung
4. gewhrleisten die Impedanzanpassung zwischen Luft und Innenohrflssigkeit

A. 1, 2 und 3 sind richtig


B. 1 und 3 sind richtig
C. 2 und 4 sind richtig
D. nur 4 ist richtig
E. alle sind richtig

Richtige Antwort: D

Frage 52
Welche Aussage zum Rckenmark ist richtig?

A. Die Anordnung der weien Substanz hnelt einem aufgefalteten Schmetterling.


B. Der Durchmesser des Rckenmarks betrgt 5 cm.
C. Die Intumescentia cervicalis befindet sich im Cervikalmark in Hhe von C2 - C4.
D. Das Filum terminale ist eine der Wurzelfasern.
E. Das Cornu laterale gibt es nicht in allen Segmenten des Sakralmarks

Richtige Antwort: E

Frage 53
Der Liquor cerebrospinalis wird grtenteils gebildet von:

A. Arachnoidea
B. Processus ciliares
C. Plexus choroideus
D. Endoneurium
E. Astrozyten

Richtige Antwort: C

Frage 54
Am Mesencephalon kann man von dorsal folgende Strukturen erkennen:

1. Lamina quadrigemina
2. Austrittsstelle des Nervus trochlearis
3. Crura cerebri
4. Fossa interpeduncularis

A. 1, 2 und 3 sind richtig


B. 1 und 3 sind richtig
C. 2 und 4 sind richtig
D. nur 4 ist richtig
E. alle sind richtig

Richtige Antwort: A

Frage 55
Welche Aussage zu folgender Abbildung ist richtig?

A. 4 ist das Tuberculum gracile und wird vom Nucleus gracilis GOLL aufgeworfen
B. 4 wird durch den Nucleus olivaris inf verursacht
C. 4 ist das Tuberculum cuneatum und wird vom Nucleus cuneatus BURDACH aufgeworfen
D. 4 ist das Turberculum trigeminale
E. 4 wird von der Pyramidenbahn aufgeworfen

Richtige Antwort: E

Frage 56
Zu den zentralen Schaltstellen der Hrbahn zhlen:

1. Kochleariskerne
2. Corpus geniculatum laterale
3. Hrrinde im Lobus temporalis
4. Colliculi superiores der Lamina tecti

A. 1, 2 und 3 sind richtig


B. 1 und 3 sind richtig
C. 2 und 4 sind richtig
D. nur 4 ist richtig
E. alle sind richtig

Richtige Antwort: B

Frage 57
Folgendes trifft auf die Trunci des Plexus brachialis zu:

1. Der Truncus superior entsteht aus den Segmenten C5 und C6


2. Die Trunci liegen in der vorderen Scalenuslcke
3. Der Truncus inferior entsteht aus dem Segment C8 und Th1
4. Der Truncus medius entsteht aus den Segmenten C7 und C8

A. 1, 2 und 3 sind richtig


B. 1 und 3 sind richtig
C. 2 und 4 sind richtig
D. nur 4 ist richtig
E. alle sind richtig

Richtige Antwort: B

Frage 58
In welcher Reihenfolge werden die Ventrikelrume durchlaufen?

A. Aquaeductus cerebri, III. Ventrikel, IV. Ventrikel, Seitenventrikel


B. Seitenventrikel, III. Ventrikel, IV. Ventrikel, Aquaeductus cerebri
C. Seitenventrikel, III. Ventrikel, Aquaeductus cerebri, IV. Ventrikel
D. IV. Ventrikel, Aquaeductus cerebri, Seitenventrikel, III. Ventrikel
E. Seitenventrikel, Aquaeductus cerebri, III. Ventrikel, IV. Ventrikel

Richtige Antwort: C

Frage 59
Welche Aussage ist richtig?
In folgender Abbildung zeigt:
A. 4 den Nucleus cuneatus
B. 3 den Nucleus gracilis
C. 1 den oberen Kleinhirnstiel
D. 6 die Fossa interpeduncularis
E. 5 die Pyramide

Richtige Antwort: D

Frage 60
Abtrennung des intakten Rckenmarks von den hheren Zentren - welche der folgenden Symptome
knnen nach dem Akutstadium auftreten?

1. gesteigerte Eigenreflexe
2. unwillkrliche rythmische Bewegungen (Kloni)
3. ein positives Babinski-Zeichen
4. permanent schlaffe Lhmung

A. 1, 2 und 3 sind richtig


B. 1 und 3 sind richtig
C. 2 und 4 sind richtig
D. nur 4 ist richtig
E. alle sind richtig

Richtige Antwort: A
Prfung C

Frage 1
Was ist keine natrliche ffnung des Ventriculus quartus?

A. Canalis centralis
B. Apertura lateralis
C. Apertura mediana
D. Aquaeductus cerebri
E. Foramen Monroi

Richtige Antwort: E

Frage 2
Welche Aussage ber die folgende Abbildung trifft zu?

A. 1 zeigt den Thalamus


B. 2 enthlt den Tractus spinocerebellaris ant.
C. 3 zeigt Pyramidenzellen
D. 1 ist Teil des extrapyramidalmotorischen Systems
E. 1 zeigt den Nucleus caudatus

Richtige Antwort: D

Frage 3
Der primre somatosensible Cortex ist vom primren Motorcortex getrennt durch:

A. den Gyrus praecentralis


B. den Gyrus postcentralis
C. die Fissura longitudinalis cerebri
D. den Sulcus lateralis
E. den Sulcus centralis

Richtige Antwort: E
Frage 4
Welche Aussage zur folgenden Abbildung ist richtig?

A. Auf der Abbidung erkennt man die Intumescentia cervicalis


B. In Struktur 1 liegen die Ursprungszellen parasympathischer Nerven
C. Struktur 3 enthlt Substantia gelantinosa
D. Efferente Fasern aus Struktur 1 und Struktur 2 verlaufen im Ramus posterior des Spinalnerven
E. Eine Zerstrung der Struktur 3 fhrt zu motorischen Ausfllen

Richtige Antwort: E

Frage 5
Welche Aussage ist richtig?

A. Der Sulcus centralis trennt den Temporallappen vom Parietal- und Frontallappen
B. Der Sucus calcarinus trennt Temporal- und Occipitallappen
C. Der Sulcus lateralis liegt im Temporallappen
D. Der Sulcus cinguli liegt zwischen Gyrus cinguli und Corpus callosum
E. Der Sulcus lateralis trennt den Parietallappen und Frontallappen vom Temporallappen

Richtige Antwort: E

Frage 6
Der N. maxillaris verlt die Schdelhhle durch:

A. die Fissura orbitalis sup.


B. die Fissura orbitalis inf.
C. das Foramen ovale
D. das Foramen rotundum
E. das Foramen lacerum

Richtige Antwort: D

Frage 7
Bei Ausfllen der Substantia nigra kommt es zu:

A. Gleichgewichtsstrungen
B. spastische Lhmungen
C. Hypertonisierung der Muskulatur
D. Sensibilittsstrungen
E. Strungen der Tiefensensibilitt

Richtige Antwort: C

Frage 8
Welche Aussage zur folgenden Abbildung trifft zu?

A. Als Spina bifida bezeichnet man eine Verschlussstrung von Struktur 1


B. Die Abbildung zeigt das mesodermale Blatt der Keimscheibe
C. Ziffer 2 weist auf die ursegmentale Gliederung hin (Somiten)
D. Bleibt der Verschluss von Struktur 3 aus, entwickeln sich keine hheren Hirnabschnitte
E. Ziffer 2 zeigt die Kiemenbogennerven

Richtige Antwort: C

Frage 9
Welche Aussage ber die ueren Augenmuskeln trifft zu?

A. Der M. rectus lateralis adduziert den Bulbus oculi


B. Der M rectus medialis wird vom N. abducens innerviert
C. Der M rectus inferior senkt den Bulbus oculi
D. Die Hauptaufgabe der Mm. recti bulbi ist die Rotation des Bulbus oculi
E. Der 4. Hirnnerv innerviert die meisten ueren Augenmuskeln

Richtige Antwort: C

Frage 10
Welche Aussagen zur Blutversorgung des Gehrins ist richtig?

A. Die Region um den Sulcus lateralis wird von der A. cerebri anterior versorgt.
B. Die Region um das Corpus callosum wird rostral von der A. cerebri anterior versorgt.
C. Die Region um die Mantelkante wird medial von der A. cerebri media versorgt.
D. Die Region um den Sulcus lateralis von der A. cerebri posterior versorgt.
E. Die Region um den Sulcus calcarinus wird von der A. cerebri anterior versorgt.

Richtige Antwort: B

Frage 11
Der Augenbecher ist:

A. eine Ausstlpung der 2. Schlundtasche


B. eine Ausstlpung des Vorderhirns
C. eine Einbuchtung der 1. Schlundfurche
D. entodermaler Abstammung
E. Ausgangsmaterial der Augenlinse

Richtige Antwort: B

Frage 12
Welche der folgenden Strukturen gehrt/gehren zum Gleichgewichtsorgan?

1. Basilarmembran
2. Statholithenmembran
3. Membrana tectoria
4. Cupula

A. 1, 2 und 3 sind richtig


B. 1 und 3 sind richtig
C. 2 und 4 sind richtig
D. nur 4 ist richtig
E. alle sind richtig

Richtige Antwort: C

Frage 13
Welche der folgenden Aussagen ist richtig?

A. Der Nervus facialis enthlt motorische Fasern fr die Kaumuskulatur.


B. Der Nucleus salivatorius superior empfngt die Geschmacksfasern aus dem N. facialis.
C. Auer dem N. facialis senden noch der N. glossopharyngeus und der N. vagus Fasern an den Nucleus
tractus solitarii.
D. Der Nucleus salivatorius superior entsendet parasympathische Fasern fr die Glandula parotis.
E. Der Nucleus tractus solitarii entsendet parasympathische Fasern fr die Glandulae submandibularis und
sublingualis.

Richtige Antwort: C

Frage 14
Welche Kerne der Extrapyramidalmotorik befinden sich im Mesencephalon?

1. Substantia nigra
2. Nucleus subthalamicus
3. Nucleus ruber
4. Globus pallidus

A. 1, 2 und 3 sind richtig


B. 1 und 3 sind richtig
C. 2 und 4 sind richtig
D. nur 4 ist richtig
E. alle sind richtig

Richtige Antwort: B

Frage 15
Der N. medianus versorgt an der Hand:

1. die oberflchliche Thenarmuskulatur


2. die zwei radialen Mm. lumbricales
3. Haut der radialen drei-einhalb Finger
4. die Mm. interossei palmares I und II

A. 1, 2 und 3 sind richtig


B. 1 und 3 sind richtig
C. 2 und 4 sind richtig
D. nur 4 ist richtig
E. alle sind richtig

Richtige Antwort: A

Frage 16
Was erkennt man auf einem Rckenmarksquerschnitt?

A. Hinterseitenstrang
B. Cornu inferius
C. Sulcus calcarinus
D. Commissura fornicis
E. Sulcus posterolateralis

Richtige Antwort: E

Frage 17
Welche Zuordnung ist falsch?

A. Sehzentrum - Area striata


B. motorisches Zentrum - Gyrus postcentralis
C. Hrrinde - Heschl'sche Querwindung
D. Wenicke - sensorisches Sprachzentrum
E. Broca - motorisches Sprachzentrum

Richtige Antwort: B

Frage 18
Was zhlt nicht zur Begrenzung des III. Ventrikels?

A. Tegmentum mesencephali
B. Thalamus
C. Hypothalamus
D. Subthalamus
E. Telo choroidea

Richtige Antwort: D

Frage 19
Welche Struktur liegt der Cortex-Oberflche am nchsten?

A. die Dura mater


B. das Spatium subarachnoideum
C. die Sinus durae matris
D. die Pia mater
E. die Arachnoidea mater

Richtige Antwort: D

Frage 20
Welche Aussage ber die ueren Augenmuskeln trifft nicht zu?

A. Die ueren Augenmuskeln bestehen aus vier geraden und zwei schrgen Muskeln
B. Die ueren Augenmuskeln werden vom III.-, IV.- und VI.-Hirnnerven versorgt
C. Die ueren Augenmuskeln sind quergestreifte Muskeln
D. Der Bulbus oculi wird von sechs ueren Augenmuskeln bewegt
E. Die ueren Augenmuskeln entspringen alle vom Anulus tendineus communis

Richtige Antwort: E

Frage 21
Welche der folgenden Aussagen ist richtig?

A. Von der Brcke geht nach lateral der Pedunculus cerebellaris superior ab
B. Die seitliche Begrenzung der Brcke bildet die Verbindungslinie zwischen dem Austritt des N.
occulomotorius und dem Austritt des N. vestibulocochlearus
C. Die seitliche Begrenzung der Brcke bildet die Verbindungslinie zwischen dem Austritt des N.
trigeminus und dem Austritt des N. facialis
D. Von der Brcke geht nach seitlich der Pedunculus cerebellaris inferior ab
E. Die seitliche Begrenzung der Brcke bildet die Verbindungslinie zwischen dem Austritt des N.
occulomotorius und dem Austritt des N. abducens

Richtige Antwort: C

Frage 22
Welche Aussage(n) zu folgender Abbildung ist/sind richtig?

1. Struktur 2 enthlt viszerosensible Afferenzen


2. Struktur 1 trgt die Neurone viszero-efferenter Fasern
3. Struktur 2 enthlt somatosensible Afferenzen
4. Struktur 1 stellt das Hinterhorn dar
A. 1, 2 und 3 sind richtig
B. 1 und 3 sind richtig
C. 2 und 4 sind richtig
D. nur 4 ist richtig
E. alle sind richtig

Richtige Antwort: D

Frage 23
Welche Angabe zu folgender Abbildung ist richtig:

A. 1 zeigt eine Extremittenknospe


B. 2 zeigt das sptere Diencephalon
C. 3 zeigt ein Foramen interventriculare primitivum
D. 2 weist auf den Conus medullaris hin
E. 3 zeigt die ffnung eines Augenblschens

Richtige Antwort: C

Frage 24
Welcher Hirnnerv ist in der folgenden Abbildung mit dem schwarzen Blttchen unterlegt, auf das der Pfeil
weist?

A. Der II. Hirnnerv


B. Der X. Hirnnerv
C. Der V. Hirnnerv
D. Der VI. Hirnnerv
E. Der VIII. Hirnnerv

Richtige Antwort: D

Frage 25
Folgende Aussage zu folgender Abbildung trifft zu:
A. 1 leitet Afferenzen zum Cerebellum
B. 1 zieht ungekreuzt
C. bei Ausfall von Struktur 2 entsteht eine schlaffe Lhmung
D. 1 gehrt zum ersten Neuron der Pyramidenbahn
E. 2 gehrt zum zweiten Neuron der Pyramidenbahn

Richtige Antwort: D

Frage 26
Bei den mit X markierten Strukturen in der Abbildung handelt es sich um:

A. Haare
B. Vater-Pacini-Lamellenkrperchen
C. Nerven
D. Ganglien
E. Talgdrsen

Richtige Antwort: C

Frage 27
Welche bertrgersubstanz und welchen Rezeptortyp findet man an der markierten Stelle (Pfeil B) des
Nervus Sympathicus?
Das Zielorgan sei eine Muskelzelle der glatten Gefmuskulatur!

1. Die bertrgersubstanz an der markierten Stelle B ist Acetylcholin


2. Die bertrgersubstanz an der markierten Stelle B ist Noradrenalin
3. Die bertrgersubstanz an der markierten Stelle B ist Glycin
4. Die Rezeptoren an der markierten Stelle B sind muskarinerge Rezeptoren
5. Die Rezeptoren an der markierten Stelle B sind nicotinerge Rezeptoren
6. Die Rezeptoren an der markierten Stelle B sind adrenerge alpha-Rezeptoren
7. Die Rezeptoren an der markierten Stelle B sind adrenerge beta-1-Rezeptoren
8. Die Rezeptoren an der markierten Stelle B sind glycinerge Renshaw-Rezeptoren

A. 2 und 7 sind richtig


B. 2 und 6 sind richtig
C. 3 und 8 sind richtig
D. 1 und 5 sind richtig
E. 1 und 4 sind richtig

Richtige Antwort: B

Frage 28
Welche Aussage ber Duftdrsen trifft nicht zu?

A. Sie sind exokrine Drsen


B. Die Sekretabgabe erfolgt holokrin
C. Sie entstehen aus dem Ektoderm
D. Sie zhlen zu den Hautanhangsgebilden
E. Sie kommen in Verbindung mit Haaren vor

Richtige Antwort: B

Frage 29
Bei der mit 5 bezeichneten Struktur in der Abbildung handelt es sich um das/die:
A. Pigmentepithel
B. Ganglienzellschicht
C. Schicht der Stbchen und Zapfen
D. innere Krnerschicht
E. uere Krnerschicht

Richtige Antwort: D

Frage 30
Ein 73-jhriger Patient entwickelt seit einigen Wochen groe Blasen, die rasch platzen und dann 'offene
Stellen' hinterlassen. Sie vermuten eine Autoimmunerkrankung im Sinne eines Pemphigus vulgaris.

Welche Struktur ist dafr in erster Linie pathogenetisch relevant?

A. Korneozyt (Hornzelle)
B. Stratum germinativum
C. Desmosomen
D. Langerhanszelle
E. Melanozyt

Richtige Antwort: C

Frage 31
Bei den in der Abbildung mit 2 markierten Strukturen handelt es sich um Anschnitte von/eines:

A. Haaren
B. Perikaryen von Nervenzellen
C. Schweidrsenausfhrungsganges
D. Arteriolen
E. markhaltige Nervenfasern

Richtige Antwort: C

Frage 32
Welche Zellen liegen nicht in der Epidermis?
A. Keratinozyten
B. Merkel-Zellen
C. Melanozyten
D. Becherzellen
E. Langerhans-Zellen

Richtige Antwort: D

Frage 33
Welche Zuordnung Ziffer-Struktur ist zutreffend?

A. 1 - Haarpapille
B. 2 - innere epitheliale Wurzelscheide
C. 3 - uere epitheliale Wurzelscheide
D. 4 - M. arrector pili
E. 5 - Schweidrse

Richtige Antwort: C

Frage 34
Ein Bndel von Nervenfasern im PNS wird umgeben von:

A. Astrozytenfortstzen
B. Myelin
C. Neurofibrillen
D. Perineurinum
E. Schwann - Zellen
Richtige Antwort: D

Frage 35
Ependymzellen bilden im ZNS:

A. die innerste Schicht der Blutkapillaren


B. die Auskleidung der Liquorrume
C. das Endoneurium
D. Markscheiden
E. Narben

Richtige Antwort: B

Frage 36
In der Abbildung sind verschiedene Strukturen mit Ziffern markiert.
Welche Zuordnung Ziffer - Struktur ist zutreffend?

A. 1 - Choroidea
B. 2 - Iris
C. 3 - Corpus ciliare
D. 4 - Cornea
E. 5 - Glaskrper

Richtige Antwort: D
Frage 37
In der mit 3 bezeichneten Schicht der abgebildeten Retina finden sich typischerweise:

A. Perikarya von bipolaren Nervenzellen


B. Perikarya von pseudounipolaren Nervenzellen
C. Zellkerne von Mller-Zellen
D. Perikarya von Sehzellen
E. Pigmentepithelzellen

Richtige Antwort: D

Frage 38
Welche Aussage ber die in der Abbildung mit 1 bezeichnete Struktur ist zutreffend?

A. Sie grenzt an den Glaskrper


B. Sie enthlt typischerweise keine Blutgefe
C. Sie besteht aus Nervenfasern
D. Sie produziert das Kammerwasser
E. Sie besteht aus kollagenfaserigen Bindegewebe

Richtige Antwort: E

Frage 39
Welche der folgenden Strukturen/Zellen kommen in der grauen Substanz des Rckenmarks nicht vor?

A. Perikaryen von Nervenzellen


B. Nervenfasern
C. Astrozyten
D. Mikrogliazellen (HORTEGA-Zellen)
E. Mantelzellen (Amphizyten)

Richtige Antwort: E

Frage 40
Welche der folgenden Strukturen ist in einer Leistenhaut nicht anzutreffen?
A. Talgdrse
B. Kapillare
C. Scheidrse
D. Stratum papillare
E. Meissner-Tastkrperchen

Richtige Antwort: A

Frage 41
Beim Perineurium handelt es sich um das/die:

A. Bindegewebshlle an der Oberflche eines Nervenfaserbndels


B. Myelinscheide markhaltiger Nervenfasern
C. Auskleidung der Liquorrume des ZNS
D. Bindegewebe zwischen einzelnen Nervenfasern
E. Gliazellhlle an der Oberflche eines Perikaryons

Richtige Antwort: A

Frage 42
Bei welchen der aufgelisteten Zellen handelt es sich um Nervenzellen?

1. Korbzellen
2. Purkinje-Zellen
3. Hortega-Zellen
4. Sternzellen
5. Mller-Zellen
6. Pyramidenzellen

A. nur 2, 5 und 6 sind richtig


B. nur 1, 2, 4 und 6 sind richtig
C. nur 1, 2, 3 und 4 sind richtig
D. alle sind richtig
E. nur 6 ist richtig

Richtige Antwort: B

Frage 43
In der Abbildung sind verschiedene Strukturen mit Pfeilen und Ziffern markiert.
Welche Zuordnungen Ziffer-Struktur sind zutreffend?

1. 1 - M. arrector pili
2. 2 - innere epitheliale Haarwurzelscheide
3. 3 - uere epitheliale Haarwurzelscheide
4. 4 - Haartrichter
5. 5 - Talgdrse
A. 3, 4 und 5 sind richtig
B. 1, 2 und 3 sind richtig
C. 1, 3 und 5 sind richtig
D. 1, 4 und 5 sind richtig
E. alle sind richtig

Richtige Antwort: C

Frage 44
Ein mehrschichtiges unverhorntes Plattenepithel findet sich:

1. in der Conjunctiva bulbi


2. an der Vorderflche der Cornea
3. an der Hinterflche der Iris
4. in der Pars caeca retinae
5. in der Pars optica retinae
6. an den Zonula-Fasern

A. nur 1 und 2 sind richtig


B. nur 2 und 5 sind richtig
C. nur 3 ist richtig
D. nur 3, 5 und 6 sind richtig
E. nur 2, 5 und 6 sind richtig

Richtige Antwort: A

Frage 45
Welche der folgenden Aussagen ber Duftdrsen ist/sind zutreffend?
1. Sie kommen regelmig in der Perianalregion vor.
2. Die Endstcke sind tubuls.
3. Die Endstckepithelzellen besitzen apikal eine Crusta.
4. Sie nehmen ihre Aktivitt mit Beginn der Geschlechtsreife auf.
5. Ihre Ausfhrungsgnge mnden in Haartrichter.

A. nur 3 ist richtig


B. 1, 2 und 3 sind richtig
C. 1, 2, 3 und 4 sind richtig
D. 1, 2, 4 und 5 sind richtig
E. alle sind richtig

Richtige Antwort: D

Frage 46
Welche der folgenden Strukturen/Zellen kommen in der weien Substanz des Rckenmarks nicht vor?

A. Endothelzellen
B. marklose Nervenfasern
C. Schwann-Zellen
D. Oligodendrozyten
E. markhaltige Nervenfasern

Richtige Antwort: C

Frage 47
Welche der folgenden Strukturen ist/sind in einer Leistenhaut anzutreffen?

1. Stratum papillare
2. Stratum granulosum
3. Stratum reticulare
4. Stratum lucidum
5. Meissner-Tastkrperchen

A. nur 4 ist richtig


B. nur 5 ist richtig
C. alle sind richtig
D. nur 2 und 4 sind richtig
E. nur 1 und 3 sind richtig

Richtige Antwort: C

Frage 48
Welche der folgenden Aussagen ber die Lamina limitans gliae superficialis ist/sind zutreffend?

1. Sie ist Teil der Leptopmeninx.


2. Sie findet sich an der Oberflche von vegetativen Ganglien.
3. Sie wird von Astrozyten gebildet.
4. Sie bedeckt Ranvier-Schnrringe an markhaltigen Nervenfasern.
5. Sie besteht aus dicht gelagerten Ependymzellen.

A. nur 1 und 3 sind richtig


B. nur 2, 4 und 5 sind richtig
C. nur 3 ist richtig
D. alle sind richtig
E. nur 1, 2, 4 und 5 sind richtig

Richtige Antwort: C
Frage 49
Eine 43-jhrige Patientin zeigt stark juckende Hautvernderungen an beiden Handrcken. Die Anamnese
ergibt, dass sie vor einigen Tagen eine neue Handcreme gekauft und angewandt hat.

Welche Struktur ist fr die Hautkrankheit in erster Linie pathogenetisch relevant?

A. Korneozyt
B. Stratum germinativum
C. Desmosomen
D. Langerhanszelle
E. Melanozyt

Richtige Antwort: D

Frage 50
Welche Zellen bzw. Krperchen liegen nicht in der Epidermis?

A. Merkel-Zellen
B. Vater-Pacini-Lamellenkrperchen
C. Langerhans-Zellen
D. Melanozyten
E. Keratinozyten

Richtige Antwort: B

Frage 51
Welche der angefhrten Drsen tritt/treten in Verbindung mit Haaren auf?

1. Milchdrse
2. Duftdrse
3. Schweidrse
4. Talgdrse

A. 1, 2 und 3 sind richtig


B. 1 und 3 sind richtig
C. 2 und 4 sind richtig
D. nur 4 ist richtig
E. alle sind richtig

Richtige Antwort: C

Frage 52
In der Abbildung sind verschiedene Strukturen mit Pfeilen und Ziffern markiert.
Welche Zuordnung Ziffer-Struktur ist nicht zutreffend?
A. 3 - uere epitheliale Wurzelscheide
B. 1 - M. arrector pili
C. 5 - Duftdrse
D. 4 - Haarpapille
E. 2 - Haarwurzel

Richtige Antwort: C

Frage 53
Welche der folgenden Strukturen findet/finden sich nicht im Perikaryon einer Nervenzelle?

A. Myelinscheide
B. Nissl-Schollen
C. Golgi-Apparat
D. Mikrotubuli
E. Lysosomen

Richtige Antwort: A

Frage 54
Die Abbildung zeigt einen Ausschnitt aus dem vorderen Augenbulbus.
Welche Zuordnung Buchstabe-Struktur ist richtig?
A. A - Cornea
B. B - Schlemm-Kanal
C. C - Sklera
D. D - M. dilatator pupillae
E. E - M. sphincter pupillae

Richtige Antwort: B

Frage 55
Welche der folgenden Aussagen ber die Cornea trifft/treffen zu?

1. Sie produziert das Kammerwasser.


2. Sie grenzt an die hintere Augenkammer.
3. Sie ist typischerweise geflos.
4. Sie zhlt zur ueren Augenhaut.
5. Sie besitzt an ihrer Vorderflche eine einschichtiges Epithel.

A. 2, 3 und 4 sind richtig


B. 3, 4 und 5 sind richtig
C. 3 und 4 sind richtig
D. nur 4 ist richtig
E. 1, 3 und 4 sind richtig

Richtige Antwort: C

Frage 56
Welche Aussage ber Duftdrsen trifft nicht zu?

A. Duftdrsen kommen beim Menschen nur in bestimmten Hautarealen vor.


B. Duftdrsen sind tubulse Drsen.
C. Die Sekretion der Duftdrsen setzt erst mit der Pubertt ein.
D. Die sezernierenden Endstckepithelzellen zeigen typischerweise eine unterschiediche Zellhhe.
E. Duftdrsen sind besonders zahlreich in der Haut der Fusohle anzutreffen.

Richtige Antwort: E

Frage 57
Aus der ueren Schicht des Augenbechers entsteht/entstehen:

1. Sinneszellen
2. Sttzzellen
3. bipolare Nervenzellen
4. Pigmentepithel

A. 1, 2 und 3 sind richtig


B. 1 und 3 sind richtig
C. 2 und 4 sind richtig
D. nur 4 ist richtig
E. alle sind richtig

Richtige Antwort: D

Frage 58
Kortex - als Module (Kolumnen) bezeichnet man:

1. Rindenfelder hnlicher histologischer Struktur


2. sulenartige Anordnung von hunderten von Neuronen
3. die Schichtengliederung der Hirnrinde
4. Gruppen von Neuronen die eine funktionelle Einheit bilden

A. 1, 2 und 3 sind richtig


B. 1 und 3 sind richtig
C. 2 und 4 sind richtig
D. nur 4 ist richtig
E. alle sind richtig

Richtige Antwort: C

Frage 59
Die Zellkerne in der mit 3 bezeichneten Struktur in der Abbildung gehren zu:

A. pseudounipolare Nervenzellen
B. Photorezeptorenzellen
C. Pigmentepithelzellen
D. bipolaren Nervenzellen
E. Mller-Zellen
Richtige Antwort: B

Frage 60
Eine Nervenzelle wird als multipolar bezeichnet, wenn sie:

A. viele verschiedene Neurotransmitter synthetisiert


B. von vielen Gliazellen umgeben ist
C. viele Synapsen an ihrer Oberflche hat
D. viele Dendrite und ein Axon besitzt
E. viele Zellkerne besitzt

Richtige Antwort: D
Prfung D

Frage 1
Das Stratum germinativum der Epidermis:

A. besteht aus Hornschuppen


B. dient der Regeneration
C. findet sich nur in der Leistenhaut
D. enthlt Keratohyalin
E. enthlt Kapilarschlingen

Richtige Antwort: B

Frage 2
Das Endoneurium wird gebildet von:

A. Nervenzellen
B. Ependymzellen
C. Mantelzellen
D. Bindegewebszellen
E. Schwann-Zellen

Richtige Antwort: D

Frage 3
Welche Aussage zu den in der Abbildung dargestellten Strukturen ist zutreffend?
A. 5 zhlt zur inneren Augenhaut
B. 5 besteht aus kollagenen Fasern
C. 2 enthlt einen quergestreiften Muskel
D. 4 grenzt mit ihrer Hinterflche an der vorderen Augenkammer
E. 3 besitzt an ihrer Vorderflche ein mehrschichtiges Epithel

Richtige Antwort: D

Frage 4
Bei der in der Abbildung mit einem Kreis markierten Struktur handelt es sich um ein/eine/einen:

A. Nerv
B. Vater-Pacini-Lamellenkrperchen
C. Haar
D. Meissner-Tastkrperchen
E. Talgdrse

Richtige Antwort: B

Frage 5
Bei der mit 3 bezeichneten Struktur in der Abbildung handelt es sich um das/die:
A. Pigmentepithel
B. Schicht der Stbchen und Zapfen
C. innere Krnerschicht
D. uere Krnerschicht
E. Ganglienzellschicht

Richtige Antwort: D

Frage 6
Welche Aussage ber Astrozyten trifft zu?

A. Sie bilden die Membrana limitans gliae pervascularis


B. Sie bilden die Myelinscheide von Nervenfasern im ZNS
C. Sie zhlen zum Mononukleren Pagozytensystem
D. Sie kleiden die Liquorrume des ZNS aus
E. Sie bilden die Arachnoidea

Richtige Antwort: A

Frage 7
Ependymzellen:

1. sind Gliazellen
2. bilden die Gefwand von Blutkapillaren im ZNS
3. kleiden die Hirnventrikel aus
4. besitzen an der Oberflche Stereozilien

A. 1, 2 und 3 sind richtig


B. 1 und 3 sind richtig
C. 2 und 4 sind richtig
D. nur 4 ist richtig
E. alle sind richtig

Richtige Antwort: B
Frage 8
Die Cornea:

1. ist an ihrer Vorderflche von einem mehrschichtigem Epithel bedeckt


2. enthlt normalerweie keine Blutgefe
3. zhlt zur ueren Augenhaut
4. grenzt mit ihrer Hinterflche an die vordere Augenkammer

A. 1, 2 und 3 sind richtig


B. 1 und 3 sind richtig
C. 2 und 4 sind richtig
D. nur 4 ist richtig
E. alle sind richtig

Richtige Antwort: E

Frage 9
Bei der in der Abbildung mit 1 markierten Struktur handelt es sich um:

A. Arachnoidea
B. Stratum granulosum
C. Stratum moleculare
D. Substantia alba
E. Lamina pyramidalis

Richtige Antwort: C

Frage 10
Welche Zellart ist in Spinalganglien des Erwachsenen in der Regel nicht anzutrffen?

A. bipolare Nervenzelle
B. Endothelzelle
C. Mantelzelle
D. pseudounipolare Nervenzelle
E. Schwann-Zelle

Richtige Antwort: A
Frage 11
Welche der folgenden Strukturen gehrt/gehren nicht zu einer markhaltigen Nervenfaser des PNS?

A. Neurit
B. Perineurium
C. Internodien
D. Ranvier-Schnrringe
E. Schmidt-Lantermann-Einkerbungen

Richtige Antwort: B

Frage 12
In der Abbildung sind verschiedene Strukturen mit Pfeilen und Ziffern markiert.
Welche Zuordnung Ziffer-Struktur ist zutreffend?

A. 1 - M. arrector pili
B. 3 - Haarschaft
C. 5 - Schweidrse
D. 2 - Haarpapille
E. 4 - Meissner-Tastkrperchen

Richtige Antwort: A

Frage 13
Bei einer 36-jhrigen Patientin findet man am Unterschenkel einen 3 cm groen, unregelmig begrenzten
und dunkelbraun und schwarz pigmentierten Fleck, der in den letzten Monaten langsam an Gre
zugenommen hat.

Welche Struktur ist dafr in erster Linie pathogenetisch relevant?


A. Korneozyt
B. Stratum germinativum
C. Desmosomen
D. Langerhanszelle
E. Melanozyt

Richtige Antwort: E

Frage 14
Wo kommen Purkinje-Zellen vor?

A. Spinalganglien
B. Kleinhirnrinde
C. Lamina pyramidalis interna der Grohirnrinde
D. Vorderhorn des Rckenmarks
E. Nucleus thoracicus des Brustmarks

Richtige Antwort: B

Frage 15
Pyramidenzellen:

1. sind Neurone des Cerebellums


2. werden durch Krnerzellen erregt
3. haben GABA als Transmittersubstanz
4. werden durch Interneurone gehemmt

A. 1 und 3 sind richtig


B. 2 und 4 sind richtig
C. nur 4 ist richtig
D. alle sind richtig
E. nichts ist richtig

Richtige Antwort: C

Frage 16
Die Membrana limitans gliae superficialis wird gebildet von:

A. Arachnoidea
B. Astrozyten
C. Endothelzellen
D. Pia mater
E. Plexus choroideus

Richtige Antwort: B

Frage 17
Die Membrana limitans gliae perivascularis wird gebildet von:

A. Ependymzellen
B. Endothelzellen
C. Plexus choroideus
D. Perizyten
E. Astrozyten

Richtige Antwort: E
Frage 18
Fibrae zonluares (Zonulafasern) verbinden:

A. Cornea und Sclera


B. Augenlinse und Glaskrper
C. Corpus ciliare und Iris
D. M. ciliaris und Schlemm-Kanal
E. Corpus ciliare und Augenlinse

Richtige Antwort: E

Frage 19
Das Kammerwasser:

1. wird vom Epithel des Ziliarkrpers gebildet


2. fliet durch die Pupille in die vordere Augenkammer
3. fliet von der vorderen Augenkammer, durch den Schlemm Kanal, in die Venen ab
4. fliet bei Glaukom von der vorderen Augenkammer, durch den Schlemm-Kanal, in den Nasengang ab

A. 1, 2 und 3 sind richtig


B. 1 und 3 sind richtig
C. 2 und 4 sind richtig
D. nur 4 ist richtig
E. alle sind richtig

Richtige Antwort: A

Frage 20
Fr den Eigenreflex gilt:

1. er ist ein monosynatischer Reflex


2. er ist nicht adaptierbar
3. er besitzt eine kurze Reflexzeit
4. bei Auslsung eines Eigenreflexes in einem Muskel, wird der Antagonist gehemmt

A. 1, 2 und 3 sind richtig


B. 1 und 3 sind richtig
C. 2 und 4 sind richtig
D. nur 4 ist richtig
E. alle sind richtig

Richtige Antwort: E

Frage 21
Die typische Form des Aktionspotenzials der Nervenfaser kommt zustande, weil:

A. sich, durch die elektrotonische Depolarisation bedingt, zuert spannungsabhngige Natriumkanle und
spter spannungsabhngige Kaliumkanle ffnen
B. sich, durch die elektrotonische Depolarisation bedingt, zuert spannungsabhngige Kaliumkanle und
spter spannungsabhngige Natriumkanle ffnen
C. Die Weiterleitung des AP entlang der gesamten Nervenfaser rein durch passive elektronische Prozesse
bewerkstelligt wird
D. das AP stehts durch unterschwellige Reize hervorgerufen wird
E. sich in der Plasmamembran der Nervenfaser keine Ionenkanle befinden

Richtige Antwort: A

Frage 22
Fr das olfaktorische System gilt:

1. die Reiz-Empfindung-Relation ist annhernd logarithmisch.


2. die Empfindungsschwelle fr alle Substanzen ist konstant.
3. die Sinneszellen durchlaufen einen Lebenszyklus von ca. 60 Tagen.
4. das olfaktorische System zeigt nur minimale Adaption

A. 1, 2 und 3 sind richtig


B. 1 und 3 sind richtig
C. 2 und 4 sind richtig
D. nur 4 ist richtig
E. alle sind richtig

Richtige Antwort: B

Frage 23
Spinalmotorik: Was versteht man unter alpha-gamma Koaktivierung?

1. die Erhhung der Muskelsteifigkeit durch Kontraktion.


2. die gemischte efferente/afferente Innervation der Muskelrezeptoren
3. die reziproke Hemmung des Antagonisten beim Dehnungsreflex
4. die gleichzeitige Aktivierung von extra- und intrafusalen Muskelfasern

A. 1, 2 und 3 sind richtig


B. 1 und 3 sind richtig
C. 2 und 4 sind richtig
D. nur 4 ist richtig
E. alle sind richtig

Richtige Antwort: D

Frage 24
Die Akkommodationsbreite ist prinzipiell immer auch verringert bei:

A. Weitsichtigkeit (Hyperopie)
B. Alterssichtigkeit (Presbyopie)
C. Astigmatismus
D. Kurzsichtigkeit (Myopie)
E. Reduzierter Visus

Richtige Antwort: B

Frage 25
Stimmgabelversuche - welche Aussagen sind richtig?

1. Lateralisierung nach links (Weber) zeigt eine linksseitige Schallempfindungsstrung an


2. keine Lateralisierung (diffuse Wahrnehmung) zeigt an, dass keine Hrstrungen vorliegen knnen
3. 'Rinne positiv' an der rechten Seite zeigt eine rechtsseitige Schallleitungsstrung an
4. Lateralisierung nach rechts (Weber) und 'Rinne positiv' an der rechten Seite zeigt eine linke
Schallempfindungsstrung an.

A. 1, 2 und 3 sind richtig


B. 1 und 3 sind richtig
C. 2 und 4 sind richtig
D. nur 4 ist richtig
E. alle sind richtig

Richtige Antwort: D
Frage 26
Wie entstehen otoakustische Emissionen?

1. durch den Totalausfall des kochleren Verstrkermechanismus


2. durch Bewegungen der inneren Haarzellen
3. durch Kontraktion der Mittelohrmuskeln
4. durch Stimulation im ueren Gehrgang (gesundes Innenohr)

A. 1, 2 und 3 sind richtig


B. 1 und 3 sind richtig
C. 2 und 4 sind richtig
D. nur 4 ist richtig
E. alle sind richtig

Richtige Antwort: D

Frage 27
Fr kognitive Aufgaben bentigt man:

1. Corticale Areale
2. Basalganglien
3. Cerebellum
4. Medulla oblongata

A. 1, 2 und 3 sind richtig


B. 1 und 3 sind richtig
C. 2 und 4 sind richtig
D. nur 4 ist richtig
E. alle sind richtig

Richtige Antwort: A

Frage 28
Folgende Aussage(n) ber die Sehbahn trifft/treffen zu:

1. Die Optikusnervenfasern aus der nasalen Retinahlfte kreuzen im Chiasma opticum.


2. Der Tractus opticus verluft bis zum Corpus geniculatum mediale.
3. Vom Tractus opticus zweigen Fasern zur Area praetectalis und Lamina tecti ab.
4. Vom Corpus geniculatum laterale setzt sich die Sehbahn als Tractus opticus fort.

A. 1, 2 und 3 sind richtig


B. 1 und 3 sind richtig
C. 2 und 4 sind richtig
D. nur 4 ist richtig
E. alle sind richtig

Richtige Antwort: B

Frage 29
Wenn die Brechkraft einer Linse durch Hinzufgen einer zweiten Linse von 1 Dioptrie verdoppelt wird auf 2
Dioptrien, dann hat sich die Brennweite der Gesamtlinse:

A. verdoppelt
B. halbiert
C. vervierfacht
D. auf ein Viertel reduziert
E. nicht gendert

Richtige Antwort: B
Frage 30
Miosis (Pupillenverengung) findet man bei:

1. Starker Beleuchtung
2. Blick auf nahe Gegenstnde
3. Lhmung des Hals-Sympathicus
4. Erregung des Sympathicus

A. 1, 2 und 3 sind richtig


B. 1 und 3 sind richtig
C. 2 und 4 sind richtig
D. nur 4 ist richtig
E. alle sind richtig

Richtige Antwort: A

Frage 31
Fr die Neurone des noradrenergen aufsteigenden Systems gilt:

1. die Neurone projezieren unspezifisch in kortikale Areale


2. die Neurone sind whrend der Wachphase aktiver als in der Schlafphase
3. die Neurone erhalten Kolaterale aus vielen Sinneskanlen
4. ihr Ursprungsgebiet ist der Nucleus coelueus

A. 1, 2 und 3 sind richtig


B. 1 und 3 sind richtig
C. 2 und 4 sind richtig
D. nur 4 ist richtig
E. alle sind richtig

Richtige Antwort: E

Frage 32
Fr Synapsen im ZNS gilt:

1. die Aktivierung der nachgeschalteten Nervenzelle erfolgt ber zeitliche und/oder rumliche Summation
von EPSPs bzw. IPSPs
2. ein singulres EPSP bewirkt eine unterschwellige lokale Depolarisation
3. IPSPs bewirken eine lokale Hyperpolarisation
4. EPSP und IPSP werden durch dieselben Transmittersubstanz vermittelt

A. 1, 2 und 3 sind richtig


B. 1 und 3 sind richtig
C. 2 und 4 sind richtig
D. nur 4 sind richtig
E. alle sind richtig

Richtige Antwort: A

Frage 33
ber den Schlemm-Kanal wird abgeleitet:

A. Kammerwasser
B. Trnenflssigkeit
C. Liquor cerebrospinalis
D. Lymphe
E. Blut
Richtige Antwort: A

Frage 34
In welcher Funktion/Wo kommt die Rckwrtshemmung in der spinalen Motorik vor:

1. ausgangsseitig zur Hemmung des nebenliegenden Motoneurone


2. eingangsseitig zur Drosselung des eigenen Kanals
3. im Reflexbogen/Reflexweg zur Hemmung des Antagonisten
4. ausgangsseitig in Form der Renshaw-Hemmung

A. 1, 2 und 3 sind richtig


B. 1 und 3 sind richtig
C. 2 und 4 sind richtig
D. nur 4 ist richtig
E. alle sind richtig

Richtige Antwort: C

Frage 35
Fr Opticus-Neuronen gilt:

1. Das On-Zentrum Neuron reagiert bei Belichtung in der Peripherie seines rezeptiven Feldes mit
Hemmung
2. Das On-Zentrum Neuron reagiert bei Belichtung in der Peripherie seines rezeptiven Feldes mit
Aktivierung
3. Das Off-Zentrum Neuron reagiert bei Belichtung im Zentrum des rezeptiven Feldes mit Hemmung
4. Das Off-Zentrum Neuron reagiert bei Belichtung im Zentrum des seines rezeptiven Feldes mit
Aktivierung

A. 1, 2 und 3 sind richtig


B. 1 und 3 sind richtig
C. 2 und 4 sind richtig
D. nur 4 ist richtig
E. alle sind richtig

Richtige Antwort: B

Frage 36
Fr das Phnomen der Adaption gilt:

1. der Adaptionsprozess besteht aus 2 Phasen


2. der Zeitverlauf des Adaptionsprozesses ist von der Reizintensitt unabhngig
3. durch Summierung von Adaptionsprozessen ist eine Zielgerichtete Entwicklung mglich (z.B. Training)
4. Adaption ist nur in nicht rckgekoppelten Systemen mglich

A. 1, 2 und 3 sind richtig


B. 1 und 3 sind richtig
C. 2 und 4 sind richtig
D. nur 4 ist richtig
E. alle sind richtig

Richtige Antwort: B

Frage 37
Auf der spinalen Ebene des motorischen Systems gilt fr die Alpha-Gamma-Koaktivierung:

1. Alpha- und Gamma-Motoneurone sind gleichzeitig erregt


2. die "Empfindlichkeit" der Muskelzellen wird konstant gehalten
3. sie tritt bei "Leerbewegungen" auf
4. sie tritt beim Heben schwerer Lasten auf
A. 1, 2 und 3 sind richtig
B. 1 und 3 sind richtig
C. 2 und 4 sind richtig
D. nur 4 ist richtig
E. alle sind richtig

Richtige Antwort: A

Frage 38
Die Auge-Hand-Koordination erfolgt:

1. im primren visuellen Kortex


2. im Cerebellum
3. in den Limbischen Strukturen
4. im posterioren parietalen Kortex

A. 1, 2 und 3 sind richtig


B. 1 und 3 sind richtig
C. 2 und 4 sind richtig
D. nur 4 ist richtig
E. alle sind richtig

Richtige Antwort: D

Frage 39
Fr die trigeminale Chemorezeption gilt:

1. beim nasal-trigeminalen System sprechen freie Nervenendigungen an


2. die Empfindungsqualitten beim nasal-trigeminalen System sind beispielsweise 'brennend', 'stechend',
'beiend', 'khlend'
3. beim oral-trigeminalen System befinden sich Rezeptoren in der gesamten Mundhhle
4. das oral-trigeminale System reagiert auf chemische Stimuli auch mit Wrme- oder Klteempfindungen

A. 1, 2 und 3 sind richtig


B. 1 und 3 sind richtig
C. 2 und 4 sind richtig
D. nur 4 ist richtig
E. alle sind richtig

Richtige Antwort: E

Frage 40
Fr das dopaminerge nigrostriatale System gilt:

1. GABA- und Substanz-P-hltige Neurone haben D1-Rezeptoren und werden ber Dopamin erregt
2. Dopaminmangel fhrt zu Parkinson
3. GABA- und Enkephalinhltige Neurone haben D2-Rezeptoren und werden ber Dopamin gehemmt.
4. Dopaminmangel fhrt zu Intentionstremor

A. 1, 2 und 3 sind richtig


B. 1 und 3 sind richtig
C. 2 und 4 sind richtig
D. nur 4 ist richtig
E. alle sind richtig

Richtige Antwort: A

Frage 41
Rauschen ist ein Schallereignis:

1. das aus zahlreichen zusammenhanglosen Frequenzen besteht


2. mit einer ausgeprgten Periodizitt im Zeitverlauf
3. das bei der Hruntersuchung zur "Verdeckung" des berhrens am gesunden Ohr verwendet werden
kann
4. das die Basilarmembran lokal an nur einer Stelle anregt

A. 1, 2 und 3 sind richtig


B. 1 und 3 sind richtig
C. 2 und 4 sind richtig
D. nur 4 ist richtig
E. alle sind richtig

Richtige Antwort: B

Frage 42
Welche Hormone werden in der Adenohypophyse gebildet?

1. Somatotropin (STH)
2. ACTH
3. Prolaktin
4. Somatostatin (SIH)

A. 1, 2 und 3 sind richtig


B. 1 und 3 sind richtig
C. 2 und 4 sind richtig
D. nur 4 ist richtig
E. alle sind richtig

Richtige Antwort: A

Frage 43
Kalorische Prfung (Gleichgewicht) - welche Aussagen sind richtig:

1. beim Sitzen sollte der Kopf um 30 Grad nach vorgeneigt werden


2. Kaltsplung bewirkt einen Nystagmus auf die gleiche Seite
3. linksseitige Warmsplung bewirkt einen rechtsschlgigen Nystagmus
4. sie ermglicht die unabhngige Testung beider Seiten

A. 1, 2 und 3 sind richtig


B. 1 und 3 sind richtig
C. 2 und 4 sind richtig
D. nur 4 ist richtig
E. alle sind richtig

Richtige Antwort: D

Frage 44
Fr Pupillenverengung (Miosis) gilt:

1. Acethylcholin ist der Transmitter bei der Erregungsbertragung auf den M. sphincter pupillae.
2. Sie ist ein Teil der Naheinstellungsreaktion.
3. Sie wird von Nervenfasern ausgelst, die mit dem N. oculomotorius verlaufen.
4. Bei Belichtung des Auges tritt sie auch im nichtbelichteten Auge auf.

A. 1, 2 und 3 sind richtig


B. 1 und 3 sind richtig
C. 2 und 4 sind richtig
D. nur 4 ist richtig
E. alle sind richtig

Richtige Antwort: E

Frage 45
Biaurales Hren, fr die Ortung einer Schallquelle im Raum spielen eine Rolle:

1. die Form der Ohrmuschel


2. Laufzeitunterschiede
3. Intensittsunterschiede
4. reflektierte otoakustische Emissionen

A. 1, 2 und 3 sind richtig


B. 1 und 3 sind richtig
C. 2 und 4 sind richtig
D. nur 4 ist richtig
E. alle sind richtig

Richtige Antwort: A

Frage 46
Welche Ionenpermeabilitt ist fr das Entstehen eines Aktionspotenzials (Hodgkin-Zyklus) verantwortlich?

A. die K+ - Permeabilitt
B. die Na+ - Permeabilitt
C. die Ca 2+ - Permeabilitt
D. die Cl- - Permeabilitt
E. gar keine. Das AP wird in seiner Entstehung durch Ionenpermeabilitten nicht beeinflusst

Richtige Antwort: B

Frage 47
Fr Geschmacksverstrker gilt:
1. Geschmacksverstrker sind Stoffe, die die geschmacklichen Eigenschaften anderer Stoffe verstrken
2. Geschmacksverstrker besitzen einen ausgeprgten Eigengeschmack
3. Glutamat zhlt zu den Geschmacksverstrkern
4. Glutamat kommt natrlicherweise in Lebensmitteln nicht vor

A. 1, 2 und 3 sind richtig


B. 1 und 3 sind richtig
C. 2 und 4 sind richtig
D. nur 4 ist richtig
E. alle sind richtig

Richtige Antwort: B

Frage 48
Fr die Basalganglien gilt:

1. Die Striatumneurone haben hohe Spontanaktivitt


2. Die Spontanaktivitt im Pallidum hemmt die thalamocorticalen Fasern
3. Durch Erregung des Striatums kommt es zur Zunahme der Aktivitt des Pallidums
4. Durch Erregung des Striatums kommt es zur Abnahme der Aktivitt des Pallidums

A. 1, 2 und 3 sind richtig


B. 1 und 3 sind richtig
C. 2 und 4 sind richtig
D. nur 4 ist richtig
E. alle sind richtig

Richtige Antwort: C

Frage 49
Die basolaterale Gruppe der Amygdalae spielt eine Rolle bei:

1. Aggression.
2. emotionellem Gedchtnis.
3. Lernen durch Belohnung.
4. Angst.

A. 1, 2 und 3 sind richtig


B. 1 und 3 sind richtig
C. 2 und 4 sind richtig
D. nur 4 ist richtig
E. alle sind richtig

Richtige Antwort: E

Frage 50
Wenn ein Auge eine Brennweite in Luft von 2 cm besitzt, betrgt seine Brechkraft:

A. 2 dpt
B. 5 dpt
C. 10 dpt
D. 20 dpt
E. 50 dpt

Richtige Antwort: E

Frage 51
Die Hhe des Aktionspotenzials einer Nervenfaser in mV:

1. hngt von der Entfernung der Messelektrode zum Axonhgel ab.


2. ist unabhngig vom Ableitungsort und damit immer gleich gro.
3. nimmt exponentiell mit der Entfernung zum Axonhgel ab.
4. betrgt ca. 100 mV.

A. 1, 2 und 3 sind richtig


B. 1 und 3 sind richtig
C. 2 und 4 sind richtig
D. nur 4 ist richtig
E. alle sind richtig

Richtige Antwort: C

Frage 52
Die Linse besitzt im allgemeinen eine chromatische Aberration.

Das hat zur Folge/Die Ursache dafr ist:

1. die Brechkraft der Linse hngt von der Wellenlnge ab.


2. die Brennweite der Linse hngt von der Wellenlnge ab.
3. die Linse hat fr blaues und rotes Licht eine andere Brennweite.
4. die Linse absorbiert polychromatisches Licht durch ihre Eigenfarbe

A. 1, 2 und 3 sind richtig


B. 1 und 3 sind richtig
C. 2 und 4 sind richtig
D. nur 4 sind richtig
E. alle sind richtig

Richtige Antwort: A

Frage 53
Der Corpus ciliare hat folgende Aufgaben:

1. Regulation des Lichteinfalls auf die Retina


2. Akkommodation
3. Produktion der Trnenflssigkeit
4. Aufhngung der Linse

A. 1, 2 und 3 sind richtig


B. 1 und 3 sind richtig
C. 2 und 4 sind richtig
D. nur 4 ist richtig
E. alle sind richtig

Richtige Antwort: C

Frage 54
Die Sehschrfe (Visus):

1. ist abhngig von der Umwelt-Helligkeit.


2. kann mit dem Augenspiegel ermittelt werden.
3. ist in verschiednene Netzhautbereichen verschieden.
4. hat die Einheit dpt.

A. 1 und 3 sind richtig


B. 1, 2 und 3 sind richtig
C. 2 und 4 sind richtig
D. nur 4 ist richtig
E. alle sind richtig

Richtige Antwort: A

Frage 55
Aktivierung des Parasympathicus bewirkt:

A. Verminderung der Dnndarmmotilitt


B. Konstriktion der Bronchialmuskulatur
C. Kontraktion des M. dilatator pupillae
D. Hemmung der Trnendrsensekretion
E. Verminderung der Sspeicheldrsensekretion

Richtige Antwort: B

Frage 56
Aufsteigende retikulre Aktivierungssysteme:

1. beeinflussen den sensorischen Zuflu ber den Thalamus


2. modulieren das kortikale und subkkortikale Erregungsniveau
3. modulieren den Wachheitstonus und leiten schnelle Weckreaktionen ein
4. entfalten ihre Wirkung vorwiegend ber ionotrope Rezeptoren

A. 1, 2 und 3 sind richtig


B. 1 und 3 sind richtig
C. 2 und 4 sind richtig
D. nur 4 sind richtig
E. alle sind richtig

Richtige Antwort: A

Frage 57
Der Overshoot des Aktionspotenzials der Nervenfaser:

A. liegt nahe dem Nernstpotential fr K+-Ionen


B. liegt nahe dem Nernstpotential fr Na+-Ionen
C. liegt nahe dem Nernstpotential fr Ca2+-Ionen
D. liegt nahe dem Nernstpotential fr Cl--Ionen
E. hngt von der Lnge der Nervenfaser ab

Richtige Antwort: B

Frage 58
Welche diagnostische Information erhlt man aus evozierten Potentialen:

1. Latenzzeit zwischen Reiz- und Ableitestelle


2. Strungen bei der Reizleitung
3. funktionelle Intaktheit einer sensorischen Struktur
4. Zusammenhang zwischen Reiz und Reizqualitt

A. 1, 2 und 3 sind richtig


B. 1 und 3 sind richtig
C. 2 und 4 sind richtig
D. nur 4 ist richtig
E. alle sind richtig

Richtige Antwort: A

Frage 59
Ein rechtsschlgiger Nystagmus wird ausgelst durch:

1. Drehung nach rechts (Drehstuhl)


2. Warmsplung links (kalorische Reizung)
3. Stop nach lngerer Linksdrehung (Frenzel-Brille)
4. Bewegung des visuellen Umfeldes nach rechts

A. 1, 2 und 3 sind richtig


B. 1 und 3 sind richtig
C. 2 und 4 sind richtig
D. nur 4 ist richtig
E. alle sind richtig

Richtige Antwort: B

Frage 60
Elektroenzephalogramm - welche Aussagen sind richtig?

1. verursachend sind Feldpotentiale infolge synaptischer Prozesse (kortikale Neurone)


2. Das EEG spiegelt den Aktivittszustand der Hirnrinde wieder
3. EEG-Wellen entstehen durch synchronisierte zellulre Aktivitt
4. im Wachzustand, bei geffneten Augen, treten Alpha-Wellen (8-13 Hz) verstrkt auf
A. 1, 2 und 3 sind richtig
B. 1 und 3 sind richtig
C. 2 und 4 sind richtig
D. nur 4 ist richtig
E. alle sind richtig

Richtige Antwort: A
Prfung E

Frage 1
Welche Aussage(n) zu den Fasersystemen ist (sind) richtig?

1. Die Ansa peduncularis besteht aus dem Pedunculus thalami post. und der Ansa lenticularis
2. Die Pars ant. und Pars post. der Cornrnissura ant. und die Ansa ienticularis verlaufen durch den
Gratiolerschen Kanal.
3. tm Crus post. der Capsula int. verlaufen unter anderern die Pedunculi thalami sup. und post. und der
Tractus temporooccipitopontinus.
4. Der Fasciculus uncinatus besteht aus Assoziationsfasem und windet sich um das hintere Ende des
Sulcus lateralis.

A. 1 und 2 sind richtig


B. 2 und 3 sind richtig
C. 3 und 4 sind richtig
D. 2 und 4 sind richtig
E. alle sind richtig

Richtige Antwort: B

Frage 2
Welche Aussage trifft nicht zu?
Das Trommelfell:

A. bertrgt die Schallwelle auf die Gehrknchelchen.


B. hat normalerweise eine perlgraue Farbe.
C. steht senkrecht zur Achse des knchemen ueren Gehrgangs.
D. trennt das Auenohr vom Mittelohr.
E. liegt etwas weniger als 4 cm medial der Ohrmuschel.

Richtige Antwort: C

Frage 3
Der Plexus sacralis liegt:

A. im Canalis sacralis
B. lateral an der Innenseite des Os ilium
C. an der ventralen Seite des Os sacrum
D. lateral an der Innenseite der Sitzbeinhcker
E. an der dorsalen Seite des Os sacrum

Richtige Antwort: C

Frage 4
Die Bruch-Membran findet sich zwischen:

A. Lamina choroidocapillaris und Pigmentepithel


B. Pigmentepithel und Photorezeptorzellen
C. Endothel und Stroma der Kornea
D. Nervenfaserschicht und Glaskrper
E. Lamina suprachoroidea und Sklera

Richtige Antwort: A

Frage 5
Ein astigmatisches Auge bricht in der Sagittalebene um eine Dioptrie stark, in
der Horizontalebene aber um eine Dioptrie zu schwach.
Diese Fehlsichtigkeit kann man korrigieren mit:

1. minus 1 dpt zylindrisch


2. minus 2 dpt zylindrisch and plus 1 dpt sphrisch hintereinander geschaltet
3. plus 1 dpt zylindrisch
4. plus 2 dpt zylindrisch and minus 1 dpt sphrisch hintereinander geschaltet

A. 1, 2 und 3 sind richtig


B. 1 und 3 sind richtig
C. 2 und 4 sind richtig
D. nur 4 ist richtig
E. alle sind richtig

Richtige Antwort: C

Frage 6
Der Liquor cerebrospinalis ist eine klare, eiweiarme und fast zellfreie Flssigkeit.
Hauptverantwortlich fr die Liquorproduktion ist /sind:

A. Meningealzellen
B. Arachnoidalzotten
C. Plexusepithel (Plexus choroideus)
D. Ziliarepithel (Processus ciliares)
E. protoplasmatische Astrozyten

Richtige Antwort: C

Frage 7
Welche Aussagen zur folgenden Abbidung ist (sind) falsch?

1. 2 zeugt die Radix ventralis, die die motorischen und vegetativen Fasern fhrt
2. 1 zeigt den Duralsack der auf der Hhe des 2. Sakralwirbels endet
3. Bei dem Prparat wirden die Wirbelkrper entfernt und man sieht das Rckenmark von ventral
4. 3 zeigt die Cauda equina, die sich zwischen dem 2. Lendenwirbel und 3. Sakralwirbel befindet

A. 1, 2 und 3 sind falsch


B. 1, 3 und 4 sind falsch
C. 1 und 3 sind falsch
D. 2 und 4 sind falsch
E. nur 2 ist falsch

Richtige Antwort: B

Frage 8
Welche Aussagen zum Hirnstamm ist (sind) richtig?

1. Der funktionelle Hirnstamm besteht aus Medulla oblongata, Pons, Cerebellum, Diencephaon und den
Basalganglien
2. Das Tegmentum des Hirnstammes enthlt nur afferente Bahnen
3. das Tectum mesencephali, das eigentlich nicht zum Hirnstamm gehrt, enthlt vor allem Kerngebiete
4. Im Pes des anatomischen Hirnstammes verlaufen ausschlielich afferente Bahnen

A. 1, 2 und 3 sind richtig


B. 2 und 3 sind richtig
C. 3 und 4 sind richtig
D. 1 und 4 sind richtig
E. alle sind richtig

Richtige Antwort: C

Frage 9
Welche Aussage(n) zur Sensibilitt ist (sind) richtig?

1. Der Lemniscus med. entsteht aus der Gesamtheit der 2. Neurone der bewuten Sensibilitt
2. Im mesencephaon liegt die Fortsetzung des Fasciculus gracilis hinter der des Fasciculus cunneatus
3. Der Tractus spinothalamicus luft ab dem Pons im Lemniscus med.
4. Das Stratum interolivare lemnisci stellt den Beginn des Lamniscus med. dar. Hier kreuzen die Fasern
des Ncl. gracilis hinten und die des Ncl cuneatus vorne.

A. 1 und und 2 sind richtig


B. 2 und 3 sind richtig
C. 3 und 4 sind richtig
D. 1 und 4 sind richtig
E. alle sind richtig

Richtige Antwort: A

Frage 10

Welche Aussage(n) folgenden Abbildung ist (sind) richtig?


1. 2 zeigt die Fovea rostralis des Sulcus limitans, in deren Bereich der Ncl. motorius n. V,
visceromotorischer Kern, liegt
2. 3 zeigt die Pyramiden, In denen die Tractus corticospinales nach unten verlaufen.
3. 1 zeigt das Trigonum n. vagi, in welchem der sensorische Kern des N. X liegt.
4. 4 zeigt den Pedunculus cerebellaris sup., der das Kleinhirn mit dem Pons verbindet.
A. 1, 3 und 4 sind richtig
B. nur 4 ist richtig
C. nur 3 ist richtig
D. nur 1 ist richtig
E. 2, 3 und 4 sind richtig

Richtige Antwort: D

Frage 11
Fr die Horner'sche Trias trifft zu:

A. Sie besteht aus hngendem Oberlid, enger Pupille und tiefliegendem Augapfel
B. Sie begrndet sich in einer Schdigung des N. vagus
C. Sie ist ein typisches Zeichen fr eine Schilddrsenberfunktion
D. Sie wird unter anderem durch eine Lhmung des M. sphincter pupillae hervorgerufen
E. Sie entsteht hufig bei einem Hypophysentumor

Richtige Antwort: A

Frage 12
Welche Aussage(n) zu den Hirnnervenkernen ist (sind) falsch?

1. Der Ncl. salivatorius sup, ist Endkern fr Geschmacksfasern aus den Nn. VII, IX und X.
2. Der Ncl. n. VII liegt ventrolateral des Colliculus facialis, und seine Fasern bilden um
den Abducenskern ein "inneres Fazialisknie".
3. Der Ncl motorius n. V ist ein in die Tiefe gelagertes GangIion und erhlt Fasern aus der Kaumuskulatur.
4. Der Ncl. pontinus n. V wirft das Tuber cinereum auf und ist ein Schmerzkern.

A. 1, 2 und 3 sind falsch


B. 1, 3 und 4 sind falsch
C. 2, 3 und 4 sind falsch
D. 1 und 4 sind falsch
E. alle sind falsch

Richtige Antwort: B
Frage 13
Folgende Aussage ber den Augapfel trifft zu:

A. Der Augapfel hat eine zweischichtige Wand.


B. Im Inneren des Augapfels befinden sich Trnenflssigkeit, Linse und Glaskrper
C. Die Tunica vasculosa bulbi wird auch Uvea genannt.
D. Der Sehnerv verlasst den Augapfel am gelben Fleck.
E. Am Discus nervi optici befindet sich die Stelle des schrfsten Sehens.

Richtige Antwort: C

Frage 14
Welche Aussage(n) zur folgenden Abbildung ist (sind) richtig?

1. 4 zeigt das Corpus pineale, das zum Epithalamus gehrt.


2. 1 zeigt das Corpus callosum, das Verbindungen zwischen benachbarten Hirnregionen/-lappen herstellt.
3. 3 zeigt den inneren Randbogen, der Efferenzen aus dem Hippocampus zu den Corpora mamillaria fhrt.
4. 2 zeigt die Cella media ventriculi lat., deren Boden vom Corpus nuclei caudati gebildet wird.

A. nur 1 ist richtig


B. nur 2 ist richtig
C. nur 3 ist richtig
D. 1 und 4 sind richtig
E. 2 und 3 sind richtig

Richtige Antwort: C

Frage 15
Welche Aussage(n) zur folgenden Abbildung ist (sind) richtig?

1. 2 zeigt den N. facialis, der im Kleinhirn-Brcken-Winkel austritt und das Gesicht sensibel innerviert
2. 1 zeigt den N. abducens, der im Sulcus bulbopontinus austritt und rein motorisch ist.
3. 4 zeigt den N. occulomotorius, der im Sulcus med. cruris cerebri austritt und motorische und vegetative
Fasern fhrt.
4. 3 zeigt den N. vestibulocochlearis, der motorisch und sensibel ist.
A. 1, 2 und 3 sind richtig
B. 1 und 3 sind richtig
C. 2, 3 und 4 sind richtig
D. 2 und 3 sind richtig
E. 2 und 4 sind richtig

Richtige Antwort: D

Frage 16
Wer wird vom N. gluteus superior nicht versorgt?

1. M. gluteus medius
2. M. tensor fasciae latae
3. M. gluteus minimus
4. M. gluteus maximus

A. 1, 2 und 3 sind richtig


B. 1 und 3 sind richtig
C. 2 und 4 sind richtig
D. nur 4 ist richtig
E. alle sind richtig

Richtige Antwort: D

Frage 17
Das Spinocerebellum:

1. erhlt Afferenzen vom Cerebellum


2. projeziert auf die Formatio reticularis
3. enthlt Informationen ber das Aktivittniveau im Thalamus
4. enthlt Informationen ber das Aktivittsniveau der Interneurone im Rckenmark

A. 1., 2. und 3. sind richtig


B. 1. und 3. sind richtig
C. 2. und 4. sind richtig
D. Nur 4. ist richtig
E. Alle 4 sind richtig

Richtige Antwort: C

Frage 18
Welche der genannter synaptischen Schaltstellen sind Teil der aufsteigenden Hrbahn?

1. Ncl. cochlearis dorsalis


2. Ncl. cochlearis ventralis
3. Ncl. lemnisci lateralis
4. Colliculi superiores

A. 1., 2. und 3. sind richtig


B. 1. und 3. sind richtig
C. 2. und 4. sind richtig
D. nur 4. ist richtig
E. alle sind richtig

Richtige Antwort: A

Frage 19
Welche Aussag(n) zum Diencephalon ist (sind) richtig?

1. Die Stria medullaris thalami gehrt zum limbischen System und geht hinten m das Trigonum habenulae
ber.
2. Der Ventriculus Monroi ist der vordeste Teil dos III Ventrikels und endet vorne am Telencephalon
medium.
3. Die Facies dorsalis thalami beginnt medial an der Stria medullaris thalami und endet lateral am Sulcus
telodiencephalicus. In welchem die Stria terminalis verluft.
4. Die Fissura transversa cerebri gehrt zum ueren Liquorraum.

A. 1. und 2. sind richtig


B. 2. und 3. sind richtig
C. 3. und 4. sind richtig
D. nur 4. richtig
E. alle sind richtig

Richtige Antwort: E

Frage 20
Welche Aussage(n) zur Capsula interna ist (sind) richtig?

1. Die Capsula interna liegt lateral des Subthalamus.


2. Ansa peduncularis und Commissura ant. sind infralentikulre Bahnen der Capsula int. und verlaufen
durch den Gartiolet'schen Kanal
3. Pedunculus thalami sup. und Tractus frontopontinus verlaufen im Crus ant. der Capsula int.
4. Die Capsula int. liegt zwischen Crus cerebri und Corona radiata

A. 1., 3. und 4. sind richtig


B. 1. und 4. sind richtig
C. nur 4. sind richtig
D. nur 1. sind richtig
E. 2. und 3. sind richtig

Richtige Antwort: B

Frage 21
Bei einer 36-jhrigen Patientin findet man am Unterschenkel einen 3 cm groen, unregelmig begrenzten
und dunkelbraun und schwarz pigmentierten Fleck, der letzten Monaten langsam an Gre angenommen
hat.

Welche Struktur ist dafr ist erster Linie pathogenetisch relevant?

A. Korneozyt
B. Stratum germinativum
C. Desmosomen
D. Langerhanszelle
E. Melanozyt

Richtige Antwort: E

Frage 22
Fr die Reiz-Empfindungs-Relation von sensorischen Systemen gilt:
1. zwischen Reiz und Empfindungsstrke besteht eine lineare Beziehung
2. Reiz und Empfindungsstrke verhaften sich indirekt (umgekehrt) proportional zueinander
3. die Empfindung ist unabhngig von der Reizstrke
4. die Empfindung ist proportional zum Logarithmus der relativen Reizstrke

A. 1., 2. und 3. sind richtig


B. 1. und 3. sind richtig
C. 2. und 4. sind richtig
D. nur 4. ist richtig
E. alle sind richtig

Richtige Antwort: D

Frage 23
Gleichgewicht - welche Symptome knnen nach einem akuten einseitigen Labyrinthausfall auftreten:

1. Schiefstellung des Kopfes in Richtung des Ausfalls


2. Fallneigung in Gegenrichtung des Ausfalls
3. Spontannystagmus
4. Verlust der Wahrnehmung einer Raumhlfte

A. 1., 2. und 3. sind richtig


B. 1. und 3. sind richtig
C. 2. und 4. sind richtig
D. nur 4. ist richtig
E. alle sind richtig

Richtige Antwort: B

Frage 24
Welche Aussage(n) zur folgenden Abbildung, ist (sind) richtig?

1. Die Ziffer 3 zeigt die Stria olfactoria med., die zum Gyrus, subcallosus verluft.
2. Die Ziffer 1 zeigt die beiden Fasciculli graciles (Goll), in deren afferente Bahnen verlaufen.
3. Die Ziffer 2 zeigt die beiden Crura cerebri. die den Pes mesencephali bilden, und in denen afferente
Bahnen verlaufen
4. Die Ziffer 4 zeigt den Pedunculus cerebellaris medius, in welchem Faserverbindunqen zwischen Pons
und Cerebellum verlaufen.
A. 1. und 2. sind richtig
B. 1. und 3. sind richtig
C. 1. und 4. sind richtig
D. 2. und 4. sind richtig
E. alle sind richtig

Richtige Antwort: C

Frage 25
Welche Aussage ber die ueren Augenmuskeln trifft zu?

A. Der M. rectus lat. adduziert den Bulbus oculi


B. Der M. rectus med. wird vom N. abducens innerviert
C. Der M. rectus inf. senkt den Bulbus oculi
D. Die Hauptaufgabe der Mm. recti bulbi ist die Rotation des Bulbus oculi
E. Der 4. Hirnnerv innerviert die meisten ueren Augenmuskeln

Richtige Antwort: C

Frage 26
Welche Aussage(n) zum Rckenmark ist (sind) richtig?

1. Der Fasciculus cuneatus leitet die Sensibilitt der unteren Krperhlfte und beginnt daher im Sakralmark
2. Der Sulcus dorsalis intermedius markiert die Eintrittsstelle der Radix dorsalis
3. Der Funiculus lat. wird durch das Cornu post. von den Funiculi postt. und durch den Sulcus
ventrolateralis vom Funiculus ant. getrennt.
4. Die Rckenmarkssegmente S1 - Co1 liegen auf Hhe des 1./2. Lumbalwirbels

A. 1. und 2. sind richtig


B. 2. und 3. sind richtig
C. 3. und 4. sind richtig
D. nur 4. ist richtig
E. alle sind richtig

Richtige Antwort: C

Frage 27
Welche Aussage(n) zur folgenden Abbildung ist (sind) richtig?
1.Die Ziffer 3 zeigt den Sulcus calcarinus, Sitz der Sehrinde
2.Die Ziffer 1 zeigt den Gyrus parahippocampalis, der gemeinsam mit dem Gyrus subcallosus, dem
Indusium griseum und dem Gyrus fasciolaris den ueren Randbogen bildet
3.Die Ziffer 2 zeigt das Splenium corporis callosi
4.Die Ziffer 4 zeigt das Uncusbndchen, das das Ende des Gyrus dentatus bildet und sich mit der Fimbria
fornicis verbindet

A. 1., 2. und 3. sind richtig


B. 2., 3. und 4. sind richtig
C. 1., 3. und 4. sind richtig
D. 3. und 4. sind richtig
E. alle sind richtig

Richtige Antwort: C

Frage 28
Welche Aussage(n) zum VNS ist (sind) richtig?

1. Das Ggl submandibulare ist ein parasympatisches Kopfganglion, an dem die Nerven fr die Ggll.
submandibularis und sublingualis umschalten
2. Der Truncus sympathicus liegt beidseits der Wirbelsule und reicht vom 8. Cervical- bis zum 2.
Lumbalwirbel
3. Vom Ncl. salivatorius sup. verlaufen parasympathische Fasern mit dem N. facialis zu den Gll.
submandibularis, sublingualis und lacrimalis
4. Das Ggl. oticum ist ein parasympathisches ganglion, an dem Fasern aus dem N. glossopharyngeus fr
die Gl. parotidea umschalten

A. 1., 2. und 3 sind richtig


B. 2., 3. und 4. sind richtig
C. 1., 3. und 4. sind richtig
D. 1. und 3. sind richtig
E. 2. und 4. sind richtig

Richtige Antwort: C

Frage 29
Welche Aussage(n) zum Vetrikelsystem ist (sind) richtig?

1. Die Taenia cerebelli verluft zum Pedunculus flocculi ber den freien Rand des velum medullare sup.
zum Nodulus und denselben Weg auf der gegenseite zurck.
2. Die mediale Begrenzung des Cornu inf., bilden Pes hippocampi und Tela choroidea; medial davon
befindet sich das Crus cerebri
3. Der Boden der Cella media wird vom lateralen Anteil der Dorsalflche des Thalamus gebildet
4. Das Dach des Ventriculus tertius wird von Tela choroidea gebildet; die Abrisslinie verluft in der Stria
terminalis

A. 1. und 2. sind richtig


B. 1. und 3. sind richtig
C. 1. und 4. sind richtig
D. 2. und 4. sind richtig
E. 2. und 3. sind richtig

Richtige Antwort: E

Frage 30
Welche der folgenden Aussagen ber Talgdrsen ist nicht zutreffend?

A. Talg (Sebum) enthlt abgestorbene Talgdrsenepithelzellen und wird in den Haartrichter der
Talgdrsen abgegeben.
B. Wachstum und Sekretionsttigkeit der Talgdrsen werden durch Androgene beeinflusst
C. Talgdrsen entstehen aus der Epidermis, von der zunchst Epithelzapfen in das daruntergelegene
Gewebe vorwachsen
D. Die Sekretionsttigkeit der Talgdrsenendstcke setzt mit dem Beginn der Geschlechtsreife ein
E. Talgdrsenepithelzellen werden laufend durch Mitosen von in der Keimschicht gelegenen Basalzellen
neu gebildet

Richtige Antwort: D

Frage 31
Im ersten Trimenon der Schwangerschaft beginnt der Umbau der ruhenden zur laktierenden Brustdrse.
In welcher der folgenden Strukturen finden sich die Stammzellen, von denen dieser Umbau zum
berwiegenden Teil ausgeht?

A. Ductus lactifer colligens


B. terminaler Ductus
C. Mantelbindegewebe
D. kleinere Ductus lactifert
E. tubuloalveolre Endstucke

Richtige Antwort: B

Frage 32
Welche der folgend aufgelisteten Zellen liegen nicht in der Epidermis?

A. Melanozyten
B. amakrine Zellen
C. Merkel-Zellen
D. Langerhans-Zellen
E. Keratinozyten

Richtige Antwort: B

Frage 33
Auf der spinalen Ebene des motorischen Systems gilt fr die Alpha-Gamma-
Koaktivierung:

1. Alpha- und Gamma-Motoneuronen sind gleichzeitig erregt


2. die "empfindlichkeit" der Muskelspindeln wird konstant gehalten
3. Sie tritt bei " Leerbewegungen" auf
4. sie tritt beim Heben schwerer Lasten auf

A. 1., 2. und 3. sind richtig


B. 1. und 3. sind richtig
C. 2. und 4. sind richtig
D. nur 4. ist richtig
E. alle sind richtig
Richtige Antwort: A

Frage 34
Luft- und Knochenleitung, welche Aussagen sind richtig?

1. beim normalen Hren ist die Luftleitung effektiver als die Knochenleitung
2. das Hren der eigenen Stimme erfolgt ausschlielich ber die Luftleitung
3. beim Weber Versuch wird die Knochenleitung getestet
4. Bei Knochenleitung erfolgt die Schallelnkopplung ber die Gehrsknchelchen des Mittelohrs

A. 1., 2. und 3. sind richtig


B. 1. und 3. sind richtig
C. 2. und 4. sind richtig
D. nur 4. ist richtig
E. alle sind richtig

Richtige Antwort: B

Frage 35
Welche der folgenden Aussage(n) ist (sind) richtig?

1. Der Ncl. motorius tegmenti besteht aus Kernen der Formatio reticularis
2. Die Tractus dentatothalamicus und cerebellorubralis gehren zum Tractus tegmentalis centralis
3. Die Tractus rubroolivaris und reticuloolivaris bilden zusammen die Stilling'sche Schere
4. Die Efferenzen des Kleinhirns treten ber den Pedunculus cerebellaris medius aus

A. nur 1. ist richtig


B. nur 2. ist richtig
C. nur 3. ist richtig
D. 2. und 4. sind richtig
E. 1. und 4. sind richtig

Richtige Antwort: A

Frage 36
Welche der folgenden Aussagen ber das Corti-Organ ist zutreffend?

A. Stereorzilien der ueren Haarzellen ragen in die Membrana tectoria


B. Stereozilien der inneren Haarzellen ragen in den Nuel-Raum
C. Von den inneren Phalangenzellen wird der innere Tunnel gebildet
D. Den ueren Haarzellen dienen die Pfeilerzellen als Sttzzellen
E. Fortstze der ueren Phalangenzellen bilden die Basilarmembran

Richtige Antwort: A

Frage 37
Normaler und ungestrter Nachtschlaf - welche der folgenden Aussagen treffen zu?

1. beginnt typisch mit dem Stadium 1 (hypnagoge Phase)


2. beginnt typisch mit einer lngeren REM-Phase
3. alle Schlafstadien werden mehrmals durchlaufen
4. endet typisch nach dem Stadium 4 (Tiefschlaf)

A. 1 + 2 +3 sind richtig
B. 1 + 3 sind richtig
C. 2 + 4 sind richtig
D. nur 4 ist richtig
E. alle sind richtig
Richtige Antwort: B

Frage 38
Welche der folgenden Aussagen zur Augenentwicklung ist nicht zutreffend?

A. Die Augenblschen entwickeln sich in der 4. Entwicklungswoche


B. Das Augenblschen induziert die Entwicklung der Linsenpiakode
C. Aus der inneren Augenbecherwand entsteht das Pigmentepithel
D. In der Augenbecherspalte ist die Arteria hyaloidea gelegen
E. Der Augenbecher entwickelt sich aus dem Augenblschen

Richtige Antwort: C

Frage 39
Die Kontraktion des Ziliarmuskels:

A. fhrt zu einer Erhhung des Augeninnendrucks


B. erweitert die Pupille
C. verursacht eine Abflachung der Linse durch vermehrten Zug auf die Zonulafasern
D. vermindert den Zug aus die Zonulafasern und fhrt zu einer Wlbung der Linse
E. kontrahiert die Pupille

Richtige Antwort: D

Frage 40
Der Plexus brachialis wird von Rr. ventrales aus den Segmenten:

A. C3 - C8
B. Cl - C4
C. C4 - C8
D. C5 - Th1
E. C6 - Th2

Richtige Antwort: D

Frage 41
Was trifft auf die Trunci des Plexus brachialis nicht zu:

A. Der Truncus superior entsteht aus den Segmenten C5 und C6.


B. Der Truncus infenor entsteht aus den Segmenten C8 und Thl.
C. C7 bildet aliein den Truncus medius.
D. Die Trunci liegen in der vorderen Skalenuslcke.
E. Aus den Trunci gehen drei Faszikeln hervor.

Richtige Antwort: D

Frage 42
Welche der folgenden Aussagen zum Trommelfell (Membrana typanica) ist nicht
zutreffend?

A. Es trennt den ueren Gehrgang vom Mittelohr ab.


B. Es ist auf beiden Seiten von Epithel berzogen.
C. Es ist typischerweise nervenfaser- und geflos.
D. Es enthlt zahlreiche kollagene Fibrillen.
E. Es bertrgt Schallwellen auf den Malleus (Hammer).
Richtige Antwort: C

Frage 43
Welche Behauptung trifft nicht zu?

Der N. peroneus (fibularis) communis:

A. luft an der Innenseite der Sehne des M. biceps femoris in Richtung Fibula.
B. versorgt die Flexoren am Unterschenkel.
C. teilt sich in den N. peroneus (fibularis) superficialis und profundus.
D. versorgt die Extensoren am Unterschenkel und die Mm. peronei.
E. betritt die Peroneusloge unter dem Caput fibulae.

Richtige Antwort: B

Frage 44
Welche Aussag(e) zu folgenden Abbildung ist (sind) falsch?

1. Die Ziffer 3 zeigt den Gyrus precentralis, Sitz des primren motorischen Rindenfeldes
2. Die Ziffer 4 zeigt die Pars orbitalis des Gyrus frontalis inf., Sitz des motorischen Sprachzentrums
3. Die Ziffer 1 zeigt den Gyrus angularis, Sitz des Lesezentrums
4. Die Ziffer 2 zeigt den Gyrus supramarginalis, Sitz des sensorischen Sprachzentrums

A. 2., 3. und 4. ist falsch


B. 2. und 3. sind falsch
C. 3. und 4. sind falsch
D. nur 3. ist falsch
E. nur 4. ist falsch

Richtige Antwort: A

Frage 45
Welche Aussage(n) zur folenden Abbildung ist (sind) richtig?

1. Die Ziffer 4 zeigt das diencephale Blschen


2. Die Ziffer 1 zeigt den primren Hemisphrenstiel
3. Die Ziffer 2 zeigt das Augenblschen
4. Die Ziffer 3 zeigt die Brckenbeuge
A. 1. und 2. sind richtig
B. 2. und 3. sind richtig
C. 3. und 4. sind richtig
D. nur 3. ist richtig
E. nur 4. ist richtig

Richtige Antwort: D

Frage 46
Dysfunktionen der Basalganglien uem sich als:

1. Chorea
2. Dyskenisie
3. Tics
4. Intentionstremor

A. 1., 2. und 3. sind richtig.


B. 1. und 3. sind richtig.
C. 2. und 4. sind richtig.
D. nur 4. ist richtig.
E. alle sind richtig.

Richtige Antwort: A

Frage 47
Was stimmt fr die Amplitudenmodulation?

A. die Lautstrke ndert sich periodisch


B. die Lautstrke ndert sich exponentiell
C. die Tonhhe ndert sich periodisch
D. die Tonhhe ndert sich exponentiell
E. die Tonhhe ndert sich linear steigend

Richtige Antwort: A
Frage 48
Welche der folgenden Aussagen ber Mikrogliazellen (Hortega-Zellen) ist zutreffend?

A. Sie sind Makrophagen, deren Vorluferzellen im Knochenmark entstehen


B. Bei Zerstrung von Nervengewebe proliferieren sie und bilden Glianarben
C. Sie tragen wesentlich zur Aufrechterhaltung der extrazellulren K+ - Konzentration bei
D. Sie besitzen FFortstze, deren Endauslufer sich an Blutgefe anlagern
E. Sie sind durch Gap junctions verbunden und bilden einen netzartigen Verband.

Richtige Antwort: A

Frage 49
Fr den Cortex gilt:

1. Der primr motorische Kortex entspricht dem Brodman Areal 4


2. Brodman Areal 17 Ist der primr visuelle Kortex
3. Der primr somatosensorische Kortex entspricht den Brodman Arealen 1, 2 und 3
4. Der primr auditorische Kortex entspricht dem Brodman Areal 18

A. 1., 2. und 3. sind richtig


B. 1. und 3. sind richtig
C. 2. und 4. sind richtig
D. nur 4. ist richtig
E. alle sind richtig

Richtige Antwort: A

Frage 50
Fr die Stbchen der Retina gilt:

1. sie haben ein Ruhepotential von -30mV


2. sie depolarisieren bei Belichtung
3. die Na+ - Leitfhigkeit der ueren Stbchenmembran nimmt bei Belichtung ab
4. die Transmitterfreisetzung In der synaptischen Region der Stbchen nimmt bei Belichtung zu

A. 1., 2. und 3. sind richtig


B. 1, und 3. sind richtig
C. 2. und 4. sind richtig
D. nur 4. ist richtig
E. alle sind richtig

Richtige Antwort: B

Frage 51
Ein Bndel von Nervenfasern im PNS wird umgeben von

A. Perineurium
B. Neurofibrillen
C. Schwann-Zellen
D. Myelinscheiden
E. Mantelzellen

Richtige Antwort: A

Frage 52
Welche Aussage(n) zur folgenden Abbildung ist (sind) falsch?
1. Die Ziffer 2 zeigt die Fissura telodiencephalica, die zum ueren Liquorraum gehrt.
2. Die Ziffer 3 zeigt den infralentikularen Teil der Capsula int., in dem dem Radiationes optica und acustica
verlaufen.
3. Die Ziffer 1 zeigt den Truncus corporis callosi, der das Cavum Vergae nach oben begrenzt.
4. Die Ziffer 4 zeigt den Sulcus calcarinus, Sitz der Sehnnde.

A. 1., 2. und 3. sind falsch


B. 1., 3. und 4. sind falsch
C. 1. und 3. sind falsch
D. 1. und 2. sind falsch
E. nur 3. ist falsch

Richtige Antwort: A

Frage 53
Welche Aussage(n) zur Sensibilitt ist (sind) richtig?

1. Der Tractus spinothalamicus beginnt in der Substantia gelantinosa Rolandi und ist die Nebenbahn fr
Druck und Berhrung
2. Die Hinterstrnge leiten Druck und Berhrung protopathisch, Schmerz und Temperatur epikritisch und
die bewusste Tiefensensibilitt
3. Die Hinterwurzelfasern entstehen aus den peripheren Neuriten pseudounipolarer Nervenzellen
4. Der Tractus spinocerebellaris bildet das 2. Neuron der unbewussten Tiefensensibilitt

A. 1. und 2. sind richtig


B. 2. und 3. sind richtig
C. 3. und 4. sind richtig
D. 1. und 4. sind richtig
E. alle sind richtig

Richtige Antwort: D

Frage 54
Welche Aussage(n) zur folgenden Abbildung ist (sind) richtig?

1. Die Ziffer 4 zeigt den Gyrus parahippocampalis, der Teil des Gyrus fornicatus ist.
2. Die Ziffer 1 zeigt den Tractus opticus, der am Corpus geniculatum lat. endet.
3. Ziffer 2 zeigt die Cella media ventriculi lat., deren mediale Wand von Fornix und Tela choroidea gebildet
wird.
4. Die Ziffer 3 zeigt das Cornu inf. ventriculi lat. dessen mediale Wand von Pes
hippocampi und Tela choroidea gebildet wird.

A. nur 1. ist richtig


B. nur 2. ist richtig
C. nur 3. ist richtig
D. 1. und 4. sind richtig
E. alle sind richtig

Richtige Antwort: A

Frage 55
Welche Aussage(n) zu den Pyramidenbahnen ist (sind) richtig?

1. Der Tractus corticonuclearis des N. VII verluft zum ispsi - und kontralateralen rostralen Kern
2. Ein Verschluss der A. cerebri ant. fhrt zu einer beinbetonten kontralateralen Hemiparese und
Hemiansthesie, sowie einer zentralen Blaseninkontinenz
3. Der tractus corticonuclearis fr den N. hypoglossus verluft durch das Centrum semiovale, die Capsula
int. und das Crus cerebri, kreuzt zur Gegenseite und endet am Ncl. n. hypoglossi
4. Der Tractus corticospinalis luft in der Pars ventralis pontis und bildet hier die Fibrae longitudinales
pontis

A. 1., 2. und 3. sind richtig


B. 2. und 3. sind richtig
C. 2., 3. und 4. sind richtig
D. 3. und 4. sind richtig
E. alle sind richtig

Richtige Antwort: C

Frage 56
Welche Aussage(n) zum Mesencephalon ist (sind) richtig?

1. Der Ncl. mesencephalicus n. V liegt lateral des Ncl. n. III; hier erfolgt die Umschaltung von Fasern aus
der Kaumuskulatur
2. Der Tractus tectospinalis ist eine optische und akustische Reflexbahn und kreuzt in der Decussatio
tegmenti dorsalis
3. Das Trigonum lemnisci wird nach ventral durch den Sulcus lat. cruris cerebri und nach dorsal unten
durch das Brachium colliculi inf. begrenzt
4. Der Fasciculus longitudinalis med. ist fr die Gleichgewichtserhaltung des Kopfes und aufrechte
Netzhautbilder verantwortlich und kreuzt in der Decussatio tegmenti ventralis

A. 1. und 4. sind richtig


B. 2. und 3. sind richtig
C. nur 2. ist richtig
D. nur 3. ist richtig
E. alle sind richtig

Richtige Antwort: C

Frage 57
Welche Bogengnge findet man im Innenohr?

1. Ductus semicircularis ant.


2. Ductus semicircularis inf.
3. Ductus semicircularis lat.
4. Ductus semicircularis med.

A. 1., 2. und 3. sind richtig


B. 1. und 3. sind richtig
C. 2. und 4. sind richtig
D. nur 4. ist richtig
E. alle sind richtig

Richtige Antwort: B

Frage 58
Welche Aussage zur Entwicklung des Nervensystems ist (sind) richtig?

1. Aus der Intermedirzone des Rckenmarks entstehen die Flgelplatte mit den Afferenzen und die
Grundplatte mit den Efferenzen.
2. Die Scheitelbeuge entsteht als Abfaltung des Neuralrohres im Bereich des mesencephalen Blschens.
3. Der primre Hemisphrenstiel besteht aus der Hirnsubstanz um das Foramen interventriculare der
Verschmelzung des Di- und Telencephalon.
4. Aus dem Telencephalon medium entstehen das Corpus callosurn, der Fornix und das Septum
pellucidum.

A. 1., 2. und 3. sind richtig


B. 1., 3. und 4. sind richtig
C. 1. und 2. sind richtig
D. 2. und 3. sind richtig
E. 3. und 4. sind richtig

Richtige Antwort: C

Frage 59
Was trifft fr die bertragungsfunktion des Ohres zu?

A. Sie ist von der Richtung und Entfernung der Schallquelle sowie von der individuellen Anatomie des
Ohres abhngig
B. ist von der Richtung, aber nicht von der Entfernung der Schallquelle and nicht von der individuellen
Anatomie des Ohres abhngig
C. ist nicht von der Richtung, aber sowohl von der Entfernung der Schallquelle als auch von der
individuellen Anatomie des Ohres abhngig
D. ist von der Richtung und Entfernung der Schallquelle aber nicht von der individuellen Anatomie des
Ohres abhngig
E. ist nur von der individuellen Anatomie des Ohres abhngig
Richtige Antwort: A

Frage 60
Fr die Alpha-Gamma-Feedbackkontrolle des motorischen Systems gilt:

1. die Zielvorgabe (Endpunkt des Weges) erfolgt durch Aktivierung der Gamma - Motoneuronen
2. der kontraktile Anteil der Muskelspindeln wird verkrzt und dadurch das sensible Element aktiviert
3. Alpha - Motoneuronen aktivieren mit zunehmender Intensitt Arbeitsmuskulatur
4. die Alpha - Aktivierung wird gestoppt, wenn die afferente Aktionspotentialfolge ber die die 1a - Fasern
endet

A. 1., 2. und 3. sind richtig


B. 1. und 3. sind richtig
C. 2. und 4. sind richtig
D. nur 4. ist richtig
E. alle sind richtig

Richtige Antwort: E
Prfung F

Frage 1
Fr das olfaktorische System gilt:

1. Riechsinneszellen sind primre Sinneszellen, die direkt in den bulbus olfactorius projezieren
2. Riechzellen durchlaufen einen Lebenszyklus von ca. 60 Tagen.
3. an der Transduktion vom Duftreizen in elektrische signale nehmen "Second-messenger-
Systeme" teil
4. die Empfindungsschwelle ist fr alle Geruchsstoffe konstant

A. 1, 2 und 3 sind richtig


B. 1 und 3 sind richtig
C. 2 und 4 sind richtig
D. nur 4 ist richtig
E. alle sind richtig

Richtige Antwort: A

Frage 2
Aktivierung des Parasympathicus bewirkt nicht:

A. Erschlaffung der Brochialmuskulatur


B. Steigerung der Dnndarmmotalitt
C. Kontraktion des M. sphincter pupillae
D. Steigerung der Trnendrsensekretion
E. Steigerung der Speicheldrsensekretion

Richtige Antwort: A

Frage 3
Fr die Amygdala gilt:

1. hat Verbindungen zu den viszeralen Kernen des Hirnstamms.


2. ist ein einzelner Nucleus in den limbischen Strukturen.
3. ist bei emotionalen Gedchtnisprozessen beteiligt.
4. gehrt zu den Basalganglien.

A. 1, 2 und 3 sind richtig


B. 1 und 3 sind richtig
C. 2 und 4 sind richtig
D. nur 4 ist richtig
E. alle sind richtig

Richtige Antwort: B

Frage 4
Kompensatorische Blickbewegungen - welche Aussagen sind richtig:

1. werden vestibulr durch Kopfbewegungen ausgelst


2. werden auf Rckenmarksebene generiert
3. werden durch Verschiebung der Bildinformation auf der Fovea centralis ausgelst
4. werden kortikal im frontalen Augenfeld generiert

A. 1, 2 und 3 sind richtig


B. 1 und 3 sind richtig
C. 2 und 4 sind richtig
D. nur 4 ist richtig
E. alle sind richtig
Richtige Antwort: B

Frage 5
Gleichgewicht - welche Symptome knnen nach einem einseitigen Labyrinthausfall auftreten:

1. Fallneigung gegen die Richtung des Ausfalls


2. Schiefstellung des Kopfes in Richtung des Ausfalls
3. Verlust der Wahrnehmung einer Raumhlfte
4. Spontannystagmus

A. 1, 2 und 3 sind richtig


B. 1 und 3 sind richtig
C. 2 und 4 sind richtig
D. nur 4 ist richtig
E. alle sind richtig

Richtige Antwort: C

Frage 6
Motorik - Kontrollinstanzen, welche Aussagen sind richtig?

1. es existiert eine parallele und hierarchische Organisation


2. im Bereich zwischen Grohirnrinde und Hirnstamm existieren Rckkopplungen ber die Basalganglien
und ber das Kleinhirn
3. die hierarchische Organisation erlaubt das Zustandekommen von Reflexen ohne Beteiligung hherer
Zentren
4. der Hirnstamm moduliert die Aktivitt der Motoneuronen durch mediale und laterale Bahnen (Parallelitt)

A. 1, 2 und 3 sind richtig


B. 1 und 3 sind richtig
C. 2 und 4 sind richtig
D. nur 4 ist richtig
E. alle sind richtig

Richtige Antwort: E

Frage 7
Fr das Gesichtsfeld gilt:

1. Der Randbereich des Gesichtsfeldes ist farbenblind.


2. Eine vollstndige Durchtrennung aller kreuzenden Fasern im Chiasma nervi optici verursacht einen
bitemporalen Gesichtsausfall.
3. Die Papillae nervi optici liegen auf nichtkorrespondierenden Netzhautstellen.
4. Der blinde Fleck liegt in der nasalen Gesichtshlfte.

A. 1, 2 und 3 sind richtig


B. 1 und 3 sind richtig
C. 2 und 4 sind richtig
D. nur 4 ist richtig
E. alle sind richtig

Richtige Antwort: A

Frage 8
Die Gesamtbrechkraft des menschlichen Auges:

1. hngt wesentlich vom Medium ab, das an die Hornhaut grenzt


2. hngt von den Krmmungen der Augenlinse ab
3. wird von der Krmmung der Hornhaut wesentlich mitbestimmt
4. ist dem Augendurchmesser direkt proportional

A. 1, 2 und 3 sind richtig


B. 1 und 3 sind richtig
C. 2 und 4 sind richtig
D. nur 4 ist richtig
E. alle sind richtig

Richtige Antwort: A

Frage 9
An der Schlaf-Wach-Steuerung sind folgende Strukturen wesentlich beteiligt:

1. Basalganglien
2. Thalamus-Hirnstamm-Komplex
3. Frontallappen
4. Nucleus suprachiasmaticus

A. 1, 2 und 3 sind richtig


B. 1 und 3 sind richtig
C. 2 und 4 sind richtig
D. nur 4 ist richtig
E. alle sin richtig

Richtige Antwort: C

Frage 10
Einen schwach leuchtenden Stern am nchtlichen Himmel sieht man nur, wenn man ihn nicht fixiert,
sondern an ihm vorbeischaut. Das liegt daran, dass:

1. das skoptische Sehen eine andere Empfindlichkeit hat als das photopische
2. die Stbchen schneller adaptieren als die Zapfen
3. bei Fixierungswechsel die laterale Umfeldhemmung (vorbergehend) verstrkt wird
4. die Lichtintensitt beim Dmmerungssehen unterschwellig fr die Zapfen der Fovea centralis retinae ist

A. 1, 2 und 3 sind richtig


B. 1 und 3 sind richtig
C. 2 und 4 sind richtig
D. nur 4 ist richtig
E. alle sind richtig

Richtige Antwort: D

Frage 11
Funktionen der Makulaorgane, welche Aussagen sind richtig:

1. sie registrieren Translations- und Winkelbeschleunigungen


2. bei Kopfbewegung werden schnelle Augenbewegungen generiert
3. sie tragen auch in Schwerelosigkeit zur Raumorientierung bei
4. sie registrieren die Stellung des Kopfes im Schwerefeld der Erde

A. 1, 2 und 3 sind richtig


B. 1 und 3 sind richtig
C. 2 und 4 sind richtig
D. nur 4 ist richtig
E. alle sind richtig

Richtige Antwort: D
Frage 12
Was trifft zu? Der konsensuelle Pupillenreflex:

A. wird durch Berhrung der Cornea ausgelst.


B. wird durch eine Kontraktion der Mm. ciliares beider Augpfel ausgelst.
C. wird ber beide Nn. occulomotor. ausgelst.
D. besteht aus einer Erweiterung der beleuchteten und unbeleuchteten Pupille.
E. ist ein pathologischer Reflex.

Richtige Antwort: C

Frage 13
Die Horner'sche Trias:

1. besteht aus Ptosis, Mydriasis und Exophthalmus.


2. begrndet sich in einer Sympathicusschdigung.
3. ist ein Zeichen einer Schilddrsenberfunktion.
4. begrndet sich in einer Innervationsstrung von M. tarsalis sup., M. dilatator pupillae und M. orbitalis.

A. 1, 2 und 3 sind richtig


B. 1 und 3 sind richtig
C. 2 und 4 sind richtig
D. nur 4 ist richtig
E. alle sind richtig

Richtige Antwort: C

Frage 14
Agnosie:

1. entsteht bei Strungen in den Basalganglien


2. entsteht bei Strungen im Parietal- und Temporallappen
3. ist eine Bewegungsunruhe
4. ist eine gestrte Objekterkennung

A. 1, 2 und 3 sind richtig


B. 1 und 3 sind richtig
C. 2 und 4 sind richtig
D. nur 4 ist richtig
E. alle sind richtig

Richtige Antwort: C

Frage 15
Fr chemische Synapsen gilt:

1. das ankommende Aktionspotential lst ber Ca++-Kanle eine Freisetzung von Transmittern aus
2. der Transmitter diffundiert an die subsynaptische Membran und bindet dort an Rezeptoren
3. der Transmitter im synaptischen Spalt wird ber Enzyme abgebaut
4. eine bestimmte Synapse kann durch unterschiedliche Transmittersubstanzen aktiviert werden

A. 1, 2 und 3 sind richtig


B. 1 und 3 sind richtig
C. 2 und 4 sind richtig
D. nur 4 ist richtig
E. alle sind richtig

Richtige Antwort: A
Frage 16
Die emotionale Bewertung eines Objekts:

1. erfolgt im Temporallappen
2. erfolgt im Okzipitallappen
3. bentigt Afferenzen aus dem Hippocampus
4. bentigt Afferenzen aus den Basalganglien

A. 1, 2 und 3 sind richtig


B. 1 und 3 sind richtig
C. 2 und 4 sind richtig
D. nur 4 ist richtig
E. alle sind richtig

Richtige Antwort: B

Frage 17
Dass Muskeln zur richtigen Zeit mit der richtigen Kraft kontrahieren (Timing von Bewegungen):

1. ist Aufgabe der Gamma-Motoneurone


2. wird ausschlielich im primr motorischen Cortex gesteuert
3. ist Hauptaufgabe des supplementr motorischen Areals
4. ist eine cerebellre Funktion

A. 1, 2 und 3 sind richtig


B. 1 und 3 sind richtig
C. 2 und 4 sind richtig
D. nur 4 ist richtig
E. alle sind richtig

Richtige Antwort: D

Frage 18
Die inneren Haarsinneszellen haben folgende Funktion:

1. sie erzeugen otoakustische Emissionen


2. am Amplitudenmaximum der Wanderwelle zeigen sie maximale Empfindlichkeit
3. sie erzeugen eine Gegenbewegung gegen den Schall
4. sie wandeln Schall in einen Sinnesreiz

A. 1, 2 und 3 sind richtig


B. 1 und 3 sind richtig
C. 2 und 4 sind richtig
D. nur 4 ist richtig
E. alle sind richtig

Richtige Antwort: C

Frage 19
Fr die Reiz-Empfindungs-Relation von sensorischen Systemen gilt:

1. zwischen Reiz und Empfindlichkeitsstrke besteht eine lineare Beziehung


2. Reiz und Empfindungsstrke verhalten sich indirekt (umgekehrt) proportional zueinander
3. die Empfindung ist unabhngig von der Reizstrke
4. die Empfindung ist proportional zum Logarithmus der relativen Reizstrke

A. 1, 2 und 3 sind richtig


B. 1 und 3 sind richtig
C. 2 und 4 sind richtig
D. nur 4 ist richtig
E. alle sind richtig

Richtige Antwort: D

Frage 20
Normaler und ungestrter Nachtschlaf - welche der folgenden Aussagen treffen zu?

1. beginnt typisch mit einer lngeren REM-Phase


2. beginnt typisch mit dem Stadium 1 (hypnagoge Phase)
3. endet typisch nach dem Stadium 4 (Tiefschlaf)
4. alle Schlafstadien werden mehrmals durchlaufen

A. 1, 2 und 3 sind richtig


B. 1 und 3 sind richtig
C. 2 und 4 sind richtig
D. nur 4 ist richtig
E. alle sind richtig

Richtige Antwort: C

Frage 21
Richtung eines Nystagmus, welche Antworten sind richtig?

1. Drehtrommel, Rotation nach links bewirkt linksschlgigen Nystagmus


2. Drehstuhl, Rotation nach rechts bewirkt perrotatorisch einen rechtsschlgigen Nystagmus
3. kalorische Reizung, Warmsplung links bewirkt einen rechtsschlgigen Nystagmus
4. Drehstuhl, Rotation nach rechts bewirkt postrotatorisch einen linksschlgigen Nystagmus

A. 1, 2 und 3 sind richtig


B. 1 und 3 sind richtig
C. 2 und 4 sind richtig
D. nur 4 ist richtig
E. alle sind richtig

Richtige Antwort: C

Frage 22
Fr Regelsysteme gilt:

1. es sind Systeme mit negativer Rckkopplung


2. die Regelgre wird trotz des Einflusses von Srgren auf einem vorgegebenen Wert
gehalten
3. die Vorstellung der Regelgre erfolgt auf Basis von Messungen
4. Regelsysteme sind statische Systeme

A. 1, 2 und 3 sind richtig


B. 1 und 3 sind richtig
C. 2 und 4 sind richtig
D. nur 4 ist richtig
E. alle sind richtig

Richtige Antwort: A

Frage 23
Fr autoregulative Prozesse im Kreislauf gilt:

1. es sind lokale Feedback Prozesse ohne Beteiligung des ZNS


2. die myogene Autoregulation spielt eine Rolle bei der Regulation der Nierendurchblutung
3. die metabolische Autoregulation ist fr die Kreislaufregulation bei Krperarbeit wesentlich
4. die sympathise Vasokonstriktion besitzt Vorrang gegenber der metabolischen Autoregulation

A. 1, 2 und 3 sind richtig


B. 1 und 3 sind richtig
C. 2 und 4 sind richtig
D. nur 4 ist richtig
E. alle sind richtig

Richtige Antwort: A

Frage 24
Wach-Schlafsteuerung ber subkortikale Systeme, welche Aussagen sind richtig?

1. im Wachzustand sind das cholinerge und das aminerge System aktiv


2. im non-REM-Schlaf nimmt die Aktivitt beider Systeme ab
3. im REM-Schlaf erreicht die cholinerge Aktivitt Wachniveau
4. im REM-Schlaf sinkt die aminerge Aktivitt wieder ab

A. 1, 2 und 3 sind richtig


B. 1 und 3 sind richtig
C. 2 und 4 sind richtig
D. nur 4 ist richtig
E. alle sind richtig

Richtige Antwort: E

Frage 25
Einen schwach leuchtenden Stern am nchtlichen Himmel sieht man nur, wenn man ihn nicht fixiert,
sondern an ihm vorbeischaut. Das liegt daran, dass:

1. das skoptische Sehen eine andere Empfindlichkeit hat als das photopische
2. die Stbchen schneller adaptieren als die Zapfen
3. die rezeptiven Felder beim photoptischen Sehen kleiner sind als beim skotopischen
4. die Lichtintensitt beim Dmmerungssehen unterschwellig fr die Zapfen der Fovea centralis retinae ist

A. 1, 2 und 3 sind richtig


B. 1 und 3 sind richtig
C. 2 und 4 sind richtig
D. nur 4 ist richtig
E. alle sind richtig

Richtige Antwort: D

Frage 26
Welche der folgenden Aussagen ber das gesunde menschliche Auge sind richtig?

1. auf der Netzhaut wird ein virtuelles, verkehrtes Bild entworfen


2. die Akkommodation des Auges wird durch Vernderung der Brechkraft bewerkstellt
3. das Auflsungsvermgen betrgt etwa eine Bogensekunde
4. die Pupille dient als strahlenbegrenzende Blende des Auges

A. 1, 2 und 3 sind richtig


B. 1 und 3 sind richtig
C. 2 und 4 sind richtig
D. nur 4 ist richtig
E. alle sind richtig

Richtige Antwort: C
Frage 27
Bei einem Mann wird der Fernpunkt bei 2 Meter und der Nahpunkt bei 1 Meter bestimmt:

1. Der Mann ist kurzsichtig (myop)


2. Der Mann ist alterssichtig (presbyop)
3. er hat eine Akkommodationsbreite von 0.5 Dioptrien
4. Er braucht bei Fernsicht (scharfes Sehen ferner Objekte) eine Zerstreuungslinse von 0.5 dpt (also -0.5
dpt)

A. 1, 2 und 3 sind richtig


B. 1 und 3 sind richtig
C. 2 und 4 sind richtig
D. nur 4 ist richtig
E. alle sind richtig

Richtige Antwort: E

Frage 28
Fr die Beziehung zwischen Reiz und Empfindung im Bereich der Sinneswahrnehmung gilt:

1. die Empfindung ist eine nichtlineare funktion der Reizstrke


2. rasche nderungen der Reizstrke fhren zu einer verstrkten Empfindung
3. bei lngerdauernden Reizen nimmt die sensorische Empfindung allmhlich ab
4. die Empfindung ist unabhngig von der Schwellenreizstrke des entsprechenden Reizes

A. 1, 2 und 3 sind richtig


B. 1 und 3 sind richtig
C. 2 und 4 sind richtig
D. nur 4 ist richtig
E. alle sind richtig

Richtige Antwort: A

Frage 29
Wichtige Aufgaben des Muskeldehnungsreflexes sind:

1. die schnelle Entfernung des Krperglieds von der schmerzauslsenden Stelle


2. die kontinuierliche Aufrechterhaltung des Krpers gegen die Schwerkraft
3. die Reduktion der Muskelspannung whrend schneller Bewegungen
4. die Stabilisierung der Muskellnge beim Halten eines Gegenstands

A. 1, 2 und 3 sind richtig


B. 1 und 3 sind richtig
C. 2 und 4 sind richtig
D. nur 4 ist richtig
E. alle sind richtig

Richtige Antwort: C

Frage 30
Ein linksschlgiger Nystagmus wird ausgelst:

1. durch Linksdrehung, in der Anfangsphase


2. durch Warmsplung am rechten Ohr
3. nach lngerer Rechtsdrehung mit Frenzelbrille
4. durch Bewegung des visuellen Umfeldes nach links (Drehtrommel)

A. 1, 2 und 3 sind richtig


B. 1 und 3 sind richtig
C. 2 und 4 sind richtig
D. nur 4 ist richtig
E. alle sind richtig

Richtige Antwort: B

Frage 31
Fr die Horner'sche Trias trifft zu

A. Sie besteht aus hngendem Oberlid, enger Pupille und tiefliegendem Augapfel
B. Sie begrndet sich in einer Schdigung des N. vagus
C. Sie ist ein typisches Zeichen fr eine Schilddrsen berfunktion
D. Sie wird unler anderem durch eine Lhmung des M sphincter pupillae hervorgerufen
E. Sie entsteht hufig bei einem Hypophysentumor

Richtige Antwort: A

Frage 32
Aktivierung des Parasympathicus bewirkt:

1. Steigerung der Trnendrsensekretion


2. Steigerung der Dnndarmmotilitt
3. Kontraktion des M. sphincter pupillae
4. Erschlaffung der Bronchialmuskulatur

A. 1, 2 und 3 sind richtig


B. 1 und 3 sind richtig
C. 2 und 4 sind richtig
D. nur 4 ist richtig
E. alle sind richtig

Richtige Antwort: A

Frage 33
Die basolaterale Gruppe der Mandelkerne hat folgende Funktionen:

1. Angst
2. emotionellem Gedchtnis
3. Lernen durch Belohnung/Bestrafung
4. Schmerz

A. 1, 2 und 3 sind richtig


B. 1 und 3 sind richtig
C. 2 und 4 sind richtig
D. nur 4 ist richtig
E. alle sind richtig

Richtige Antwort: A

Frage 34
Aufgaben des Mittelohrs! Welche Aussagen sind richtig?

1. bertragung akustischer Energie an das Innenohr


2. Impedanzanpassung Luft - Innenohrflssigkeit
3. Schutz des Innenohrs vor starker berbeschallung
4. Frequenzanalyse

A. 1, 2 und 3 sind richtig


B. 1 und 3 sind richtig
C. 2 und 4 sind richtig
D. nur 4 ist richtig
E. alle sind richtig

Richtige Antwort: A

Frage 35
Spinale Motoneurone (MN)! Welche Aussagen sind richtig?

1. es existiert eine somatotope Ordnung hinsichtlich verschiedener Muskelgruppen


2. typischerweise bilden mehrere 100 MN sulenartige motorische Kerne
3. die MN bilden die letzte Endstelle fr alle absteigenden Bahnen
4. die MN bilden auch aufsteigende Bahnen

A. 1, 2 und 3 sind richtig


B. 1 und 3 sind richtig
C. 2 und 4 sind richtig
D. nur 4 ist richtig
E. alle sind richtig

Richtige Antwort: A

Frage 36
Welche Ionenpermeabilitt bestimmt das Ruhepotenzial der Nervenfaser?

A. die K+ - Permeabilitt.
B. die Na+ - Permeabilitt
C. die Ca2+ - Permeabilitt
D. die Cl- - Permabililt
E. gar keine. Das Ruhepotenzial wird durch Ionenpermeabilitten nicht beeinflusst

Richtige Antwort:

Frage 37
Warum kann die Tonhhe eines Schallreizes unabhngig von der Lautstrke wahrgenommen werden?
1. weil die Nervenleitung von der Reizfrequenz abhngig ist
2. weil hohe Reizfrequenzen das Innenohr frher erreichen
3. weil die Transduktioskanle im takt der Reizfrequenz ffnen
4. weil verschiedene Frequenzen an verschiedenen Stellen der Basilarmembran abgebildet werden

A. 1, 2 und 3 sind richtig


B. 1 und 3 sind richtig
C. 2 und 4 sind richtig
D. nur 4 ist richtig
E. alle sind richtig

Richtige Antwort: D

Frage 38
Gleichgewicht - Welche Probleme sind typisch fr einen beidseitigen chronischen Labyrinthausfall?

1. bei geschlossenen Augen kommt es zu Strungen in der Geh- und Stehmotorik


2. der optokinetische Nystagmus nimmt ab bzw. ist kaum noch feststellbar
3. Es kommt zu Problemen bei der Raumorientierung insbesondere unter Wasser
4. Es kommt zu einer starken Seitenneigung verbunden mit Schwindel und belkeit

A. 1, 2 und 3 sind richtig


B. 1 und 3 sind richtig
C. 2 und 4 sind richtig
D. nur 4 ist richtig
E. alle sind richtig

Richtige Antwort: B

Frage 39
Die Basalganglien sind beteiligt an:

1. der Koordination von Bewegungsablufen


2. Emotionen
3. motorischem Lernen
4. kognitiven Prozessen

A. 1, 2 und 3 sind richtig


B. 1 und 3 sind richtig
C. 2 und 4 sind richtig
D. nur 4 ist richtig
E. alle sind richtig

Richtige Antwort: E

Frage 40
Reflexprfung - Fr den H-Reflex gilt:

1. er kommt durch Reizung der Muskelspindeln zustande


2. in der Reflexantwort zeigt sich die Funktion der Motoneurone
3. es handelt sich um einen polysynaptischen Reflex
4. der H-Reflex eignet sich zur Bestimmung der Nervenlaufzeit

A. 1, 2 und 3 sind richtig


B. 1 und 3 sind richtig
C. 2 und 4 sind richtig
D. nur 4 ist richtig
E. alle sind richtig

Richtige Antwort: C

Frage 41
Welche Zeichen sind charakteristisch fr eine REM-Schlafphase:

1. verstrkte Traumaktivitt
2. erschwerte Erweckbarkeit
3. schnelle Augenbewegungen
4. allgemein erhhter Muskeltonus

A. 1, 2 und 3 sind richtig


B. 1 und 3 sind richtig
C. 2 und 4 sind richtig
D. nur 4 ist richtig
E. alle sind richtig

Richtige Antwort: B

Frage 42
Die Iris des menschlichen Auges kann folgende Funktionen ausben:

1. Ausblendung achsenferner Strahlen


2. Erhhung der Brechkraft des Auges
3. Regulation des Lichteinfalles
4. Vernderung der Wlbung der Linsenvorderflche
A. 1, 2 und 3 sind richtig
B. 1 und 3 sind richtig
C. 2 und 4 sind richtig
D. nur 4 ist richtig
E. alle sind richtig

Richtige Antwort: B

Frage 43
Fr das olfaktorische System trifft zu:

1. Riechsinneszellen sind primre Sinneszellen


2. die Sinneszellen durchlaufen einen Lebenszyklus von ca. 120 Tagen
3. die Reiz-Empfindung-Relation ist annhernd logarithmisch
4. die Decodierung bzw. Klassifikation von Duftreizen findet auf der Rezeptorebene statt

A. 1, 2 und 3 sind richtig


B. 1 und 3 sind richtig
C. 2 und 4 sind richtig
D. nur 4 ist richtig
E. alle sind richtig

Richtige Antwort: B

Frage 44
Bei Nahakkommodation:

1. wird die Akkommodationsbreite verndert


2. verengen sich die Pupillen
3. wird die mechanische Spannung der Zonulafasern erhht
4. kommt es zur Kontraktion des M. ciliaris

A. 1, 2 und 3 sind richtig


B. 1 und 3 sind richtig
C. 2 und 4 sind richtig
D. nur 4 ist richtig
E. alle sind richtig

Richtige Antwort: C

Frage 45
Tonische Aktivierung funktioneller Extensoren durch Gebiete des Hirnstamms - welche Antworten sind
richtig:

1. die laterale Formatio reticularis (FR) wirkt hemmend auf Extensor Motoneurone
2. die mediale FR wirkt hemmend auf Extensor Motoneurone
3. die Vestibulariskerne wirken erregend auf Extensor Motoneurone
4. die Unterbrechung absteigender Einflsse auf die laterale FR fhrt zur sogenannten alpha-Starre

A. 1, 2 und 3 sind richtig


B. 1 und 3 sind richtig
C. 2 und 4 sind richtig
D. nur 4 ist richtig
E. alle sind richtig

Richtige Antwort: B

Frage 46
Nach dem Brechungsgesetz:

1. wird ein schrg einfallender Lichtstrahl beim bergang vom optisch dnneren zum optisch dichteren
Medium zum Lot gebrochen.
2. ist der Sinus des Einfallswinkels dividiert durch den Sinus des Winkels des gebrochenen Strahls gleich
dem Verhltnis er Lichtgeschwindigkeiten.
3. ist der Sinus des Einfallswinkels aus Vakuum dividiert durch den Sinus des Winkels des gebrochenen
Strahls in Materie gleich der Brechzahl der Materie.
4. wird ein schrg einfallender Lichtstrahl beim bergang vom optisch dichteren zum optisch dnneren
Medium vom Lot gebrochen.

A. 1, 2 und 3 sind richtig


B. 1 und 3 sind richtig
C. 2 und 4 sind richtig
D. nur 4 ist richtig
E. alle sind richtig

Richtige Antwort: E

Frage 47
Das Nernst Potential:

1. hngt von der Temperatur ab.


2. gibt das Membranpotential an, bei dem der makroskopische Ionenfluss fr die betreffende Ionensorte
zum Erliegen kommt.
3. hngt vom Konzentrationsgradienten fr die betreffende Ionensorte ab.
4. hngt von der Valenz der Ionensorte ab.

A. 1, 2 und 3 sind richtig


B. 1 und 3 sind richtig
C. 2 und 4 sind richtig
D. nur 4 ist richtig
E. alle sind richtig

Richtige Antwort: E

Frage 48
Optisch dichtere Stoffe:

1. absorbieren das Licht strker


2. haben eine grere Brechzahl als optisch dnnere
3. lassen weniger Licht durch als optisch dnnere
4. haben eine kleiner Lichtausbreitungsgeschwindigkeit als optisch dnnere

A. 1, 2 und 3 sind richtig


B. 1 und 3 sind richtig
C. 2 und 4 sind richtig
D. nur 4 ist richtig
E. alle sind richtig

Richtige Antwort: C

Frage 49
Welche Phasen des Aktionspotentials existieren tatschlich?

1. Aufstrich
2. Overshoot
3. Repolarisation
4. Underscore
A. 1, 2 und 3 sind richtig
B. 1 und 3 sind richtig
C. 2 und 4 sind richtig
D. nur 4 ist richtig
E. alle sind richtig

Richtige Antwort: A

Frage 50
Polarisiertes Licht kann man herstellen:

1. durch Reflexion des Lichtes an einer Glasplatte


2. durch ein Beugungsfilter
3. mit einem Nicol'schen Prisma
4. durch geeignete Linsenkombination

A. 1, 2 und 3 sind richtig


B. 1 und 3 sind richtig
C. 2 und 4 sind richtig
D. nur 4 ist richtig
E. alle sind richtig

Richtige Antwort: A

Frage 51
Bei folgenden Vorgngen spielt die beugung des Lichtes die entscheidende Rolle:

1. weies Licht wird durch ein Prisma in seine Spektralfarben zerlegt


2. weies Licht wird durch ein Beugungsgitter in seine Spektralfarben zerlegt
3. kohrentes Licht kann durch berlagerung eine Interferenzfigur erzeugen
4. ideale Lichtpunkte werden durch optische Systeme als kleine Scheibchen abgebildet

A. 1, 2 und 3 sind richtig


B. 1 und 3 sind richtig
C. 2 und 4 sind richtig
D. nur 4 ist richtig
E. alle sind richtig

Richtige Antwort: C

Frage 52
Die saltatorische Erregungsleitung:

1. bewirkt hohe Leitungsgeschwindigkeiten, auch bei relativ dnnen Nervenfasern.


2. stellt den Mechanismus der Erregungsleitung bei markhaltigen, myelinisierten Nervenfasern dar.
3. existiert nur bei Riesenaxonen von Tintenfischen.
4. stellt den Mechanismus der Erregungsleitung bei nicht-myelinisierten C-Fasern dar.

A. 1, 2 und 3 sind richtig


B. 1 und 3 sind richtig
C. 2 und 4 sind richtig
D. nur 4 ist richtig
E. alle sind richtig

Richtige Antwort: B

Frage 53
Fr die saltatorische Erregungsleitung trifft zu:
1. die Amplitude des Aktionspotentials bleibt ber die Nervenfaserdistanz annhernd konstant
2. sie tritt bei markhaltigen Nervenfasern auf
3. die Aktionspotentialneubildung erfolgt an den Ranvierschen Schnrringen (Internodien)
4. sie bedingt eine hohe Nervenleitungsgeschwindigkeit

A. 1, 2 und 3 sind richtig


B. 1 und 3 sind richtig
C. 2 und 4 sind richtig
D. nur 4 ist richtig
E. alle sind richtig

Richtige Antwort: E

Frage 54
Welche aussage zur Elektromotorischen Kraft (EMK) ist richtig?

A. Die EMK gibt es nicht.


B. Die EMK stellt die treibende Kraft fr den lonenfluss einer lonensorte ber die Zellmembran dar.
C. Die EMK fr eine lonensorte ist \om Nemstpotential fr diese lonensorte unabhngig.
D. Die EMK fr eine ionensorte ist vom Membranpotential unabhngig.
E. Die EMK hngt von der Leitfhigkeit der Zellmembran fr die spezifische lonensorte ab.

Richtige Antwort: E

Frage 55
Grere Leitungsgeschwindigkeit kann bei der myelinisierten Nervenfasern erreicht werden:

1. durch kleineren Durchmesser


2. durch greren Durchmesser.
3. durch niedrigere Temperatur.
4. durch hhere Temperatur.

A. 1, 2 und 3 sind richtig


B. 1 und 3 sind richtig
C. 2 und 4 sind richtig
D. nur 4 ist richtig
E. alle sind richtig

Richtige Antwort: C

Frage 56
Das 'Alles oder Nichts' Gesetz:

A. ist ein Begriff der in italo-amerikanischen Wildwestfilmen, nicht aber in der Elektrophysiologie,
verwendet wird.
B. besagt, dass Aktionspotentiale, wenn sie erst einmal enstanden sind, in allen Bereichen einer
Nervenfaser gleich gro sind.
C. besagt dass die Gre des fortgeleiteten Aktionspotentials exponentiell, milder
Lngs konstante der Nervenfaser, abnimmt
D. besagt dass die Gre des fortgeleiteten Aktionspotentials exonentiell, mit der Zeitkonstante der
Zellmembran, abnimmt
E. gibt es nicht

Richtige Antwort: B

Frage 57
Fr die Membranleitfhigkeit einer lebenden Zelle am Ruhepotential treffen folgende Aussagen zu:

1. die Kalium Leitfhigkeit ist sehr hoch


2. die Chlorid Leitfhigkeit ist hoch
3. die Natrium Leitfhigkeit ist gering
4. die Leitfhigkeit fr die Eiweiss-Anionen ist hoch

A. 1, 2 und 3 sind richtig


B. 1 und 3 sind richtig
C. 2 und 4 sind richtig
D. nur 4 ist richtig
E. alle sind richtig

Richtige Antwort: A

Frage 58
Fr das Aktionspotential einer erregbaren Zelle gilt:

1. es ist ein konstanter Ablauf von Depolarisation und Repolarisation


2. die Depolarisation beginnt mit einem K+-Einstrom
3. die Repolarisation erfolgt durch einen K+-Ausstrom
4. die Membranleitfhigkeit fr Na+ und K+ bleiben whrend des Ablaufs des Aktionspotentials konstant

A. 1, 2 und 3 sind richtig


B. 1 und 3 sind richtig
C. 2 und 4 sind richtig
D. nur 4 ist richtig
E. alle sind richtig

Richtige Antwort: B
Prfung G

Frage 1
Frage 1 von 60
Welche Aussage(n) zur Entwicklung des Nervensystems ist (sind) richtig?

1. Die Scheitel beuge ist eine Abfaltung des Neuralrohres im Bereich des Mesencephalons.
2. Aus dem Telencephalon medium entstehen das Corpus caliosum und Teile der Vorderwand des
Ventriculus tertius.
3. Die Mantelzone des Rckenmarks differenziert sich zur Flgelplatte mit den Efferenzen und zur
Grundplatte mit den Afferenten.
4. Der sekundre Hemisphrenstiel besteht aus der Hirnsubstanz um das Foramen Interventriculare und
der Verschmelzung des Di- und Telencephalon.

A. 1., 2. und 3. sind richtig


B. 1., 2. und 4. sind richtig
C. 2., 3. und 4. sind richtig
D. 2. und 4. sind richtig
E. alle sind richtig

Richtige Antwort: B

Frage 2
Dysfunktionen der Basalganglien uern sich als:

1. Chorea
2. Dyskinesie
3. Tics
4. Intentionstremor

A. 1., 2. und 3. sind richtig,


B. 1. und 3. sind richtig.
C. 2. und 4. sind richtig.
D. nur 4. ist richtig.
E. alle sind richtig.

Richtige Antwort: A

Frage 3
In welche Hauptstmme teilt sich der N. radialis?

A. In einen Ramus lateralis und medialis


B. In einen Ramus anterior und posterior
C. In einen Ramus superior und inferior
D. In einen Ramus superficialis und profundus
E. In einen Ramus muscularis und cutaneus

Richtige Antwort: D

Frage 4
Welche Aussage(n) zur folgenden Abbildung ist (sind) richtig?

1. Die Ziffer 4 zeigt das Indusium griseum, das zusammen mit dem Gyrus
paraterminalis, dem Gyrus fasciolaris und dem Gyrus dentatus den ueren Randbogen bildet.
2. Die Ziffer 1 zeigt den Gyrus cinguli, der zusammen mit der Area adolfactoria, dem Isthmus gyri cinguli
und dem Gyrus parahippocampalis den Gyrus fornicatus bildet.
3. Die Ziffer 2 zeigt die Commissura ant, deren Pars ant. die beiden Corpora
amygdaloidea und die beiden Schlfenpole mit einander verbindet.
4. Die Ziffer 3 zeigt das Velum medullare inf., das im bulbren Teil des IV. Ventrikels das Dach bildet.
A. 1, 2 und 3 sind richtig
B. 1 und 2 sind richtig
C. 2 und 3 sind richtig
D. 1, 3 und 4 sind richtig
E. alle sind richtig

Richtige Antwort: B

Frage 5
Was ist der Zwickersche Nachton?

A. Nach Beschallung durch einen Klang, dessen Spektrum eine Lcke aufweist, hrt man genau den
fehlenden Ton.
B. Nach Beschallung durch einen Ton einer bestimmten Frequenz, hrt man genau diesen Ton und gleich
laut.
C. Nach Beschallung durch einen Ton einer bestimmten Frequenz, hrt man genau diesen Ton, allerdings
lauter.
D. Nach Beschallung durch einen Ton mit variabler Frequenz, hrt man einen Ton, dessen Frequenz
genau der Frequenzvariation entspricht.
E. Nach Beschallung durch einen Klang, dessen Spektrum eine Lcke aufweist, hrt man nur den hheren
Anteil des Spektrums.

Richtige Antwort: A

Frage 6
Welche Aussage(n) zu den Ventriculi laterales ist (sind) richtig?

1. Die mediale Wand des Cornu post. wird vom Bulbus cornu post. gebildet, der vom Forceps post.
aufgeworfen wird.
2. Im Dach des Cornu inf. liegen die Stria terminalis (medial) und die Cauda nuclei caudati (lateral).
3. Der Boden der Cella media wird vom Ncl. caudatus gebildet.
4. Das Cornu post. wird nach unten durch das Trigonum collaterale. durch welches Fasern der Radiatio
optica verlaufen, begrenzt.

A. 1., 2. und 3. sind richtig


B. 2., 3. und 4. sind richtig
C. 1., 3. und 4. sind richtig
D. 1., 2. und 4. sind richtig
E. alle sind richtig

Richtige Antwort: D

Frage 7
Welche Aussage(n) zur Sensibilitt ist (sind) richtig?

1. Der Lemniscus medialis entsteht aus der Gesamtheit der zweiten Neurone der bewussten Sensibilitt.
2. Das Stratum Interolivare lemnisci stellt den Beginn des Lemniscus med. dar, Hier kreuzen die Fasern
des Ncl. gracilis hinten und die des Ncl. cuneatus vorne.
3. Der Tractus spinothalamicus luft ab der Medulla oblongata im Lemniscus medialis.
4. Im Mesencephalon liegen die Fasern aus dem Ncl. gracilis vor denen des Ncl.
cuneatus.

A. nur 1. ist richtig


B. 1. und 2. sind richtig
C. nur 3. Ist richtig
D. 3. und 4. sind richtig
E. 2., 3. und 4. sind richtig

Richtige Antwort: A

Frage 8
Welche Aussage(n) zur Capsula interna ist (sind) richtig?

1. Die Capsula int. reicht vom Cortex bis zu den Crura cerebri.
2. Die Capsula int. liegt lateral des Subthalamus.
3. Die Capsula int. liegt zwischen Corona radiata und Crus cerebri.
4. Im Crus post. der Capsula int. verlauten unter anderem der Pedunculus thaiami ant. und der Tractus
corticonudearis.

A. 1., 2. und 3. sind richtig


B. 2., 3. und 4. sind richtig
C. 1. und 2. sind richtig
D. 2. und 3. sind richtig
E. 3. und 4. sind richtig

Richtige Antwort: D

Frage 9
Welche Aussage(n) zum vegetativen Nervensystem ist (sind) richtig?

1. Der Ncl. dorsalis n. X ist der Ursprungskern der parasympathischen Fasern fr die Eingeweide bis zum
Cannon-Bhm'schen Punkt.
2. Vom Ncl. accessorius n. III entspringen Fasern, die den M. dilatator pupillae und M. ciliaris innervieren.
3. Das Ganglion stellatum ist ein sympathisches Ggl.. die Ganglien ciliare,
submandibulare und oticum sind parasympathische Ganglien.
4. Der Truncus sympathicus reicht von der Schdelbasis bis zum Steibein und besteht - analog zu den 31
Rckenmarkssegmenten - aus 31 Paar sympathischen Ganglien.

A. 1., 3. und 4. sind richtig


B. 2., 3. und 4. sind richtig
C. 1. und 2. sind richtig
D. 3. und 4. sind richtig
E. 1. und 3. sind richtig

Richtige Antwort: E

Frage 10
Luft- und Knochenleitung, welche Aussagen sind richtig?
1. Beim normalen Hren ist die Luftleitung effektiver als die Knochenleitung
2. das Hren der eigenen Stimme erfolgt ausschlielich ber die Luftleitung
3. Beim Weber Versuch wird die Knochenleitung getestet
4. Bei Knochenleitung erfolgt die Schalleinkopplung ber die Gehrsknchelchen des Mittelohrs

A. 1, 2 und 3 sind richtig


B. 1 und 3 sind richtig
C. 2 und 4 sind richtig
D. nur 4 ist richtig
E. alle sind richtig

Richtige Antwort: B

Frage 11
Welche Aussage(n) zur folgenden Abbildung ist (sind) falsch?

1. Die Ziffer 4 zeigt den Limbus Giacomini, der das Ende des Gyrus dentatus bildet und sich mit der
Fimbria fomicis verbindet.
2. Die Ziffer 3 zeigt den Sulcus calcarinus, der im Cornu post. ventriculi lateralis den Calcar avis aufwirft.
3. Die Ziffer 1 zeigt den Gyrus parahippocampalis. der gemeinsam mit dem Gyrus subcalosus, dem
Indusium griseum und dem Gyrus fasciolaris den ueren Randbogen bildet.
4. Die Ziffer 2 zeigt das Splenium corporis callosi.

A. 1., 2. und 4. sind falsch


B. 2. und 3. sind falsch
C. 1. und 2. sind falsch
D. nur 3. ist falsch
E. 2. und 4. sind falsch

Richtige Antwort: D

Frage 12
Der abgebildete histologischen Schnitt zeigt einen Querschnitt durch ein Rckenmark in einer Heidenhain-
Frbung.
Die dunkle Frbung der mit X markierten Regionen beruht hauptschlich auf dem
Vorkommen von vielen:
A. Nervenfasern
B. Blutkapillaren
C. Neurofibrillen
D. Neuriten
E. Nissl-Schollen

Richtige Antwort: A

Frage 13
Die Bruch-Membran findet sich zwischen

A. Endothel und Stroma der Kornea


B. Pigmentepithel und Photorezeptorzellen
C. Nervenfaserschicht und Glaskrper
D. Lamina suprachoroidea und Sklera
E. Lamina choroidocapillaris und Pigmentepithel

Richtige Antwort: E

Frage 14
Welche der folgenden Aussagen zur Augenentwicklung ist zutreffend?

A. Das Augenblschen entsteht aus dem Augenbecher.


B. Die Linsenplakode induziert die Entwicklung des Augenblschens.
C. Aus der inneren Schicht des Augenbechers entsteht das Pigmentepithel.
D. Der Augenbecher ist ber den Augen Becherstiel mit dem Oberflchenektoderm verbunden.
E. In der Augenbecherspalte ist die A. hyaloidea gelegen.

Richtige Antwort: E

Frage 15
Wichtige an der Blickmotorik beteiligte Areale und Strukturen Im Bereich des Hirnstamms sind:

1. die mesencephale retikulre Formation


2. der Nucleus praepositus hypoglossi
3. der Edinger-Westphal Kern
4. die paramediane pontine retikulre Formation

A. 1., 2. und 3. sind richtig


B. 1. und 3. sind richtig
C. 2. und 4. sind richtig
D. nur 4. ist richtig
E. alle sind richtig

Richtige Antwort: E

Frage 16
Welche Aussage(n) zu den Fasersystemen ist (sind) richtig?

1. Durch den Gratiolet'schen Kanal verlaufen die Ansa lenticularis, der Pedunculus thalami ant. und die
Commissura ant.
2. Der Fasciculus uncinatus besteht aus Assoziationsfasern und verbindet den Frontal- mit dem
Temporallappen.
3. Das Corpus callosum ist eine Kommissur des Isokortex und verbindet identische Areale beider
Hemisphren.
4. Die gesamte Sensibilitt verluft ber den Pedunculus thaiami sup. zur
" Krperfhlsphre".

A. 1., 2. und 4. sind richtig


B. 2., 3. und 4. sind richtig
C. 3. und 4. sind richtig
D. 1. und 2. sind richtig
E. alle sind richtig

Richtige Antwort: B

Frage 17
Welche der folgend aufgelisteten Zellen liegen nicht in der Epidermis?

A. Langerhans-Zellen
B. Melanozyten
C. amakrine Zellen
D. Keratinozyten
E. Merkel-Zellen

Richtige Antwort: C

Frage 18
Ein 17-jhriges Mdchen leidet darunter, dass es im Gesicht immer wieder entzndliche Kntchen
('Wimmerl', 'Pickel') hat. Es liegt das klassische Bild einer Acne vulgaris vor.
Welche Struktur ist dafr in erster Linie pathogenetisch relevant?

A. Kollagene Fasern
B. Mastzellen
C. Talgdrsen
D. Langerhanszelle
E. Melanozyt

Richtige Antwort: C

Frage 19
In der Abbildung sind verschiedene Strukturen mit Pfeilen und Ziffern markiert.
Welche Zuordnung Ziffer-Struktur ist nicht zutreffend?
A. 4 - Haarpapille
B. 3 - uere epitheliale Haarwurzelscheide
C. 1 - M. arrector pili
D. 5 - Talgdrse
E. 2 - innere epitheliale Haarwurzelscheide

Richtige Antwort: E

Frage 20
Ependymzellen bilden:

A. die Pia mater der Leptomeninx


B. den Plexus choroideus
C. Gliagrenzmembranen
D. die Auskleidung von Liquorrumen
E. die BIut-Hirn-Schranke

Richtige Antwort: D

Frage 21
Welche Aussage(n) zur Sensibilitt ist (sind) falsch?

1. Der Tractus spinothaiamicus entsteht aus den kurzen Hinterwurzelfasern und leitet epikritische Druck-
und Berhrungsempfindung und die bewute Tiefensensibilitt.
2. Protopathische Druck-, Berhrungs-, Schmerz- und Temperaturempfindung werden vom Tractus
spinocerebellaris geleitet, der aus mittellangen Hinterwurzelfasern entsteht.
3. Der Tractus nucleothalamicus leitet nur protopathische Druck- und Berhrungsempfindung.
4. Die unbewute Tiefensensibilitt wird vom Tractus bulbothalamicus geleitet, der aus den langen
Hinterwurzelfasem entsteht.

A. 1., 2. und 3. sind falsch


B. 2., 3. und 4. sind falsch
C. 1., 3. und 4. sind falsch
D. 1. und 2. sind falsch
E. alle sind falsch
Richtige Antwort: E

Frage 22
Welche der folgenden Aussage(n) ist (sind) richtig?

1. Ncl. motorius tegmenti Ist ein berbegriff fr alle motorischen Hirnnervenkerne im Tegmentum des
Hirnstammes.
2. Efferenzen des Kleinhirns treten ber den Pedunculus cerebellaris sup. aus.
3. Der Tractus tegmentalis centralis besteht aus den Tractus rubroolivaris und
reticuloolivaris.
4. Die Tractus dentatothalamicus und cerebellorubralis bilden zusammen die Stilling'sche Schere.

A. 1., 2. und 3. sind richtig


B. 2., 3. und 4. sind richtig
C. 2. und 4. sind richtig
D. 2. und 3. sind richtig
E. 1. und 4. sind richtig

Richtige Antwort: B

Frage 23
Welche Aussage(n) zu den Pyramidenbahnen ist (sind) richtig?

1. Bei zentraler Fazialisparese resultieren ein hngender Mundwinkel und ein fehlender Lidschluss.
2. Der Tractus corticonuclearis des N. VII verluft zum ipsi- und kontralateralen dorsalen Kern.
3. Der Tractus corticonuclearis fr den N. hypoglossus verluft durch das Centrum semiovale, die Capsula
int. und das Crus cerebri und endet am kontralateralen Ncl n. hypoglossi.
4. Der Tractus corticospinalis lauft In der Pars ventralis pontis und bildet hier die Fibrae longitudinales
pontis.

A. 1., 2. und 3. sind richtig


B. 2., 3. und 4. sind richtig
C. 1., 3. und 4. sind richtig
D. nur 4. ist richtig
E. alle sind richtig

Richtige Antwort: B

Frage 24
Welche Aussage(n) zu den Hirnnervenkernen ist (sind) richtig?

1. Der Ncl. ambiguus liegt im metencephalen Anteil der Fossa rhomboidea und ist der motorische Kern fr
die Nn. glossopharyngeus und vagus.
2. Der Ncl. n. IV liegt im Tegmentum mesencephali auf Hhe der Colliculi inferiores.
3. Der Ncl. spinalis n. v wirft das Tuberculum cinereum auf und ist ein Schmerzkem.
4. Der Ncl. salivatorius sup. liegt im metencephalen Anteil der Fossa rhomboidea. Seine Fasern verlaufen
mit dem N. glossopharyngeus zur Glandula parotidea.

A. 1.. 2. und 3. sind richtig


B. 1. und 2. sind richtig
C. 2. und 3. sind richtig
D. 3. und 4. sind richtig
E. alle sind richtig

Richtige Antwort: C

Frage 25
Gleichgewicht - welche Symptome knnen nach einem akuten einseitigen
Labyrinthausfall auftreten:
1. Schiefstellung des Kopfes in Richtung des Ausfalls
2. Fallneigung in Gegenrichtung zum Ausfall
3. Spontannystagmus
4. Verlust der Wahrnehmung einer Raumhlfte

A. 1, 2 und 3 sind richtig


B. 1 und 3 sind richtig
C. 2 und 4 sind richtig
D. nur 4 ist richtig
E. alle sind richtig

Richtige Antwort: B

Frage 26
Fr den Cortex gilt:

1. Der primr motorische Kortex entspricht dem Brodman Areal 4


2. Brodman Areal 17 ist der primr visuelle Kortex
3. Der primr somatosensorische Kortex entspricht den Brodman Arealen 1, 2 und 3
4. Der primr auditorische Kortex entspricht dem Brodman Areal 18

A. 1, 2 und 3 sind richtig


B. 1 und 3 sind richtig
C. 2 und 4 sind richtig
D. nur 4 ist richtig
E. alle sind richtig

Richtige Antwort: A

Frage 27
Welche Aussage(n) zum Rckenmark ist (sind) richtig?
1, Die Fasciculi proprii sind Bestandteile des Eigenapparates des Rckenmarks und werden von
Schaltneuronen gebildet.
2. Die Stiiling-Clark-Sule Ist eine Umschaltstelle fr Bahnen der Oberflchensensibilitat.
3. Die Rckenmarkssegmente S1 - Co1 liegen auf Hhe des 1./2. Lumbalwirbels.
4. Das Septum medianum post. besteht aus Glia und trennt die beiden Hinterstrnge von einander.

A. 1, 2 und 3 sind richtig


B. 1 und 2 sind richtig
C. 2 und 3 sind richtig
D. 3 und 4 sind richtig
E. nur 4 ist richtig

Richtige Antwort: D

Frage 28
Was trifft fr die Schallausbreitung durch den Knochen zu?

A. Sie Ist ca. 50dB niedriger als der Luftschallanteil.


B. Sie ist vernachlssigbar klein.
C. Sie ist ca. 5dB niedriger als der Luftschallanteil.
D. Sie ist ca. 5dB hher als der Luftschallanteil.
E. Sie ist ca. gleich hoch wie der Luftschallanteil.

Richtige Antwort: A

Frage 29
Reflexprfung - fr den H-Reflex (Beinextensoren) gilt:
1. er kommt durch elektrische Reizung von 1a- Afferenzen zustande
2. die M-Welle reflektiert die Funktion des spinalen Motoneurons
3. die H-Antwort eignet sich zur Bestimmung der Nervenlaufzeit
4. es handelt sich um einen polysynaptischen Reflex

A. 1, 2 und 3 sind richtig


B. 1 und 3 sind richtig
C. 2 und 4 sind richtig
D. nur 4 ist richtig
E. alle sind richtig

Richtige Antwort: B

Frage 30
Der N. dorsalis scapulae versorgt:

A. den M. scalenus medius


B. den M. sternocleidomastoideus.
C. den M. levator scapulae und die Mm. rhomboidei.
D. den M. trapezius.
E. die Haut an der Dorsalseite der Scapula.

Richtige Antwort: C

Frage 31
Welche der folgenden Aussagen ber Keratinozyten ist/sind zutreffend?

1. Sie stehen durch Desmosomen miteinander In Verbindung.


2. Sie besitzen zahlreiche dendritische Fortstze.
3. Sie enthalten Zytokeratinfilamente im Zytoplasma.
4. Sie synthetisieren Melanin in den Melanosomen.

A. nur 2. ist richtig


B. nur 1. und 3. sind richtig
C. nur 1., 2. und 3. sind richtig
D. nur 2. und 4. sind richtig
E. alle sind richtig

Richtige Antwort: B

Frage 32
Der Fasciculus mediaiis des Plexus brachialis entlsst folgende Nerven:

A. Radix medialis des N. medianus. N. ulnaris, N. radialis. N. cutaneus antebrachii medialis


B. Radix medialis des N. medianus, N. cutaneus brachii medialis, N. cutaneus antebrachii medialis, N.
axillaris
C. Radix medialis des N. medianus, N. ulnaris, N. axillaris. N. cutaneus brachii medialis, N. cutaneus
antebrachii medialis
D. N. axillaris, N. radialis, N ulnaris, Radix medialis des N. medianus
E. Radix medialis des N. medianus, N. ulnaris, N. cutaneus brachii medialis, N. cutaneus antebrachii
medialis

Richtige Antwort: E

Frage 33
Die Abbildung zeigt einen Ausschnitt aus dem vorderen Abschnitt des Bulbus oculi. Verschiedene
Strukturen sind mit Buchstaben markiert.
Welche der folgenden Aussagen dazu trifft zu?

A. Struktur A ist typischerweise frei von Blutgefen.


B. ber Struktur B kann die Trnenflssigkeit aus dem Kammerwinkel abflieen.
C. Struktur C ist an Ihrer Vorderflche von der Conjuntiva bulbi bedeckt.
D. Struktur D enthlt den M. sphincter und den M. dilatator pupillae.
E. Struktur E grenzt mit ihrer Hinterflche an die hintere Augenkammer.

Richtige Antwort: E

Frage 34
Ein Ausfall der Im Chiasma nervi optici kreuzenden Nervenfasern fhrt zu:

A. einer Erblindung eines Auges


B. einer Erblindung beider Augen
C. einer homonymen Hemianopsie
D. einer bitemporalen Hemianopsie
E. einem rhrenfrmigen Gesichtsfeld

Richtige Antwort: D

Frage 35
Fr Information aus korrespondierenden Netznautstellen gut:
1. sie wird In der selben Hemisphre der Sehrinde abgebildet
2. sie vermittelt den plastischen Unterschied zwischen rechtem und linkem Gesichtsfeld
3. sie vermittelt Information zur binokularen Fusion
4. sie kreuzt auf der zentripetale Bahn auf die kontralaterale Seite

A. 1, 2 und 3 sind richtig


B. 1 und 3 sind richtig
C. 2 und 4 sind richtig
D. nur 4 ist richtig
E. alle sind richtig

Richtige Antwort: B

Frage 36
Der N. ischiadicus verlasst das Becken durch das:

A. Foramen ischiadicum minus


B. Foramen suprapiriforme
C. Foramen obturatum
D. Foramen infrapiriforme
E. Foramen intervertebrale zwischen L5 und S1

Richtige Antwort: D

Frage 37
Welche Aussage(n) zum Mesencephalon ist (sind) richtig?

1. Der Tractus tectospinalis ist eine optische und akustische Reflexbahn und kreuzt in der Decussatio
tegmenti dorsalis.
2. Der Fasciculus longitudinalis med. reicht vom Mittelhirn bis ins obere Halsmark und ist unter anderem fr
aufrechte Netzhautbilder verantwortlich.
3. Der Tractus bulbothalamicus schliet sich im Tegmentum mesencephali dem Lemniscus med. an.
4. Das Trigonum lemnisci wird nach ventral durch den Sulcus lat. cruris cerebri und nach dorsal/unten
durch das Brachium colliculi inf. begrenzt.

A. 1., 2. und 3. sind richtig


B. 1. und 2. sind richtig
C. 3. und 4. sind richtig
D. 1. und 4. sind richtig
E. 1. und 3. sind richtig

Richtige Antwort: B

Frage 38
Fr Opticus-Neuronen gilt:

1. Das On-Zentrum Neuron reagiert bei Belichtung in der Peripherie seines rezeptiven Feldes mit
Hemmung
2. Das On-Zentrum Neuron reagiert bei Belichtung In der Peripherie seines rezeptiven Feldes mit
Aktivierung
3. Das Off-Zentrum Neuron reagiert bei Belichtung Im Zentrum des rezeptiven Feldes mit Hemmung
4. Das Off-Zentrum Neuron reagiert bei Belichtung im Zentrum des rezeptiven Feldes mit Aktivierung

A. 1, 2 und 3 sind richtig


B. 1 und 3 sind richtig
C. 2 und 4 sind richtig
D. nur 4 ist richtig
E. alle sind richtig

Richtige Antwort: B

Frage 39
Welche Aussage(n) zum Ventriculus quartus ist (sind) falsch?

1. Die Taenia cerebelli verluft vom Pedunculus flocculi ber den freien Rand des Velum medullare sup.
und ber den Nodulus zum Pedunculus flocculi der Gegenseite.
2. Das Dach wird im cerebellren Anteil vom Velum medullare sup., im bulbren Anteil vom Velum
medullare inf. gebildet.
3. Die Taenia bulbi verluft vom Obex ber die Tubercula gracile und cuneatum zum Pedunculus
cerebellaris Inf. und geht am Pedunculus flocculi in die Taenia cerebelli ber.
4. ber die Aperturae laterales und mediana besteht eine Verbindung mit dem
Subduralraum.

A. 1., 2. und 3. sind falsch


B. 2., 3. und 4. sind falsch
C. 1., 2. und 4. sind falsch
D. 2. und 4. sind falsch
E. alle sind falsch

Richtige Antwort: C
Frage 40
Zur Hrbahn zahlen folgende Strukturen:

1. Kerne des Lemniscus lateralis


2. Colliculi inferiores der Lamina tecti
3. Corpus trapezoideum
4. Corpus geniculatum mediale

A. 1, 2 und 3 sind richtig


B. 1 und 3 sind richtig
C. 2 und 4 sind richtig
D. nur 4 ist richtig
E. alle sind richtig

Richtige Antwort: E

Frage 41
Reiz-Empindungsrelationen:

1. charakterisieren die Empfindungsstrke als Funktion der Reizstrke von Sinnesreizen


2. werden durch nichtlineare Funktionen charakterisiert
3. knnen beispielsweise durch eine logarithmische Funktion charakterisiert werden
4. sind unabhngig von der Schwellen reizstrke des Sinnesreizes

A. 1, 2 und 3 sind richtig


B. 1 und 3 sind richtig
C. 2 und 4 sind richtig
D. nur 4 ist richtig
E. alle sind richtig

Richtige Antwort: A

Frage 42
Der N. tibialis versorgt:
1. den M. soleus
2. den M. gastrocnemius
3. den M. semitendinosus
4. den M. sartorius

A. 1., 2. und 3. sind richtig


B. 1. und 3. sind richtig
C. 2. und 4. sind richtig
D. nur 4. Ist richtig
E. alle sind richtig

Richtige Antwort: A

Frage 43
Welche Aussage(n) zum Rhinencephalon ist (sind) richtig?

1. Oberhalb der Area olfactoria liegen das Caput nuclei caudati und der vordere Teil der Capsula int.
2. Die Bandaletta diagonalis verbindet den Ncl. amygdale mit dem Trigonum olfactorium.
3. Die Stria olfactoria med. stellt eine Verbindung zwischen Rhinencephalon und emotionalem Gehirn her.
4. Die Geruchserkennung erfolgt Im Bereich des Ncl. amygdale und der periamygdalren Rinde, wo die
Stria olfactoria lateralis endet.

A. 1., 2. und 3. sind richtig


B. 2., 3. und 4. sind richtig
C. 1., 2. und 4. sind richtig
D. 1., 3. und 4. sind richtig
E. alle sind richtig

Richtige Antwort: D

Frage 44
Welche der folgenden Strukturen zahlen zu den Gehrknchelchen?
1. Incus
2. Malleolus
3. Stapes
4. Os auriculare

A. 1, 2 und 3 sind richtig


B. 1 und 3 sind richtig
C. 2 und 4 sind richtig
D. nur 4 ist richtig
E. alle sind richtig

Richtige Antwort: B

Frage 45
Welche Aussage(n) zum Rckenmark ist (sind) richtig?

1. Der Sulcus dorsolateralis markiert die Eintrittsstelle der Radix dorsalis.


2. Der Fasciculus cuneatus beginnt im Sakralmark und leitet die Sensibilitt der unteren Krperhlfte.
3. Der Funiculus lateralis wird durch das Cornu post. von den Funiculi postt. und durch den Sulcus
ventrolateralis vom Funiculus ant. getrennt.
4. Unterhalb des 2. Sakralwirbels liegen die Spinalganglien im Canalis sacralis.

A. 1., 2. und 3. sind richtig


B. 1., 3. und 4. sind richtig
C. 2., 3. und 4. sind richtig
D. 2. und 4. sind richtig
E. nur 4. ist richtig

Richtige Antwort: B

Frage 46
Welche der folgenden Strukturen gehrt NICHT zu einer markhaltigen Nervenfaser des PNS?

A. Neurit
B. Perineuralscheide
C. Ranvier-Schnrring
D. Schmidt-Lanterman-Einkerbung
E. Internodium

Richtige Antwort: B

Frage 47
Welche Aussage(n) zur folgenden Abbildung ist (sind) richtig?

1. Die Ziffer 3 zeigt die Pars centralis ventriculi lateralis.


3. Die Ziffer 4 zeigt den Tractus opticus.
2. Die Ziffer 1 zeigt das Caput nuclei caudati.
4. Die Ziffer 2 zeigt den Hypothalamus.
A. 1., 2. und 3. sind richtig
B. 2., 3. und 4. sind richtig
C. 1. und 3. sind richtig
D. 2. und 4. sind richtig
E. alle sind richtig

Richtige Antwort: B

Frage 48
Welche Aussage(n) zum Hirnstamm ist (sind) richtig?

1. Der funktionelle Hirnstamm besteht aus Medulla oblongata, Pons, Cerebellum, Diencephalon und den
Basalganglien.
2. Das Tegmentum des Himstammes enthlt nur afferente Bahnen.
3. Das Tectum mesencephali, das eigentlich nicht zum Hirnstamm gehrt, enthlt vor allem Kerngebiete.
4. Im Pes des anatomischen Hirnstammes verlaufen ausschlielich efferente Bahnen.

A. 1, 2 und 3 sind richtig


B. 2 und 3 sind richtig
C. 3 und 4 sind richtig
D. 1 und 4 sind richtig
E. alle sind richtig

Richtige Antwort: C

Frage 49
Welche Aussage(n) zur folgenden Abbildung ist (sind) richtig?

1. Die Ziffer 1 zeigt die beiden Fasciculi graciles (Goll).


2. Die Ziffer 3 zeigt die Stria olfactoria medialis. die zum Gyrus subcallosus verluft.
3. Die Ziffer 4 zeigt den Pedunculus cerebellaris medius. In welchem Faserverbindungen zwischen
Cerebellum und Pons verlaufen.
4. Die Ziffer 2 zeigt die beiden Crura cerebri, die das Tegmentum mesencephali bilden.
A. 1, 2 und 3 sind richtig
B. 1, 3 und 4 sind richtig
C. 2 und 3 sind richtig
D. 2 und 4 sind richtig
E. alle sind richtig

Richtige Antwort: C

Frage 50
Welche Aussage(n) zu den Hirnnerven kernen ist (sind) falsch?

1. Der Ncl. pontinus n. V ist Endkern fr Schmerzfasern aus dem Gesichtsbereich.


2. Der Ncl. n. VII liegt ventrolateral des Colliculus facialis, und seine Fasern bilden um den Abducenskern
das "innere Fazialisknie".
3. Der Ncl. salivatorius sup. ist Endkern fr Geschmacksfasern aus den Nn. VII, IX und X.
4. Der Ncl. motorius n. V ist ein in die Tiefe gelagertes Ganglion und erhlt Fasern aus der Kaumuskulatur.

A. 1., 2. und 3. sind falsch


B. 1., 2. und 4. sind falsch
C. 1., 3. und 4. sind falsch
D. 2., 3. und 4. sind falsch
E. alle sind falsch

Richtige Antwort: C

Frage 51
Normaler und ungestrter Nachtschlaf - welche der folgenden Aussagen treffen zu?

1. beginnt typisch mit dem Stadium 1 (hypnagoge Phase)


2. beginnt typisch mit einer lngeren REM-Phase
3. alle Schlaf Stadien werden mehrmals durchlaufen
4. endet typisch nach dem Stadium 4 (Tiefschlaf)

A. 1 + 2 + 3 sind richtig
B. 1 + 3 sind richtig
C. 2 + 4 sind richtig
D. nur 4 ist richtig
E. alle sind richtig
Richtige Antwort: B

Frage 52
Welche Aussage(n) zum Diencephalon ist (sind) richtig?

1. Die Facies dorsalis thalami beginnt medial an der Stria medullaris thalami und endet lateral am Sulcus
telodiencephalicus, in welchem die Stria terminalis verluft.
2. Die Facies basalis thalami geht vorne unten in den Subthalamus, hinten unten in den Hypothalamus
ber.
3. Die Facies dorsalis thalami schaut z.T. in den III. Ventrikel, z.T. in die Fissura telodiencephalica.
4. Der Subthalamus liegt zwischen Hypothalamus und Capsula interna.

A. 1., 2. und 3. sind richtig


B. 2., 3. und 4. sind richtig
C. 2. und 3. sind richtig
D. 1. und 4. sind richtig
E. alle sind richtig

Richtige Antwort: D

Frage 53
Ein Bndel von Nervenfasern im PNS wird umgeben von:

A. Neurofibrillen
B. Myelinscheiden
C. Schwann-Zellen
D. Perineurium
E. Mantelzellen

Richtige Antwort: D

Frage 54
Welche Aussage(n) zu den Ganglien ist (sind) richtig?

1. In einem bipolaren Ganglion erfolgt keine Umschaltung.


2. Ein pseudounipolares Ganglion enthlt Synapsen.
3. Ganglien existieren sowohl im zentralen als auch im peripheren Nervensystem.
4. Ein multipolares Ganglion enthlt Synapsen.

A. 1., 3. und 4. sind richtig


B. 1. und 4. sind richtig
C. 2. und 3. sind richtig
D. 3. und 4. sind richtig
E. nur 4. ist richtig

Richtige Antwort: B

Frage 55
Fr den Eigenreflex gilt:
1. er Ist ein monosynaptischer Reflex
2. er ist nicht adaptierbar
3. er besitzt eine kurze Reflexzeit
4. bei Auslsung eines Eigenreflexes in einem Muskel wird der Antagonist gehemmt

A. 1, 2 und 3 sind richtig


B. 1 und 3 sind richtig
C. 2 und 4 sind richtig
D. nur 4 ist richtig
E. alle sind richtig
Richtige Antwort: E

Frage 56
Das Spinocerebellum:

1. erhlt Afferenzen vom Cerebrum.


2. projeziert auf die Formatio Reticularis.
3. enthlt Information ber das Aktivittsniveau im Thalamus.
4. enthlt Informationen ber das Aktivittsniveau der Interneurone im Rckenmark.

A. 1, 2 und 3 sind richtig


B. 1 und 3 sind richtig
C. 2 und 4 sind richtig
D. nur 4 ist richtig
E. alle sind richtig

Richtige Antwort: C

Frage 57
Welche Aussage(n) zum Cerebellum ist (sind) richtig?

1. Das Palocerebellum ist fr die Aufrechterhaltung des Gleichgewichtes verantwortlich und entspricht
dem Lotus ant. des Corpus cerebelli.
2. Die Fibrae arcuatae externae ventrales sind eine indirekte Verbindung des Grohirns mit dem Kleinhirn
und verlaufen ber den Pedunculus cerebellaris inf.
3. Der Tractus spinocerebellaris post. verluft ber den Pedunculus cerebellaris inf.
4. Es ist ein Abkmmling der Grundplatte und gehrt daher zum System der
Extrapyramidalmotorik.

A. 1. und 2. sind richtig


B. 2. und 3. sind richtig
C. 3. und 4. sind richtig
D. 1. und 4. sind richtig
E. alle sind richtig

Richtige Antwort: B

Frage 58
Fr die Horner'sche Trias trifft zu:

A. Sie besteht aus hngendem Oberlid, enger Pupille und tiefliegendem Augapfel
B. Sie begrndet sich In einer Schdigung des N. vagus
C. Sie ist ein typisches Zeichen fr eine Schilddrsenberfunktion
D. Sie wird unter anderem durch eine Lhmung des M. sphincter pupillae hervorgerufen
E. Sie entsteht hufig bei einem Hypophysentumor

Richtige Antwort: A

Frage 59
Das Corpus ciliare hat folgende Aufgaben:

1. Regulation des Lichteinfalls auf die Retina


2. Akkomodation
3. Produktion der Trnenflssigkeit
4. Aufhngung der Linse

A. 1., 2. und 3. sind richtig


B. 1. und 3. sind richtig
C. 2. und 4. sind richtig
D. nur 4. ist richtig
E. alle sind richtig

Richtige Antwort: C

Frage 60
Aktivierung des Parasympathikus bewirkt nicht:

A. Steigerung der Dnndarmmotilitt


B. Steigerung der Trnendrsensekretion
C. Erschlaffung der Bronchialmuskulatur
D. Steigerung der Speicheldrsensekretion
E. Kontraktion des M.sphincter pupillae

Richtige Antwort: C
Prfung H

Frage 1
Was trifft fr die wahrgenommene Tonhhe bei steigender Lautstrke zu?

A. Hohe Frequenzen (>3kHz) werden hher wahrgenommen.


B. Hohe Frequenzen (>3kHz) werden niedriger wahrgenommen.
C. Niedrige Frequenzen (
D. Niedrige Frequenzen (
E. Mittlere Frequenzen (ca. 2kHz) werden hher wahrgenommen.

Richtige Antwort: A

Frage 2
Welche Aussage ber die ueren Augenmuskeln trifft zu?

A. Der M. rectus lateralis adduziert den Bulbus oculi.


B. Der M. rectus medialis wird vom N. abducens innerviert.
C. Der M. rectus inferior senkt den Bulbus oculi.
D. Die Hauptaufgabe der Mm. recti bulbi ist die Rotation des Bulbus oculi.
E. Der 4. Hirnnerv innerviert die meisten ueren Augenmuskeln.

Richtige Antwort: C

Frage 3
Emotion wird bewut wahrgenommen durch die Verbindung der Umbischen Strukturen mit

1. dem Temporallappen
2. dem Okzipitallappen
3. dem Prefrontalen Cortex
4. dem Cerebellum

A. 1., 2. und 3. sind richtig


B. 1. und 3. sind richtig
C. 2. und 4. sind richtig
D. nur 4. ist richtig
E. alle sind richtig

Richtige Antwort: B

Frage 4
Welche Aussage(n) zum vegetativen Nervensystem ist (sind) richtig?

1. Vom Ncl. salivatorius sup. verlaufen parasympathische Fasern mit dem N. facialis zu den GH.
submandibulahs, subungualis und lacrimalis.
2. Das Ganglion submandibulare ist ein parasympathisches Kopfganglion, an dem die Nerven fr die
Glandulae submandibularis und subungualis umschalten.
3. Das Ganglion oticum ist ein parasympathisches Ganglion, an dem Fasern aus dem N. glossopharyngeus
fr die Glandula parotidea umschalten.
4. Der Truncus sympathicus liegt beidseits an der Wirbelsule und reicht von der
Schdelbasis bis zum Steibein.

A. 1. und 3. sind richtig


B. 1. und 4. sind richtig
C. 2. und 4. sind richtig
D. 2., 3. und 4. sind richtig
E. alle sind richtig

Richtige Antwort: E
Frage 5
Welche Aussage(n) zur Entwicklung des Nervensystems ist (sind) richtig?

1. Die Scheitelbeuge ist eine Abfaltung des Neuralrohres im Bereich des Mesencephalons.
2. Aus dem Telencephalon medium entstehen das Corpus callosum und Teile der Vorderwand des
Ventriculus tertius.
3. Die Mantelzone des Rckenmarks differenziert sich zur Flgelplatte mit den Efferenzen und zur
Grundplatte mit den Afferenzen.
4. Der sekundre Hemisphrenstiel besteht aus der Himsubstanz um das Foramen interventrtculare und
der Verschmelzung des Di- und Telencephalon.

A. 1., 2. und 3. sind richtig


B. 1., 2. und 4. sind richtig
C. 2., 3. und 4. sind richtig
D. 2. und 4. sind richtig
E. alle sind richtig

Richtige Antwort: B

Frage 6
Welche Aussage(n) zum Kleinhirn ist (sind) falsch?

1. Der Tractus arcuatocerebellaris ist eine Afferenz des Kleinhirns aus der Grohirnrinde und luft ber den
oberen Kleinhlmstiel.
2. Der Tractus vestibulofloccularis, Afferenz des Kleinhirns aus dem Gleichgewichtsorgan, schaltet im Ncl.
vestibularis um.
3. Der Tractus spinocerebellaris ant. leitet die unbewusste Tiefensensibilitt aus der oberen Extremitt und
luft ber das untere Kleinhirnsegel,
4. Der Tractus cuneocerebellaris leitet die unbewusste Tiefensensibilitt der unteren Extremitt und luft
ber den oberen Kleinhirnstiel.

A. 1., 2. und 3. sind falsch


B. 1. und 3. sind falsch
C. 2. und 4, sind falsch
D. nur 4. ist falsch
E. alle sind falsch

Richtige Antwort: E

Frage 7
Welche Behauptung trifft nicht zu?

Der N. peroneus (fibulans) communis:

A. luft an der Innenseite der Sehne des M. biceps femoris in Richtung Fibula.
B. versorgt die Flexoren am Unterschenkel.
C. teilt sich in den N. peroneus (fibularis) superficialis und profundus.
D. versorgt die Extensoren am Unterschenkel und die Mm. peronei.
E. betritt die Peroneusloge unter dem Caput fibulae.

Richtige Antwort: B

Frage 8
Welche Aussage(n) zur Capsula interna ist (sind) richtig?

1. Die Capsula interna liegt lateral des Subthalamus.


2. Ansa peduncularis und Commissura ant. sind infralentikulre Bahnen der Capsula int. und verlaufen
durch den GraUotet'schen Kanal.
3. Pedunculus thalami sup. und Tractus frontopontinus verlaufen im Crus ant. der
Capsula int.
4. Die Capsula int. liegt zwischen Crus cerebr) und Corona radiata.

A. 1., 3. und 4. sind richtig


B. 1. und 4. sind richtig
C. nur 4. ist richtig
D. nur 1. ist richtig
E. 2. und 3. sind richtig

Richtige Antwort: B

Frage 9
Richtung eines Nystagmus, welche Antworten sind richtig?

1. Drehtrommel, Rotation nach links bewirkt linksschlgigen Nystagmus


2. Drehstuhl, Rotation nach rechts bewirkt perrotatorisch einen rechtsschlagigen
Nystagmus
3. kalorische Reizung, Warmsplung links bewirkt einen rechtsschlgigen Nystagmus
4. Drehstuhl, Rotation nach rechts bewirkt postrotatorisch einen linksschlgigen
Nystagmus

A. 1., 2. und 3. sind richtig


B. 1. und 3. sind richtig
C. 2. und 4. sind richtig
D. nur 4. ist richtig
E. alle sind richtig

Richtige Antwort: C

Frage 10
Fr die Membranleitfhigkeiten einer lebenden Zelle am Ruhepotential treffen folgende Aussagen zu:

1. die Kalium - Leitfhigkeit ist sehr hoch


2. die Chlorid - Leitfhigkeit ist hoch
3. die Natrium - Leitfhigkeit ist gering
4. die Leitfhigkeit fr die Eiwei - Anionen ist hoch

A. 1., 2. und 3. sind richtig


B. 1. und 3. sind richtig
C. 2. und 4. sind richtig
D. nur 4. ist richtig
E. alle sind richtig

Richtige Antwort: A

Frage 11
Welche Aussage(n) zur folgenden Abbildung ist (sind) richtig?

1. Die Ziffer 2 zeigt das Trigonum lemnisci, das zum Tegmentum mesencephali gehrt.
2. Die Ziffer 4 zeigt den Pedunculus cerebellaris medius, der das Kleinhirn mit dem Pons verbindet.
3. Die Ziffer 3 zeigt das Corpus geniculatum laterale, an dem die Radiatio optica beginnt
4. Die Ziffer 1 zeigt die Fortsetzung der Substantia gelatinosa Rolandi des Rckenmarks (Tuberculum
cinereum).
A. 1., 2. und 4. sind richtig
B. 1. und 4. sind richtig
C. 1., 2. und 3. sind richtig
D. 1. und 3. sind richtig
E. 2. und 3. sind richtig

Richtige Antwort: B

Frage 12
Eine 43-jhrige Patientin zeigt stark juckende Hautvernderungen an beiden Handrcken. Die Anamnese
ergibt, dass sie vor einigen Tagen eine neue Handcreme gekauft und angewandt hat.
Welche Struktur ist fr die Hautkrankheit in erster Linie pathogenetisch relevant?

A. Korneozyt (Homzelle)
B. Stratum germinativum
C. Desmosomen
D. Langerhanszelle
E. Melanozyt

Richtige Antwort: D

Frage 13
Welche Aussage(n) zur folgenden Abbildung ist (sind) richtig?

1. Die Ziffer 3 zeigt das Velum medullre Inf., das im bulbaren Bereich des IV. Ventrikels das Dach bildet.
2. Die Ziffer 1 zeigt den Gyrus cinguli, der gemeinsam mit der Area adolfactoria, dem Isthmus gyri Cinguli
und dem Gyrus parahippocampalis den Gyrus fornicatus bildet.
3. Die Ziffer 2 zeigt die Commissura ant., deren Pars ant. die beiden Corpora
amygdaloidea und die beiden Schlfenpole mit einander verbindet.
4. Die Ziffer 4 zeigt das Indusium griseum, das gemeinsam mit dem Gyrus
paraterminalis, dem Gyrus fasciolaris und dem Gyrus dentatus den ueren Randbogen bildet.
A. 1. und 2. sind richtig
B. 1. und 3. sind richtig
C. 3. und 4. sind richtig
D. 2. und 4. sind richtig
E. 2., 3. und 4. sind richtig

Richtige Antwort: D

Frage 14
Was trifft auf die Trunci des Plexus brachialis NICHT zu:

A. Der Truncus superior entsteht aus den Segmenten C5 und C6.


B. Der Truncus inferior entsteht aus den Segmenten C8 und Th1.
C. C7 bildet allein den Truncus medius.
D. Die Trunci liegen in der vorderen Skalenuslcke.
E. Aus den Trunci gehen drei Faszikeln hervor.

Richtige Antwort: D

Frage 15
Die Bruch-Membran findet sich zwischen

A. Pigmentepithel und Photorezeptorzellen


B. Lamina suprachoroidea und Sklera
C. Endothel und Stroma der Kornea
D. Nervenfaserschicht und Glaskrper
E. Lamina choroidocapillaris und Pigmentepithel

Richtige Antwort: E

Frage 16
Welche Aussage(n) zur Sensibilitt ist (sind) richtig?

1. Im Mesencephalon liegt die Fortsetzung des Fasciculus gracilis hinter


der des Fasciculus cuneatus
2. Der Lemniscus medialis entsteht aus der Gesamtheit der zweiten Neurone der bewussten Sensibilitt
3. Der Tractus spinothlamicus luft ab dem Pons im Lemniscus medialis.
4. Das Strtum interolivare lemnisci stellt den Beginn des Lemniscus medialis dar. Hier kreuzt die Fasern
des Ncl. gracilis hinten und die des Ncl. cuneatus vorne.

A. 1.. 2. und 3. sind richtig


B. 1. und 2. sind richtig
C. 2. und 3. sind richtig
D. 3. und 4. sind richtig
E. nur 4. ist richtig

Richtige Antwort: B

Frage 17
Der Plexus brachialis besteht aus Rr. ventrales aus den Segmenten:

A. C3 - C8
B. C1 - C4
C. C4 - C8
D. C5 - Th1
E. C6 - Th2

Richtige Antwort: D

Frage 18
Ein mehrschichtiges unverhorntes Plattenepithel findet sich:

1. in der Conjunctiva bulbi


2. an der Vorderflche der Cornea
3. an der Vorderflche der Linse
4. am quator der Linse
5. an der Papilla nervi optici
6. an den Zonula-Fasern

A. nur 2. und 6. sind richtig


B. nur 1. und 2. sind richtig
C. nur 1., 4., 5, und 6. sind richtig
D. nur 2., 4., 5. und 6. sind richtig
E. alle sind nichtig

Richtige Antwort: B

Frage 19
Gehr - wodurch knnen otoakustische Emissionen entstehen?

1. infolge eines Totalausfalls des kochlearen Verstrkermechanismus


Z. durch Mikrobewegungen der inneren Haarzellen
3. durch Kontraktion der Mittelohrmuskeln
4. durch spontane und stimulationsinduzierte Bewegungen an den ueren Haarzellen

A. 1., 2. und 3. sind richtig


B. 1. und 3. sind richtig
C. 4. und 2. sind richtig
D. nur 4. ist richtig
E. alle sind richtig

Richtige Antwort: D

Frage 20
Das Cerebellum:
1. ist bei kognitiven Leistungen eingebunden
2. erhlt Afferenzen vom Grohirn
3. ist fr das Timing von Bewegungen verantwortlich
4. erhlt Afferenzen vom Rckenmark

A. 1., 2. und 3. sind richtig


B. 1. und 3. sind richtig
C. 2. und 4. sind richtig
D. nur 4. ist richtig
E. alle sind richtig

Richtige Antwort: E

Frage 21
Die Abbildung zeugt einen Ausschnitt aus dem vorderen Abschnitt des Bulbus oculi.
Verschiedene Strukturen sind mit Buchstaben markiert.
Welche der folgenden Aussagen dazu trifft zu?

A. Struktur A ist typischerwerse frei von Blutgefen.


B. ber Struktur B kann die Trnenflssigkeit aus dem Kammerwinkel abflieen.
C. Struktur C ist an ihrer Vorderflche von der Conjuntiva bulbi bedeckt.
D. Struktur D enthlt den M. sphincter und den M. dilatator pupillae.
E. Struktur E grenzt mit Ihrer Hinterflche an die hintere Augenkammer

Richtige Antwort: E

Frage 22
Weiche Aussage(n) zum Mesencephalon ist (sind) richtig?

1. Der Fasciculus longitudinalis med. ist fr die Gleichgewichtshaltung des Kopfes und aufrechte
Netzhautbilder verantwortlich und kreuzt in der Decussatio tegmenti ventralis
2. Der Ncl. mesencephalicus n. V liegt lateral des Ncl. n. III; hier erfolgt die Umschaltung von Fasern aus
der Kaumuskulatur
3. Das Trigonum lemnisci wird nach ventral durch den Sulcus lat. cruris cerebri und nach dorsal/unten
durch das Brachium colliculi inf. begrenzt
4. Der Tractus tectospinalis ist eine optische und akustische Reflexbahn und kreuzt in der Decussatio
tegmenti dorsalis

A. 1. und 2. sind richtig


B. nur 2. ist richtig
C. 1. und 4. sind richtig
D. nur 4. ist richtig
E. nur 3. ist richtig
Richtige Antwort: D

Frage 23
Welche der folgend aufgelisteten Zellen liegen nicht in der Epidermis?

A. amakrine Zellen
B. Merkel-Zellen
C. Melanozyten
D. Keratinozyten
E. Langerhans-Zellen

Richtige Antwort: A

Frage 24
Weiche Aussage(n) zum Diencephalon ist (sind) richtig?

1. Das Corpus geniculatum mediale gehrt zum Metathalamus; hier beginnt die Radiatio acustica, die an
den Gyri temporales transversi endet.
2. Der Subthalamus befindet sich zwischen Capsula interna und Hypothalamus.
3. Die Facies medialis thalami reicht vom Sulcus hypothalamicus bis zur Stria medullaris thalami.
4. Die Facies dorsalis des Thalamus Ist Im medialen Teil von der Lamina affixa bedeckt und schaut in die
Celia media, im lateralen Teil ist sie von Pia mater bedeckt und schaut in die Fissura telodiencephalica.

A. 1., 2. und 3. sind richtig


B. 2., 3. und 4. sind richtig
C. 1., 3. und 4. sind richtig
D. 2. und 4. sind richtig
E. alle sind richtig

Richtige Antwort: A

Frage 25
Welche Aussag(n) zur folgenden Abbildung ist (sind) richtig?

1. Die Ziffer 4 zeigt den Limbus Giacomlni. der das Ende des Gyrus dentatus bildet und sich mit der
Fimbria fornicis verbindet.
2. Die Ziffer 2 zeigt das Splenium corporis callosi.
3. Die Ziffer 3 zeigt den Sulcus calcarinus, der im Cornu post, des Ventriculus lateralis den Calcar avis
aufwirft.
4. Die Ziffer 1 zeigt den Gyrus parahippocampalis, der gemeinsam mit der Area
adolfactorta, dem Gyrus cinguli und dem Isthmus gyri cinguli den Gyrus fornicatus bildet.
A. 1., 2. und 3. sind richtig
B. 1. und 3. sind richtig
C. 2. und 3. sind richtig
D. nur 4. ist richtig
E. alle sind richtig

Richtige Antwort: E

Frage 26
Ein Bndel von Nervenfasern im PNS wird umgeben von

A. Perineurium
B. Mantelzellen
C. Myelinscheiden
D. Neurofibrillen
E. Schwann-Zellen

Richtige Antwort: A

Frage 27
Fr die Horner'sche Trias trifft zu:

A. Sie besteht aus hngendem Oberlid, enger Pupille und tiefliegendem Augapfel
B. Sie begrndet sich in einer Schdigung des N. vagus
C. Sie ist ein typisches Zeichen fr eine Schilddrsenberfunktion
D. Sie wird unter anderem durch eine Lhmung des M. sphincter pupillae hervorgerufen
E. Sie entsteht hufig bei einem Hypophysentumor

Richtige Antwort: A

Frage 28
Fr das Sehen in dunkler Umgebung kann man sich durch ca. 30 minubges Tragen einer roten Schutzbrille
(die man dann in der Dunkelheit abnimmt) vorbereiten weil

1. Die Dunkeladaptation der Stbchen und Zapfen nach 30 Minuten abgeschlossen ist.
2. Stbchen fr Licht am roten Ende des Spektrums praktisch unempfindlich sind.
3. In der dunkel adaptierten Retina die Stbchen fr grnes Licht empfindlicher sind.
4. Stbchen fr rotes Licht die hchste Sensitivitt besitzen.

A. 1., 2. und 3. sind richtig


B. 1. und 3. sind richtig
C. 2. und 4. sind richtig
D. nur 4. ist richtig
E. alle sind richtig

Richtige Antwort: A

Frage 29
Welche Aussage(n) zum Cerebellum ist (sind) richtig?

1. Die Fibrae arcuatae extemae ventrales sind eine indirekte Verbindung des Grohirns mit dem Kleinhirn
und verlaufen ber den Pcdunculus cerebellaris sup.
2. Das Archicerebellum ist fr die Aufrechterhaltung des Gleichgewichtes verantwortlich und entspricht dem
Lobus flocculonodularis.
3. Es ist ein Abkmmling der Grundplatte und gehrt daher zum System der
Extrapyramidalmotorik.
4. Der Tractus spinocerebellaris post. verluft Uber den Pedunculus cerebellaris sup.

A. 1. und 2. sind richtig


B. nur 2. ist richtig
C. 2. und 3. sind richtig
D. nur 3. ist richtig
E. 3. und 4. sind richtig

Richtige Antwort: B

Frage 30
Der Plexus sacralis liegt:

A. im Canalis sacralis
B. lateral an der Innenseite des Os ilium
C. an der ventralen Seite des Os sacrum
D. lateral an der Innenseite der Sitzbeinhcker
E. an der dorsalen Seite des Os sacrum

Richtige Antwort: C

Frage 31
Welche der folgenden Strukturen gehrt nicht zu einer markhaltigen Nervenfaser des PNS?

A. Ranvier-Schnrring
B. Perineuralscheide
C. Neurit
D. Internodium
E. Schmidt-Lanterman-Einkerbung

Richtige Antwort: B

Frage 32
Folgende Aussage ber den Augapfel trifft zu:

A. Der Augapfel hat eine zweischichtige Wand.


B. Im Inneren des Augapfels befinden sich Trnenflssigkeit, Linse und Glaskrper
C. Die Tunica vasculosa bulbi wird auch Uvea genannt
D. Der Sehnerv verlsst den Augapfel am gelben Fleck.
E. Am Discus nervi optici befindet sich die Stelle des schrfsten Sehens.

Richtige Antwort: C

Frage 33
Ependymzellen bilden

A. die Auskleidung von Liquorrumen


B. Markscheiden
C. den Plexus choroideus
D. die Pia mater der Leptomeninx
E. die Blut-Hirn-Schranke

Richtige Antwort: A

Frage 34
Biaurales Hren, fr die Ortung einer Schallquelle im Raum spielen eine Rolle:

1. die Form der Ohrmuschel


2. Laufzeitunterschiede
3. Intensittsunterschiede
4. reflektierte otoakustische Emissionen
A. 1., 2. und 3. sind richtig
B. 1. und 3. sind richtig
C. 2. und 4. sind richtig
D. nur 4. ist richtig
E. alle sind richtig

Richtige Antwort: A

Frage 35
Welche der aufgelisteten Zellen ist keine Nervenzelle?

A. Sternzelle
B. Mller-Zelle
C. Purkinje-Zelle
D. Korbzelle
E. Pyramidenzelle

Richtige Antwort: B

Frage 36
Gyrus cinguli ist:

1. eine Limbische Struktur


2. fr autonome Reaktionen relevant
3. fr den Muskeltonus entscheidend
4. bei rhythmischen Bewegungen aktiv

A. 1., 2. und 3. sind richtig


B. 1. und 3. sind richtig
C. 2. und 4. sind richtig
D. nur 4. ist richtig
E. alle sind richtig

Richtige Antwort: E

Frage 37
Welche Aussage(n) zur Sensibilitt ist (sind) richtig?

1. Die Fasciculi gracilis und cuneatus sind die Nebenbahnen fr Schmerz und Temperatur.
2. Der Tractus spinothalamicus ant. ist die Nebenbahn fr Druck und Berhrung, der Tractus
spinothalamicus lat. ist die Hauptbahn fr Schmerz und Temperatur.
3. Der Tractus nucleothalamicus ist die Fortsetzung der Fasciculi gracilis und cuneatus und beginnt am Ncl.
gracilis bzw. cuneatus.
4. Der Fasciculus cuneatus leitet die bewusste Sensibilitt der oberen Krperhlfte, der oberen Extremitt,
des Halses und des Kopfes.

A. 1. und 2. sind richtig


B. 2. und 3. sind richtig
C. 1. und 4. sind richtig
D. 2. und 4. sind richtig
E. alle sind richtig

Richtige Antwort: A

Frage 38
Welche Aussage(n) zur Extrapyramidalmotorik ist (sind) richtig?

1. Die Ansa lenticularis verluft durch die Forel'schen felder H1 und H2 und gibt Fasern zur Formatio
reticularis.
2. Der Fasciculus lenticularis fhrt pallidofugale Fasern und besteht aus dem Fasciculus thalamicus und
der Ansa lenticularis.
3. Die Radiatio thalami ist eine reziproke Verbindung zwischen Thalamus und Pallidum.
4. Das Striatum erhlt Afferenzen aus dem Cerebellum und dem Thalamus und steht mit dem Pallidum in
Verbindung.

A. 1., 2. und 3. sind richtig


B. 1. und 3. sind richtig
C. 2. und 4. sind richtig
D. nur 4. ist richtig
E. alle sind richtig

Richtige Antwort: D

Frage 39
Welche Aussage(n) zur folgenden Abbildung ist (sind) richtig?

1. Die Ziffer 2 zeigt die Cella media ventriculi lateralis, deren mediale Wand von Fornix und Tela choroidea
gebildet wird.
2. Die Ziffer 1 zeigt den Tractus opticus, der am Corpus geniculatum laterale endet.
3. Die Ziffer 3 zeigt das Cornu inf. ventriculi lateralis, dessen mediale Wand von Pes hippocampi und Tela
choroidea gebildet werden.
4. Die Ziffer 4 zeigt den Gyrus parahippocampalis, der Teil des Gyrus fornicatus ist.

A. 1. und 2. sind richtig


B. nur 2. ist richtig
C. 3. und 4. ist richtig
D. nur 3. ist richtig
E. nur 4. ist richtig

Richtige Antwort: E

Frage 40
Welche der folgenden Aussagen ber Keratinozyten ist/sind zutreffend?

1. Sie stehen durch Desmosomen miteinander tn Verbindung.


2. Sie besitzen zahlreiche dendritische Fortstze.
3. Sie enthalten Zytokeratinfilamente im Zytoplasma.
4. Sie synthetisieren Melanin in den Melanosomen.

A. nur 2. ist richtig


B. nur 2. und 4. sind richtig
C. alle sind richtig
D. nur 1. und 3. sind richtig
E. nur 1., 2. und 3. sind richtig

Richtige Antwort: D

Frage 41
Wie kann die bertragungsfunktion des Innenohres beschrieben werden?

A. Als Verhltnis der Auslenkung der asilarmembran zur Auslenkung des Steigbgels im ovalen Fenster
und somit vom Messpunkt an der Basilarmembran abhngig.
B. Als Verhltnis der Auslenkung der Basilarmembran zur Auslenkung des Steigbgels im ovalen Fenster
und nicht vom Messpunkt an der Basilarmembran abhngig.
C. Als Verhltnis der Schall impedanz der Basilarmembran zur Schall impedanz des Steigbgels Im ovalen
Fenster und nicht vom Messpunkt an der Basilarmembran abhngig.
D. Als Verhltnis der Schallimpedanz der Basilarmembran zur Schallimpedanz des Steigbgels im ovalen
Fenster und somit vom Messpunkt an der Basilarmembran abhngig.
E. Als Verhltnis der Schallimpedanz der Basilarmembran zur Schallimpedanz des Steigbgels im ovalen
Fenster und somit frequenzabhngig.

Richtige Antwort: A

Frage 42
Motorik - die Afferenzen der Muskelspindeln signalisieren:

1. die Muskeldehnung und Dehnungsgeschwindigkeit


2. die Kraft die am Muskel wirkt
3. die Strke der gamma motorischen Innervation
4. die Temperatur der Arbeitsmuskulatur

A. 1., 2. und 3. sind richtig


B. 1. und 3. sind richtig
C. 2. und 4. sind richtig
D. nur 4. ist richtig
E. alle sind richtig

Richtige Antwort: B

Frage 43
Nervenleitgeschwindigkeit (NLG) - welche Antworten sind richtig?

1. sie gibt an, wie schnell elektrische Impulse entlang einer Nervenfaser bertragen werden
2. die NLG hngt von Adaptationsverhalten des jeweiligen Mechanorezeptors ab
3. dicke Axone bertragen mit hheren Geschwindigkeiten als dnne
4. die NLG hngt von der Aktivitt In den nebenliegenden Fasern ab

A. 1., 2. und 3. sind richtig


B. 1. und 3. sind richtig
C. 2. und 4. sind richtig
D. nur 4. ist richtig
E. alle sind richtig

Richtige Antwort: B
Frage 44
Welche Aussage(n) zur folgenden Abbildung ist (sind) richtig?

1. Die Ziffer 3 zeigt den N. trochlearis, der motorische und parasympathische Fasern fhrt.
2. Die Ziffer 1 zeigt die Pyramiden, in denen die Tractus corticospinales nach unten verlaufen.
3. Die Ziffer 4 zeigt die Vagusgruppe", die im Sulcus retroolivaris aus dem Hirnstamm austritt.
4. Die Ziffer 2 zeigt den Pedunculus cerebellarls sup., in welchem Faserverbindungen zwischen Cerebellum
und Mesencephalon verlaufen.

A. 1., 2. und 3. sind richtig


B. 1. und 3. sind richtig
C. 2. und 4. sind richtig
D. nur 4. ist richtig
E. alle sind richtig

Richtige Antwort: D

Frage 45
Wer wird vom N. gluteus superior nicht versorgt?

1. M. gluteus medius
2. M. tensor fasciae latae
3. M. glutaeus minimus
4. M. gluteus maximus

A. 1., 2. und 3. sind richtig


B. 1. und 3. sind richtig
C. 2. und 4. sind richtig
D. nur 4. ist richtig
E. alle sind richtig

Richtige Antwort: D

Frage 46
Welche Aussage (n) zu den Pyramidenbahnen ist (sind) richtig?

1. Der Tractus corticospinalis lauft in der Pars ventralis pontis und bildet hier die Fibrae longitudinales
pontis.
2. Der Tractus corticonudearis des N. VII verluft zum ipsi- und kontralateralen rostralen Kern.
3. Ein Verschluss der A. cerebri ant. fhrt zu einer beinbetonten kontralateralen
Hemiparese und Hemiansthesie, sowie zu einer zentralen Blaseninkontinenz.
4. Der Tractus corticonuclearis fr den N. hypoglossus verlauft durch das Centrum semiovale, die Capsula
Interna und das Crus cerebri, kreuzt zur Gegenseite und endet am Ncl. n. hypoglossi.

A. 1., 2. und 3. sind richtig


B. 2., 3. und 4. sind richtig
C. 3. und 4. sind richtig
D. 1., 3. und 4. sind richtig
E. 1. und 4. sind richtig

Richtige Antwort: D

Frage 47
Weiche Aussage(n) zu den Ventriculi laterales ist (sind) richtig?

1. Der Boden der Cella media wird vom Ncl. caudatus gebildet.
2. Das Cornu post. wird nach unten durch das Trigonum collaterale, durch welches Fasern der
Sehstrahlung verlaufen, begrenzt.
3. Im Dach des Comu inf. liegen die Stria terminalis (lateral) und die Cauda nuclei caudati (medial).
4. Der Bulbus cornu post. wird durch den Forceps post. aufgeworfen und bildet die mediale Wand des
Comu post.

A. 1., 2. und 3. sind richtig


B. 1. und 3. sind richtig
C. 2. und 4. sind richtig
D. nur 4. ist richtig
E. alle sind richtig

Richtige Antwort: C

Frage 48
Welche der folgenden Aussage(n) Ist (sind) richtig?

1. Die Tractus dentatothalamicus und cerebellorubralis gehren zum Tractus teqmentalis centralis.
2. Die Efferenzen des Kleinhirns treten ber den Pedunculus cerebellaris medius aus.
3. Der Ncl. motorius tegmenti besteht aus Kernen der Formatio reticularis.
4. Die Tractus rubroolivaris und reticuloolivaris bilden zusammen die Stilling'sche Schere.

A. 1., 2. und 3. sind richtig


B. 2. und 3. sind richtig
C. 1. und 4. sind richtig
D. nur 3. ist richtig
E. nur 4. ist richtig

Richtige Antwort: D

Frage 49
Welche der folgenden Aussagen zur Augenentwicklung ist zutreffend?

A. Aus der inneren Schicht des Augenbechers entsteht das Pigmentepithel.


B. Die Linsenplaktode induziert die Entwicklung des Augenblschens.
C. Das Augenblschen entsteht aus dem Augenbecher.
D. Der Augenbecher ist ber den Augenbecherstiel mit dem Oberflchenektoderm verbunden.
E. In der Augenbecherspalte ist die A. hyaloidea gelegen.

Richtige Antwort: E

Frage 50
Welche Aussage(n) zu den Fasersystemen ist (sind) richtig?
1. Im Crus post. der Capsula int. verlaufen unter anderem die Pedunculi thalami sup. post. und der Tractus
temporooccipitopontinus.
2. Der Fasciculus uncinatus besteht aus Assoziationsfasem und windet sich um das hintere Ende des
Sulcus lateralis.
3. Die Ansa peduncularis besteht aus dem Pedunculus thalami post. und der Ansa lenticularis.
4. Die Commissura ant. vertuft durch den Gratiolet'schen Kanal.

A. 1. und 2. sind richtig


B. 2. und 3. sind richtig
C. 1. und 3. sind richtig
D. 3. und 4. sind richtig
E. 1. und 4. sind richtig

Richtige Antwort: E

Frage 51
Welche Aussage(n) zu den Hirnnervenkernen ist (sind) richtig?

1. Der Nd. ambiguus ist ein viszeromotonscher Kern und liegt im metencephalen Teil der Fossa
rhomboidea.
2. Der Ncl. spinalis n. V besteht aus bipolaren Nervenzellen, ist also ein in die Tiefe gelagertes Ganglion.
3. Der N. facialis bildet um den Ncl. nervt III ein inneres Fazialisknie".
4. Der Ncl. principalis n. V ist der Endkern fr Druck- und Berhrungsempfindungen aus dem Gesichts- und
Zungenbereich.

A. 1., 2. und 3. sind richtig


B. 1. und 3. sind richtig
C. 2. und 4. sind richtig
D. nur 4. ist richtig
E. alle sind richtig

Richtige Antwort: D

Frage 52
Welche Aussage(n) zum Rckenmark ist (sind) richtig?

1. Der Sulcus intermedius posterior markiert die Eintrittsstelle der Radix dorsalis.
2. Unterhalb des 2. Sakralwirbels liegen die Spinalganglien im Canalis sacralis.
3. Der Fasciculus gracilis leitet die Sensibilitt der unteren Krperhlfte und beginnt daher im Sacralmark.
4. Der Funiculus lateralis wird durch das Cornu post, von den Funiculi postt. und durch den Sulcus
ventrolateral is vom Funiculus ant. getrennt.

A. 1., 2. und 3. sind richtig


B. 2. und 3. sind richtig
C. 2., 3. und 4. sind richtig
D. 3. und 4. sind richtig
E. alle sind richtig

Richtige Antwort: C

Frage 53
Welche bertragersubstanz und welchen Rezeptortyp findet man an der markierten Stellen (Pfeil B) des
Nervus Parasympathicus? Das Zielorgan sei eine Muskelzelle des M. sphincter pupillae!

1) Die bertragersubstanz an der markierten Stelle B ist Acetycholin


2) Die bertrgersubstanz an der markierten Stelle B Ist Noradrenalin
3) Die bertragersubstanz an der markierten Stelle B ist Glycin
4) Die Rezeptoren an der markierten Stelle B sind muskarinerge Rezeptoren
5) Die Rezeptoren an der markierten Stelle B sind nicotinerge Rezeptoren
6) Die Rezeptoren an der markierten Stelle B sind adrenerge alpha-Rezeptoren
7) Die Rezeptoren an der markierten Stelle B sind adrenerge beta-l-Rezeptoren
8) Die Rezeptoren an der markierten Stelle B sind glycinerge Renshaw-Rezeptoren

A. 1 und 5 sind richtig


B. 3 und 8 sind richtig
C. 2 und 7 sind richtig
D. 2 und 6 sind richtig
E. 1 und 4 sind richtig

Richtige Antwort: E

Frage 54
Welche Aussage(n) zur folgenden Abbildung ist (sind) richtig?

1. Die Ziffer 3 zeigt den Ncl. motorius n. v, einen viszeromotorischen Kern fr die
Innervation der Kaumuskulatur.
2. Die Ziffer 4 zeigt den Ncl. n. XI, einen viszeromotorischen Hirnnervenkern.
3. Die Ziffer 1 zagt den Ncl. solitarius. den Endkern fr Geschmacksfasern aus den Nn. VII, IX und X.
4. Die Ziffer 2 zeigt den Ncl. dorsalis n. X, einen parasympathischen Hirnnervenkern.

A. 1., 2. und 3. sind richtig


B. 2., 3. und 4. sind richtig
C. 1., 3. und 4. sind richtig
D. nur 2. ist richtig
E. alle sind richtig
Richtige Antwort: E

Frage 55
Unter Akkommodationsbreite versteht man:

1. Den Bereich des Raumes, in dem man scharf sehen kann


2. Die maximale Brechkraftnderung zu der das Auge fhig ist
3. Die Differenz zwischen Fernpunkt und Nahpunkt in Dioptrien ausgedrckt
4. Den Kehrwert der kleinsten Brennweite minus dem Kehrwert der grten Brennweite des Auges

A. 1., 2. und 3. sind richtig


B. 1. und 3. sind richtig
C. 2. und 4. sind richtig
D. nur 4. ist richtig
E. alle sind richtig

Richtige Antwort: C

Frage 56
Elektroenzephalogramm (EEG) - welche Aussagen sind richtig?

1. Hauptverursachend sind Feldpotentiale infolge synaptischer Prozesse (kortikale Neurone)


2. Das EEG spiegelt das Aktivierungsniveau der Hirnrinde wieder
3. EEG-Wellen entstehen durch synchronisierte zellulre Aktivitt
4. Im aktiven Wachzustand, bei geffneten Augen, dominieren Alpha-Wellen (8-13 Hz)

A. 1., 2. und 3. sind richtig


B. 1. und 3. sind richtig
C. 2. und 4. sind richtig
D. nur 4. ist richtig
E. alle sind richtig

Richtige Antwort: A

Frage 57
Welche Aussage trifft nicht zu?
Das Trommelfell:

A. bertrgt die Schallwelle auf die Gehrknchelchen.


B. hat normalerweise eine perlgraue Farbe.
C. steht senkrecht zur Achse des knchernen ueren Gehrgangs.
D. trennt das Auenohr vom Mittelohr.
E. liegt etwas weniger als 4 cm medial der Ohrmuschel.

Richtige Antwort: C

Frage 58
Welche Aussage(n) zur folgenden Abbildung ist (sind) richtig?

1. Die Ziffer 2 zeigt den Ventriculus tertius.


2. Die Ziffer 1 zeigt den Forceps posterior.
3. Die Ziffer 3 zeigt das Cornu inf. des Seitenventrikels.
4. Die Ziffer 4 zeigt das Crus fornicis.
A. 1., 2. und 3. sind richtig
B. 1., 2. und 4. sind richtig
C. 2., 3. und 4. sind richtig
D. nur 2. ist richtig
E. alle sind richtig

Richtige Antwort: B

Frage 59
Weiche Struktur durchbricht der R. profundus n. radialis, um an die Streckseite des Unterarmes zu
gelangen?

A. den M. brachioradialis
B. den M. pronator teres
C. den M. extensor carpi radialis longus
D. den M. supinator
E. die Membrana interossea antebrachii

Richtige Antwort: D

Frage 60
Welche Aussage(n) zu den Taenien ist (sind) richtig?

1. Die Tela choroidea bildet das Dach des Foramen interventriculare.


2. Die Taenia thalami verluft In der Stria terminalis und geht am Foramen Monroi in die Taenia choroidea
ber.
3. Die Taenia fomicis geht am Foramen interventriculare in die Taenia fornicis der
Gegenseite ber.
4. Die Taenia choroidea verluft in der Stria medullaris und geht an der Spitze des Unterhorns in die Taenia
fornicis ber.

A. 1., 2. und 3. sind richtig


B. 1. und 3. sind richtig
C. 2. und 4. sind richtig
D. nur 4. ist richtig
E. alle sind richtig

Richtige Antwort: B
Prfung I

Frage 1
Der Nervus tibialis versorgt:

1. den M.soleus
2. den M.gastrocnemius
3. den M.semitendinosus
4. den M.satorius

A. 1, 2 und 3 sind richtig


B. 1 und 3 sind richtig
C. 2 und 4 sind richtig
D. nur 4 ist richtig
E. alle sind richtig

Richtige Antwort: A

Frage 2
Welche Aussage zum Mesencephalon ist richtig:

1. Im Pes mesencephali verlaufen (von lateral nach medial) die Tractus temporooccipitopontinus,
corticonuclearis, corticospinalis, frontopontinus.
2. Der Tractus rubrospinalis, eine extrapyramidalmotorische Bahn, kreuzt im Tegmentum mesencephali in
der Decussatio tegmenti dorsalis
3. Der Fasciculus longitudinalis medialis reicht vom Mittelhirn bis ins obere Halsmark und ist unter anderem
fr aufrechte Netzhautbilder verantwortlich
4. Der tractus bulbothalamicus schliet sich im Tegmentum mesencephali dem Lemniscus medialis an.

A. nur 1 ist richtig


B. nur 2 ist richtig
C. nur 3 ist richtig
D. 3 und 4 sind richtig
E. 2 und 4 sind richtig

Richtige Antwort: C

Frage 3
Welche Aussagen zum Ventrikelsystem sind richtig:

1. Die Taenia cerebelli verluft vom Pedunculus flocculli ber den freien Rand des
Vellum medullare sup. Zum Nodulus und denselbern Weg auf der Gegenseite zurck.
2. Der Boden der Cella media wird vom medialen Anteil der Dorsalflche des
Thalamus gebildet
3. Das Dach des Ventrikulus tertius wird von Tela choroidea, ihre Abrisslinie verluft in der Stria terminalis.
4. Die mediale Begrenzung des Cornu inf. Bilden Pes hippocampi und Tela choroidea, medial davon
befindet sich das Crus cerebri

A. 1, 2 und 3 sind richtig


B. 1 und 3 sind richtig
C. 2 und 4 sind richtig
D. nur 4 ist richtig
E. alle sind richtig

Richtige Antwort: E

Frage 4
Welche Aussagen(n) zur folgenden Abbildungen ist (sind) richtig?

1. Die Ziffer 3 zeigt den N.VII, der motorisch, sensorisch und parasympathisch ist.
2. Die Ziffer 4 zeigt den N.V, der motorisch, sensibel und parasympathisch ist.
3. Die Ziffer 2 zeigt den N.III, der rein motorisch ist.
4. Die Ziffer 1 zeigt den N.X der motorisch, parasympathisch, sensorisch und sensibel ist.

A. 1 und 4 sind richtig


B. 2 und 3 sind richtig
C. 3 und 4 sind richtig
D. 2 und 4 ist richtig
E. alle sind richtig

Richtige Antwort: A

Frage 5
In der Abbildung sind verschiedene Strukturen mit Pfeilen und Ziffern markiert.
Welche Zuordnung Ziffer-Struktur ist/sind zutreffend?

1. 1- M.arrector pilli
2. 2- innere epitheliale Wurzelscheide
3. 3- Huxley-Schicht
4. 4-Matrixzellen
5. 5-Schweidrse
A. 1, 3, 4 und 5 sind richtig
B. nur 1 ist richtig
C. 2, 3 und 4 sind richtig
D. 1 und 3 sind richtig
E. alle sind richtig

Richtige Antwort: B

Frage 6
Welche Aussage(n) zur folgenden Abbildungen ist (sind) falsch?

1. Die Ziffer 3 zeigt die Pars triangularis des Gyrus frontalis inf., Sitz des
motorischen Sprachzentrums
2. Die Ziffer 4 zeigt das motorische Reprsentationsfeld der Beine
3. Die Ziffer 1 zeigt den Gyrus angularis, Sitz des Lesezentrums.
4. Das primre Hrzentrum befindet sich in den Gyri temporalis transversi, die auf dem Gyrus temporalis
sup. liegen.

A. 1 und 4 sind falsch


B. 1 und 2 sind falsch
C. 2 und 3 sind falch
D. nur 3 ist falsch
E. alle sind falsch

Richtige Antwort: C

Frage 7
Welche Aussage (n) zur Entwicklung des ZNS ist (sind) falsch?

1. Das Augenblschen ist eine Ausstlpung des telencepahlen Blschens.


2. Die Scheitelbeuge ist eine Abfsaltung des Neuralrohres im Bereich des
mesencepahlen Blschens.
3. Aus der Ganglienleiste des Hirnstamms entstehten die sensiblen Ganglien der Nn. V, XII, IX, X.
4. Aus der Marginalzone des Rckenmarks entstehen die Flgel und Grundplatte.

A. 1, 2 und 4 sind falsch


B. 2 und 3 sind falsch
C. 1 und 4 sind falsch
D. nur 3 ist falsch
E. nur 2 ist falsch

Richtige Antwort: C

Frage 8
Welche der folgenden Strukturen ist/sind in der Abbildung vorhanden?

1. Stratum reticulare
2. Stratum lucidum
3. Stratum germinativum
4. Stratum granulosum
5. Stratum spinosum

A. nur 2, 4 und 5 sind richtig


B. alle sind richtig
C. nur 2 ist richtig
D. nur 3, 4 und 5 sind richtig
E. nur 2, 3, 4 und 5 sind richtig

Richtige Antwort: B

Frage 9
In welche Hauptstmme teilt sich der N.radialis?

A. in einen Ramus lateralis und medialis


B. in einen Ramus anterior und posterior
C. in einen Ramus superior und inferior
D. in einen Ramus profundus und superficialis
E. in einen Ramus muscularis und cutaneus

Richtige Antwort: D

Frage 10
Wichtige an der Blickmotorik beteiligte Areale des Hirnstamms sind:

1. die mesencepahle, retikulre Formation (MRF)


2. der Nucleus praepositus hypoglossi
3. der Edinger-Westphal-Kern
4. die paramediane pontine retikulre Formation (PPRF)

A. 1, 2 und 3 sind richtig


B. 1 und 3 sind richtig
C. 2 und 4 sind richtig
D. nur 4 ist richtig
E. alle sind richtig

Richtige Antwort: E

Frage 11
Fr die Chemorezeption gilt:

1. beim nasal-trigeminalen System sprechen freie Nervenendigungen auf starke


chemische Reize an
2. der Geruchssinn weist eine rschere Adaption auf als der Geschmackssinn
3. die Empfindlichkeit des Geschmackssinns nimmt im Alter ab
4. das oral-trigeminale System beantwortet chemische Stimuli auch mit Wrme und Klteempfindungen

A. 1, 2 und 3 sind richtig


B. 1 und 3 sind richtig
C. 2 und 4 sind richtig
D. nur 4 ist richtig
E. alle sind richtig

Richtige Antwort: E

Frage 12
Welche Aussage(n) zur folgenden Abbildung ist (sind) falsch?

1. Die Ziffer 3 zeigt den infralentikulren Teil der Capsula interna, indem die
Radiationes optica und acustica verlaufen.
2. Die Ziffer 1 zeigt den Truncus corporis callosi, der das Cavum vergae nach oben begrenzt.
3. Die Ziffer 4 zeigt den Sulcus calcarinus, Sitz der Sehrinde.
4. Die Ziffer 2 zeigt die Fissura telodiencephalica, die zum ueren Liquorraum
gehrt.
A. 1,2 und 3 sind falsch
B. 1,3 und 4 sind falsch
C. 1,2 und 4 sind falsch
D. 1 und 2 sind falsch
E. 2 und 3 sind falsch

Richtige Antwort: C

Frage 13
Welche der folgenden Strukturen gehren zum Gleichgewichtsorgan?

1. Maccula sacculi
2. Cupula
3. Statholithenmembran
4. Membrana tectoria

A. nur 1, 2 und 3 sind richtig


B. nur 1 und 2 sind richtig
C. alle sind richtig
D. nur 3 und 4 ist richtig
E. nur 1 und 3 sind richtig

Richtige Antwort: A

Frage 14
Fr die Hornersche Trias trifft zu:

A. sie besteht aus hngendem Oberlid, enger Pupille und tiefliegendem Augapfel
B. sie begrndet sich in einer Schdigung des N. vagus
C. sie ist ein typisches Zeichen fr die Schilddrsenberfunktion
D. sie wird unter anderem durch eine Lhmung des M.sphincter pupillae hervorgerufen
E. sie entsteht hufig bei einem Hypophysentumor
Richtige Antwort: A

Frage 15
Welche der folgenden Aussagen ber Keratinozyten sind zutreffend?

1. Sie synthetisieren Melanin.


2. Sie besitzen zahlreiche dendritische Fortstze.
3. Sie stehen durch Desmosomen miteinander in Verbindung.
4. Sie entstehten aus Mesenchymzellen.

A. nur 4 ist richtig


B. nur 3 ist richtig
C. nur 1 ist richtig
D. nur 1 und 2 sind richtig
E. nur 3 und 4 sind richtig

Richtige Antwort: B

Frage 16
Aktivierung des Parasympathicus bewirkt nicht:

A. Erschlaffung der Bronchialmuskulatur


B. Steigerung der Dnndarmmotilitt
C. Kontraktion des M. sphincter pupillae
D. Steigerung der Speicheldrsensekretion
E. Steigerung der Trnendrsensekretion

Richtige Antwort: A

Frage 17
Welche Aussage(n) zur folgenden Abbildung ist (sind) richtig?

1. Die Ziffer 2 zeigt das Corpus mamillare, an dem der Fornix endet.
2. Die Ziffer 3 zeigt den Pons, der den Pes des Myencephalon bildet.
3. Die Ziffer 4 zeigt das Corpus pineale, das zum Epithalamus gehrt.
4. Die Ziffer 1 zeigt den Gyrus paraterminalis, an dem die Stria olfactoria medialis endet.

A. 1, 2 und 3 sind richtig


B. 1 und 3 sind richtig
C. 2 und 4 sind richtig
D. nur 4 ist richtig
E. alle sind richtig

Richtige Antwort: D

Frage 18
Welche Aussage(n zum Cerebellum ist (sind) richtig?

1. Die Fibrae arcuate externae ventrales sind eine indirekte Verbindung des Grohirns mit dem Kleinhirn
und verlaufen ber den unteren Kleinhirnstiel
2. Es ist ein Abkmmling der Grundplatte und gehrt daher zum System der Extrapyramidalmotorik.
3. Das Palocerebellum ist fr die Aufrechterhaltung des Gleichgewichts verantwortlich und entspricht dem
Lobus ant. des Corpus cerebelli.
4. Der Tractus spinocerebellaris post. luft ber den Pedunculus cerebellaris inf.

A. 1 und 3 sind richtig


B. 1 und 4 sind richtig
C. 2 und 4 sind richtig
D. nur 4 ist richtig
E. nur 2 ist richtig

Richtige Antwort: B

Frage 19
Welche Aussage(n) zum Rckenmark ist (sind) richtig?

1. Die beiden Hinterstrnge sind durch das Septum medianum post. das aus Glia besteht, von einander
getrennt.
2. Die Rckenmarkssegmente S1- Co1 liegen auf Hhe des 1./2. Lumbalwirbels.
3. Die Stilling-Clark Sule ist eine Umschaltstelle fr Bahnen der
Oberflchensensibilitt.
4. Die Fasciculi proprii werden von Schaltneuronen gebildet.

A. 1 und 2 sind richtig


B. 2 und 3 sind richtig
C. 3 und 4 sind richtig
D. 1 und 4 sind richtig
E. 1, 2 und 4 sind richtig

Richtige Antwort: A

Frage 20
Welche Aussage(n) zur folgenden Abbildung ist (sind) richtig?

1. Die Ziffer 3 zeigt den Gyrus dentatus, der gemeinsam mit dem Gyrus
parahippocampalis, dem Iduseum griseum und dem Gyrus fasciolaris den ueren Randbogen bildet
2. Die Ziffer 4 zeigt die Fimbria fornicis, die wie der Gyrus dentatus und der Pes
hippocampi zur Hippocampusfromation gehrt.
3. Die Ziffer 2 zeigt den Sulcus collateralis, der die Eminentia collateralis aufwirft.
4. Die Ziffer 1 zeigt die Eminentia collateralis, die den Boden des Cornu inf.
Ventriculis lat. bildet.
A. 1 und 2 sind richtig
B. 2 und 4 sind richtig
C. 1 und 4 sind richtig
D. Nur 4 ist richtig
E. Alle sind richtig

Richtige Antwort: B

Frage 21
Was ist ein Klang?

A. Ein Gemisch aus mehreren Frequenzen, die untereinander in einem ganzzahligen Verhltnis stehen
B. Ein Gemisch aus mehreren Frequenzen, die untereinander in einem beliebigen Verhltnis stehen
knnen
C. Ein Gemisch aus mehreren Tnen mit gleicher Frequenz, deren Amplitude in einem beliebigen
Verhltnis zueinander stehen
D. Ein Gemisch aus mehreren Tnen mit gleicher Frequenz, deren Amplitude in einem ganzzahligen
Verhltnis stehen
E. Ein Gemisch aus mehreren Frequenzen, deren Frequenz zusammen addiert die Grundfrequenz
ergeben

Richtige Antwort: A

Frage 22
Welche Aussage(n) zur Motorik ist (sind) richtig?

1.Die Pyramidenbahn bewirkt eine Hemmung der motor. Vorderhornzelle und fhrt daher bei Ausfall zu
einer schlaffen Phase.
2. Der Tractus corticospinalis ant. verluft nahe der Fissura mediana ant. Und endet im oberen Brustmark.
3.Der Tractus corticonuclearis fr den N.VI hat folgenden Verlauf: Gyrus precentralis, Meditullium, Capsula
interna, Crus cerebri, tegmentum des hirnstamms, ipsilateraler Hirnnervenkern
4.Der rechte Tractus corticonuclearis nervi VII luft zum rechten und linken rostralen Kern und zum linken
dorsalen Kern.

A. 1, 2 und 3 sind richtig


B. 2, 3 und 4 sind richtig
C. Nur 4 ist richtig
D. Nur 3 ist richtig
E. Nur 2 ist richtig
Richtige Antwort: E

Frage 23
Welche Aussage(n) zur Sensibilitt ist (sind) richtig?

1.Die Hinterwurzelfasern entstehen aus den peripheren Neuriten pseudounipolarer Nervenzellen.


2. Die Hinterstrnge leiten Druck und Berhrung protopathisch, Schmerz und Temperatur epikrtisch und die
bewusste Tiefensensibilitt.
3.Der tractus spinothalamicus beginnt in der Substantia gelatinosa Rolandi und ist Nebenbahn fr Druck
und Berhrung.
4.Der Tractus spinocerebellaris bildet das 2. Neuron der unbewussten Tiefensensibilitt.

A. 1 und 2 sind richtig


B. 2 und 3 sind richtig
C. 3 und 4 sind richtig
D. 1 und 4 sind richtig
E. alle sind richtig

Richtige Antwort: C

Frage 24
Welche Aussage(n) zur folgenden Abbildung ist (sind) richtig?

1. Die Ziffer 3 zeigt das Corpus mammillare, das Teil der lateralen Wand des III
Ventrikels ist.
2. Die Ziffer 2 zeigt das Cornu ant. Ventriculi lat., dessen laterale Begrenzung vom Caput n. caudati
gebildet wird.
3.Die Ziffer 4 zeigt das Corpus amygdaloideum, das Teil des limbischen Systems ist.
4. Die Ziffer 1 zeigt den Globus pallidus, der entwicklungsgeschichtlich eine
Abspaltung des Diencepahlon ist.

A. 1 und 4 sind richtig.


B. 1, 2 und 3 sind richtig
C. 1, 2 und 4 sind richtig
D. 3 und 4 sind richtig
E. Nur 4 ist richtig

Richtige Antwort: D

Frage 25
Das Muskeln zur richtigen Zeit mit der richtigen Kraft kontrahieren (timing von
Bewegungen)
1. ist Aufgabe der Gamma-Motoneurone
2. wird ausschlielich im primr motorischen Cortex gesteuert
3. ist Hauptaufgabe des supplementr motorischen Areals
4. ist eine cerebellre Funktion

A. 1, 2 und 3 sind richtig


B. 1 und 3 sind richtig
C. 2 und 4 sind richtig
D. nur 4 ist richtig
E. alle sind richtig

Richtige Antwort: D

Frage 26
Der N. dorsalis scapulae versorgt:

A. den M. scalenus medius


B. den M. sternocleidomastoideus
C. den M. levator scapulae und die Mm. rhomboidei
D. den M. trapezius
E. die Haut an der Dorsalseite der Scapula

Richtige Antwort: C

Frage 27
Die emotionale Bewertung eines Objekts:

1. erfolgt im Temporallappen
2. erfolgt im Okzipitallappen
3. bentigt Afferenzen aus dem Hippocampus
4. bentigt Afferenzen aus dem Basalganglien

A. 1, 2 und 3 sind richtig


B. 1 und 3 sind richtig
C. 2 und 4 sind richtig
D. nur 4 ist richtig
E. alle sind richtig

Richtige Antwort: B

Frage 28
Welche Aussage(n) zur folgenden Abbildung ist (sind) richtig?

1. Die Ziffer 2 zeigt das Trigonum lemnisci, das zum Tegmentum mesencephali
gehrt.
2. Die Ziffer 4 zeigt den Pedunculus cerebellaris medius, der das Kleinhirn mit dem Pons verbindet.
3. Die Ziffer 3 zeigt das Corpus geniculatum laterale, an dem die Radiatio optica
beginnt.
4. Die Ziffer 1 zeigt das Tuber cinerum, die Fortsetzung der Substantia gelatinosa Rolandi des
Rckenmarks.
A. 1, 2 und 3 sind richtig
B. 1, 2 und 4 sind richtig
C. 1 und 4 sind richtig
D. nur 3 ist richtig
E. alle sind richtig

Richtige Antwort: C

Frage 29
Welche Muskeln werden vom N. musculocutaneus versorgt?

1. M. biceps brachii
2. M. brachialis
3. M. coracobrachialis
4. M. deltoideus

A. 1, 2 und 3 sind richtig


B. 1 und 3 sind richtig
C. 2 und 4 sind richtig
D. Nur 4 ist richtig
E. Alle sind richtig

Richtige Antwort: A

Frage 30
Folgende Aussage ber den Augapfel trifft zu:

A. Der Augapfel hat eine zweischichtige Wand.


B. Im Inneren des Augapfels befinden sich Trnenflssigkeit, Linse und Glaskrper
C. Die Tunica vasculosa bulbi wird auch Uvea genannt
D. Der Sehnerv verlsst den Augapfel am gelben Fleck.
E. Am Discus nervi optici befindet sich die Stelle des schrfsten Sehens.

Richtige Antwort: C

Frage 31
Zur Hrbahn zhlen folgende Strukturen:
1. Kerne des Lemniscus lateralis
2. Colliculi inferiores der Lamina tecti
3. Corpus trapezoideum
4. Corpus geniculatum mediale

A. 1, 2 und 3 sind richtig


B. 1 und 3 sind richtig
C. 2 und 4 sind richtig
D. nur 4 ist richtig
E. alle sind richtig

Richtige Antwort: E

Frage 32
Ein 17-jhriges Mdchen leidet darunter, dass es im Gesicht immer wieder
entzndliche Knthcne (Wimmerl, Pickel) hat. Es liegt das klassische Bild einer Acne vulgaris vor.
Welche Struktur ist dafr in erster Linie pathogenetisch relevant?

A. kollagene Fasern
B. Mastzellen
C. Talgdrsen
D. Langerhanszelle
E. Melanozyt

Richtige Antwort: C

Frage 33
Welche Aussage(n) zum Hirnstamm ist (sind) richtig?

1. Der funktionelle Hirnstamm gliedert sich (von vorne nach hinten) in Pes,
Tegmentum und tectum.
2. Das Tegmentum mesencepahli enthlt Kerne der Extrapyramidalmotorik,
Hirnnervenkerne und afferente Bahnen
3. Das Cerebellum ist das Tectum des Rhombencepahlon
4. Zischen Tectum und Tegmentum befindet sich das Ventrikelsystem.

A. 1, 2 und 3 sind richtig


B. 2, 3 und 4 sind richtig
C. 1, 3 und 4 sind richtig
D. 1, 2 und 4 sind richtig
E. alle sind richtig

Richtige Antwort: B

Frage 34
Welche Aussage(n) zum Cerebellum ist (sind) richtig?

1. Der Tractus spinocerebellaris ant. Leitet die unbewusste Tiefensensibilitt der unteren Extremitt und
luft ber den oberen Kleinhirnstiel.
2. Der Tractus nucleocerebellaris fhrt zu Fasern der unbewussten Tiefensensibilitt aus dem N. V und
luft ber den unteren Kleinhirnstiel.
3. Der Tractus cuneocerebellaris entspringt im Ncl. cuneatus accessorious und leitet die unbewusste
Tiefensensibilitt der oberen Extremitt ber den oberen Kleinhirnstiel.
4. Die indirekte sensorische Kleinhirnbahn ist eine Afferenz aus dem Gleichgewichtsorgan und schaltet in
den Ncll. vestibulares um.

A. 1, 2 und 3 sind richtig


B. 1, 2 und 4 sind richtig
C. 2, 3 und 4 sind richtig
D. nur 1 ist richtig
E. nur 2 ist richtig

Richtige Antwort: B

Frage 35
Welche der folgenden Aussagen ber die Paukenhhle (Cavitas tympani) sind
zutreffend?

1. sie gliedert sich in einen knorpeligen und einen kncheren Abschnitt.


2. Sie enthlt Gehrknchelchen.
3. Sie wird durch das Trommelfell vom ueren Gehrgang abgegrenzt.
4. Sie ist mit Endolymphe gefllt.
5. Sie ist von Schleimhaut ausgekleidet.

A. alles sind zutreffend


B. nur 2, 3, 4 und 5 sind zutreffend
C. nur 1, 2 und 3 sind zutreffend
D. nur 2 und 4 sind zutreffend
E. nur 2, 3 und 5 sind zutreffend

Richtige Antwort: E

Frage 36
Die Basalganglien sind beteiligt an:

1.der Koordination von Bewegungsablufen


2.Emotionen
3.motorischem Lernen
4.kognitiven Prozessen

A. 1, 2 und 3 sind richtig


B. 1 und 3 sind richtig
C. 2 und 4 sind richtig
D. nur 4 ist richtig
E. alle sind richtig

Richtige Antwort: E

Frage 37
Bei dem mit X markierten Strukturen in der Abbildung handelt es sich um:
A. Vater- Pacini- Lamellenkrperchen
B. Muskelspindeln
C. Nervenfaserbndel
D. Talgdrsenendstcke
E. Meissner-Tastkrperchen

Richtige Antwort: C

Frage 38
Spinale Motoneurone (MN)!
Welche Aussagen sind richtig?

1. Es existiert eine somatotrope Ordnung hinsichtlich verschiedener Muskelgruppen


2. typischerweise bilden mehrere 100 MN sulenartige motorische Kerne
3. die MN bilden die letzte Endstelle fr alle absteigenden Bahnen
4. die MN bilden auch aufsteigende Bahnen

A. 1,2 und 3 sind richtig


B. 1 und 3 sind richtig
C. 2 und 4 sind richtig
D. nur 4 ist richtig
E. alle sind richtig

Richtige Antwort: A

Frage 39
Welche Aussage(n) zur Sensorik ist (sind) richtig?

1.Die Radiatio acustica verluft vom Corpus geniculatum mediale durch dern sublentikulren Abschnitt der
Capsula interna zum primren akustischen Rindenzentrum in den Gyri temporales transversi.
2.Bei Schdigung des Tractus opticus ensteht eine homonyme kontralaterale
Hemianopsie.
3.Die Ncll. Salivatorious sup. und inf. sind Endkerne fr Geschmacksfasern aus dem Nn. VII, IX und X.
4.Bewusste Geruchsimpulse verlaufen ber den Tractus olfactorious zur Area
olfactoria, wo die Riechrinde liegt.

A. 1, 2 und 3 sind richtig


B. 1 und 2 sind richtig
C. 3 und 4 sind richtig
D. 2, 3 und 4 sind richtig
E. Nur 2 ist richtig

Richtige Antwort: B

Frage 40
Welche Aussage(n) zur Capsula interna ist (sind) richtig?

1. Ansa peduncularis und Commissura ant. sind infralentikulre Bahnen der Capsula interna und verlaufen
durch Gratioletschen Kanal.
2. Pedunculus thalami sup. und Tractus frontopontinus verlaufen im Crus ant. der Capsula interna.
3. Die Capsula interna liegt zwischen Crus cerebri und Corona radiata.
4. Die Capsula interna liegt lateral des Subthalamus.

A. 1 und 2 sind richtig


B. 2 und 3 sind richtig
C. 3 und 4 sind richtig
D. 1 und 4 sind richtig
E. alle sind richtig

Richtige Antwort: C

Frage 41
Welche Aussage(n) zum vegetativen Nervensystem ist (sind) richtig?

1. Im Ganglion submandibulare schalten parasympathisceh Fasern aus dem N.facialis fr die Glandulae
submandibularis und sublingualis um.
2. Aus dem Ncl. salivatorius inf. Stammen die parasympathsichen Fasern des N.X fr die Glandula
parotidea.
3. Die parasympathsiceh Fasern fr die Mm. Ciliaris und dilatator pupillae stammen aus dem Ncl.
accessorious n.III
4. Der Truncus sympathicus reicht vom 8. Halswirbel bis zum 2. Lendenwirbel.

A. nur 1 ist richtig


B. nur 3 ist richtig
C. 1 und 3 sind richtig
D. 2 und 4 sind richtig
E. 2, 3 und 4 sind richtig

Richtige Antwort: A

Frage 42
Die Stria vascularis im Innenohr hat die Aufgabe:

1. einen Druckausgleich durchzufhren


2. Kalium Ionen in die Endolymphe abzugeben
3. die Cochlea dicht abzuschlieen
4. die Haarsinneszellen mit Energie und Sauerstoff zu versorgen

A. 1, 2 und 3 sind richtig


B. 1 und 3 sind richtig
C. 2 und 4 sind richtig
D. nur 4 ist richtig
E. alle sind richtig

Richtige Antwort: C

Frage 43
Welche Aussage(n) zu den Hirnnervenkerenen ist (sind) falsch?
1.Der Ncl. salivatorious sup. ist Endkern fr Geschmacksfasern aus den Nn. VII, IX und X.
2.Der Ncl. pontinus n.V wirft das Tuber cinerum auf und ist Schmerzkern.
3.Der Ncl. motorious n. V ist ein in die Tiefe gelagertes Ganglion und erhlt Fasern aus der Kaumuskulatur
4.Der Ncl. n .VII liegt ventrolateral des Colliculus facialis und seine Fasern bilden um den Abducenskern ein
inneres Facialisknie.

A. 1, 2 und 3 sind falsch


B. 2, 3 und 4 sind falsch
C. nur 4 ist falsch
D. 2 und 4 sind falsch
E. alles sind falsch

Richtige Antwort: A

Frage 44
Obige Abbildung stellt in Kurve 1 den Zeitverlauf des arteriellen Blutdrucks eines
Versuchtieres dar, dem zum Zeitpunkt des Pfeils rasch arteriell Adrenalin injiziert wurde.
(Ausschlag nach oben Zunahme des arteriellen Blutdruck).
Nach Gabe eines Rezeptorenblockers ndert sich der Blutdruck nach derselben Adrenalingabe per Kurve
2.

Welche Rezeptoren sind blockiert worden?

A. nur die adrenergen alpha-Rezeptoren


B. nur die Acetylcholin-Rezeptoren und die adrenergen beta- Rezeptoren
C. nur die acetylcholin-Rezeptoren
D. adrenerge alpha und beta- Rezeptoren
E. nur die adrenergen beta- Rezeptoren

Richtige Antwort: A

Frage 45
Der N. ischiadicus verlsst das Becken durch das:

A. Foramen ischiadicum minus


B. Foramen suprapiriforme
C. Foramen obturatum
D. Foramen infapiriforme
E. Foramen intervertebrale zwischen L5 und S1

Richtige Antwort: D

Frage 46
Fr die Membranleitfhigkeit einer lebenden Zelle am Ruhepotential treffen folgende Aussagen zu:

1. die Kalium - Leitfhigkeit ist sehr hoch


2. die Chlorid - Leitfhigkeit ist hoch
3. die Natrium - Leitfhigkeit ist gering
4. die Leitfhigkeit fr die Eiwei - Anionen ist hoch

A. 1, 2 und 3 sind richtig


B. 1 und 3 sind richtig
C. 2 und 4 sind richtig
D. Nur 4 ist richtig
E. Alles sind richtig

Richtige Antwort: A

Frage 47
Bei einem Mann wird der Fernpunkt bei 2 Meter und der Nahpunkt bei 1 Meter bestimmt:

1. Der Mann ist kurzsichig (myop)


2. Der Mann ist alterssichtig (presbyop)
3. Er hat eine Akkommodationsbreite von 0,5 Dioptrien
4. Er braucht eine Fernsicht (scharfes Sehen ferner Objekte) eine Zerstreuungslinse von 0,5 dpt (also -0,5
dpt)

A. 1, 2 und 3 sind richtig


B. 1 und 3 sind richtig
C. 2 und 4 sind richtig
D. nur 4 ist richtig
E. alle sind richtig

Richtige Antwort: E

Frage 48
Welche der folgenden Abbildungen zur Augenentwicklung ist/sind zutreffend?

1. Das Augenblschen induziert die Entwicklung der Linsenplakode.


2. Der Augenbecher entsteht aus dem Augenblschen
3. Aus der ueren Schicht des Augenbechers entsteht das Pigmentepithel.
4. Die Augenblschen sind Ausstlpungen des Vorderhirns.

A. 1, 2 und 3 sind richtig


B. 1 und 3 sind richtig
C. 2 und 4 sind richtig
D. nur 4 ist richtig
E. alle sind richtig

Richtige Antwort: E

Frage 49
Was stimmt fr die bertragungsfunktion des Mittelohres?

A. Tiefpassfunktion mit einer Grundfrequenz von ca. 1,5 kHz.


B. Hochpassfunktion mit einer Grenzfrequenz von ca. 1,5 kHz.
C. Tiefpassfunktion mit einer Grenzfrequenz von ca. 150 Hz.
D. Bandpassfunktion mit einer Grenzfrequenz von ca. 150 Hz.
E. Hochpassfunktion mit einer Grenzfrequenz von ca. 1500 Hz.

Richtige Antwort: A

Frage 50
Welche Aussage ber die ueren Augenmuskeln trifft zu?

A. Der M.rectus lateralis adduziert den Bulbus oculi.


B. Der M. rectus medialis wird vom N.abducens innerviert.
C. Der M. rectus inferior senkt den Bulbus oculi.
D. Die Hauptaufgabe der Mm. Recti bulbi ist die Rotation des Bulbus oculi.
E. Der 4. Hirnnerv innerviert die ueren Augenmuskeln.

Richtige Antwort: C

Frage 51
Der Fasciculus medialis des Plexus brachialis entlsst folgende Nerven:

A. Radix medialis des N. medianus, N. ulnaris, N.radialis, N.cutaneus brachii


B. Radix medialis des N.medianus, N.cutaneus brachii medialis, N.cutaneus
antebrachii medialis, N.axillaris
C. Radix medialis des N.medianus, N.ulnaris, N.axillaris, N.cutaneus brachii medialis, N.cutaneus
antebrachii medialis
D. N.axillaris, N.radialis, N.ulnaris, Radix medialis des N.medianus
E. Radix medialis des N.medianus, N.ulnaris, N.cutaneus brachii medialis, N.cutaneus antebrachii
medialis

Richtige Antwort: E

Frage 52
Welche Aussage(n) zur Sensibilitt ist (sind) falsch?

1.Ganglion trigeminale und Ganglion geniculi sind sensible Ganglien und bestehen aus pseudounipolaren
Nervenzellen.
2.Nach dem Gesetz der Exzentrizitt von langen Leitungsbahnen liegt der
Fasciculus cuneatus oberflchlich, der Fasciculus gracilis tief.
3.Der Tractus spinothalamicus luft ab der Medulla oblongata im Lemniscis medialis.
4. Der Lemniscus trigeminalis bildet den Hauptteil des Tractus nucleothalamicus und stammt aus den Ncll.
Pontinus und spinalis n.V.

A. 1 und 2 sind falsch


B. 1 und 3 sind falsch
C. 2 und 3 sind falsch
D. 3 und 4 sind falsch
E. Alle sind falsch

Richtige Antwort: C

Frage 53
Welche Aussage ber die in Abbildung mit 1 bezeichneten Struktur ist zutreffend?
A. Sie enthlt typischerweise keine Blutgefe
B. Sie grenzt an den Glaskrper.
C. Sie besteht aus Nervenfasern.
D. Sie besteht aus kollagenfasrigen Bindegewebe.
E. Sie produziert Kammerwasser.

Richtige Antwort: D

Frage 54
Welche der genannten synaptischen Schaltstellen sind Teil der aufsteigenden Hrbahn?

1. Ncl. cochlearis dorsalis


2. Ncl. cochlearis ventralis
3. Ncl. Lemnisci lateralis
4. Colliculus superior

A. 1, 2 und 3 sind richtig


B. 1 und 3 sind richtig
C. 2 und 4 sind richtig
D. nur 4 ist richtig
E. alle sind richtig

Richtige Antwort: A

Frage 55
Welche Aussage(n) zur Extrapyramidalmotorik ist (sind) richtig?

1.Die Ansa lenticularis besteht aus Efferenzen des Pallidum zum Thalamus
2.Die Fasciculus lenticularis besteht aus Efferenzen des Pallidum zum Thalamus.
3.Die Ansa lenticularis verluft durch die Forelschen Felder H1 und H2.
4. Der Fasciculus lenticularis verluft durch die Forelschen Felder H1 und H2.

A. 1, 2 und 3 sind richtig


B. 1, 2 und 4 sind richtig
C. 2, 3 und 4 sind richtig
D. 2 und 4 sind richtig
E. Nur 1 ist richtig

Richtige Antwort: B

Frage 56
In Folge der Dunkeladaption erfolgt:

1. Skoptische Sehen
2. Abnahme der retinalen Rhodopsinkonzentration
3. Umschichtung der rezeptiven Felder
4. Verbesserung der Sehschrfe im Vergleich zur Helladaption

A. 1, 2 und 3 sind richtig


B. 1 und 3 sind richtig
C. 2 und 4 sind richtig
D. nur 4 ist richtig
E. alle sind richtig

Richtige Antwort: B

Frage 57
Die Abbildung zeigt einen Ausschnitt aus dem vorderen Augenbulbus.
Welche Zuordnung/en Buchstabe-Struktur ist/sind richtig?

1. A - Kornea
2. B - Schlemm-Kanal
3. C - Sklera
4. D - M.ciliaris
5. E - Iris

A. 2, 4 und 5 ist richtig


B. 1, 2, 3 und 4 sind richtig
C. Alle sind richtig
D. Nur 2 ist richtig
E. 2, 3, 4 und 5 sind richtig

Richtige Antwort: A
Frage 58
Welche der folgenden Aussagen zum Trommelfell ist/sind zutreffend?

1. Es wird durch Schallwellen in Schwingung versetzt.


2. Es enthlt kollagene Fasern.
3. Es ist auf beiden Seiten von Epithel berzogen.
4. Es ist typischerweise geflos.

A. 1, 2 und 3 sind richtig


B. 1 und 3 sind richtig
C. 2 und 4 sind richtig
D. nur 4 ist richtig
E. alle sind richtig

Richtige Antwort: A

Frage 59
Welche Aussage(n) zu den Fasertypen ist (sind) richtig?

1. Der Fasciculus arcuatus besteht aus Assoziationsfasern und windet sich um das hintere Ende des
Sulcus lateralis
2. Das Corpus callosum ist eine Kommissur des Isokortex und verbindet identische Areale beider
Hemisphren
3. Durch die Gratioletschen Kanal verlaufen die Ansa lenticularis, der Pedunculus thalami inf. und die
Commissura ant.
4. Die gesamte Sensibilitt luft ber den Pedunculus thalami sup. zur Krperfhlsphre.

A. 1, 2 und 4 sind richtig


B. 2, 3 und 4 sind richtig
C. 1 und 4 sind richtig
D. 2 und 3 sind richtig
E. alle sind richtig

Richtige Antwort: E

Frage 60
Welche Aussage(n) zum Diencephalon ist (sind) richtig?

1. Der Ventriculus Monroi ist der vorderste Teil des III.Ventrikels und endet vor dem Telencephalon
medium.
2. Durch das Forelsche Feld H2 verlaufen die Ansa lenticularis und der Fasciculus lenticularis.
3. Die Fissura transversa cerebri gehrt zum ueren Liqourraum.
4. Die Stria medullaris thalami gehrt zum limbischen System und geht hinten in das Trigonum habenulae
ber.

A. 1, 2 und 3 sind richtig


B. 1, 2 und 4 sind richtig
C. 1, 3 und 4 sind richtig
D. nur 2 ist richtig
E. alle sind richtig

Richtige Antwort: C

Das könnte Ihnen auch gefallen